You are on page 1of 111

Capital Gains

[Income Tax]
Assessment Year 2016-17

Complete coverage of tax provisions for Capital Gains


in Question & Answer format with practical illustration
Compilation of
CMA Suggested Answers [December 13 to June 15]
CA IPCC Suggested Answers [May 98 to November 15]

T K SRIDHAR

Price: 150
CALL OR VISIT FOR COPIES
Published by
SINGAR BOOKS AND PUBLICATIONS
A unit of SINGAR ACADEMY
Head Office: 52, Salai Road, Woriur, Trichy 620 003, Tamil Nadu
Branch Office: 76/1, New Street, Valluvar Kottam High Road,
Nungambakkam, Chennai 600 034
Ph: Trichy: 0431-4024489 | 93451 22645 | 93446 04489 & Chennai: 93453 96855
www.singaracademy.com | singaracademy@yahoo.com

10. CAPITAL GAINS


Content
Section

Important Definitions in Capital Gains

Page

2(14)

Capital Asset (CA)

10.4

2(29A)

Long term Capital Asset (LTCA)

10.5

2(29B)

Long term capital gains (LTCG)

10.6

2(42A)

Short Term Capital Asset (STCA)

10.6

2(42B)

Short Term Capital Gains (STCG)

10.6

2(42C)

Slump sale

10.6

2(47)

Transfer

10.6

2(48)

Zero Coupon Bond

10.8

10(33)

Exemption of capital gain on transfer of US64

10.8

10(36)

Exemption in respect of long-term capital gains on transfer of listed shares

10.8

10(37)

Exemption on compulsory acquisition of agriculture land

10.9

10(38)

Exemption of LTCG from listed equity shares u/h capital gain

10(41)

Exemption of capital gain for power sector companies

10(43)

Exemption of reverse mortgage

10.10

45(1)

General computation of short term and long term capital gains

10.12

45(1A)

Insurance claim received on loss of assets

10.13

45(2)

Conversion of capital asset into stock-in-trade

10.14

45(2A)

Sale of shares held as depository

10.15

45(3)

Introduction of capital asset by a partners into the firm or by a member into the AOP or

10.16

10.10
-

BOI
45(4)

Distribution of capital asset to partners / members on dissolution of firm / AOP / BOI

10.17

45(5)

Compulsory acquisition of capital asset by Govt. / Approved authority

10.18

(a) Normal Compensation and (b) Enhanced Compensation


45(6)

Repurchase of units by company

10.19

46

Capital gains on distribution of assets by companies in liquidation

10.20

46A

Capital gains on purchase by company of its own shares or other specified securities

10.22

47

Transactions not regarded as transfer

10.23

47A

Withdrawal of exemption in certain cases

10.29

48

Mode of computation

10.11

49

Cost with reference to certain modes of acquisition

10.29

50

Special provision for computation of capital gains in case of depreciable assets

10.32

50A

Transfer of Depreciable Assets by Power Sector Units

10.33

50B

Sale or undertaking as a going concern or Slump Sale

10.33

50C

Transfer of Land or Building or both at less than stamp duty value

10.36

50D

If consideration received or accruing as a result of transfer of a capital asset is not

10.37

ascertainable or cannot be determined


51

Forfeiture of Advance Money received in respect of unfructified transfer

10.37

54

Profit on sale of property used for residence

10.47

54B

Capital gain on transfer of land used for agricultural purposes not to be charged in

10.47

certain cases
54D

Capital gain on compulsory acquisition of lands and buildings not to be charged in

10.47

certain cases
54EC

Capital gain not to be charged on investment in certain bonds

10.47

54F

Capital gain on transfer of certain capital assets not to be charged in case of investment

10.47

in residential house
54G

Exemption of capital gains on transfer of assets in cases of shifting of industrial

10.47

undertaking from urban area


54GA

Exemption of capital gains on transfer of assets in cases of shifting of industrial

10.47

undertaking from urban area to any Special Economic Zone


54GB

Capital gain on transfer of residential property

10.47

54H

Extension of time-limit for acquiring new asset

10.49

55

Meaning of adjusted, cost of improvement and cost of acquisition for the purpose of

10.39

S.48 & 49
55A

Reference to Valuation Officer

10.44

111A

Short-term Capital Gain on Listed Equity Shares

10.46

115A

Computation of Capital Gains for Non-resident

10.45

10. CAPITAL GAINS


Cost inflation index for different previous years
PY

CII

PY

CII

PY

CII

PY

CII

1981-82

100

1990-91

182

1999-00

389

2008-09

582

1982-83

109

1991-92

199

2000-01

406

2009-10

632

1983-84

116

1992-93

223

2001-02

426

2010-11

711

1984-85

125

1993-94

244

2002-03

447

2011-12

785

1985-86

133

1994-95

259

2003-04

463

2012-13

852

1986-87

140

1995-96

281

2004-05

480

2013-14

939

1987-88

150

1996-97

305

2005-06

497

2014-15

1024

1988-89

161

1997-98

331

2006-07

519

2015-16

1081

1989-90

172

1998-99

351

2007-08

551

IMPORTANT DEFINITIONS IN CAPITAL GAINS


Section
2(14)

Description
Capital Asset (CA)

Definition
Includes property of any kind except stock, personal effects, Rural
agricultural land etc. [Jewellery is a capital asset]

2(29A)

Long term Capital

Not a short term capital asset

Asset (LTCA)
2(29B)

Long term capital

Capital gains arising from transfer of LTCA

gains (LTCG)
2(42A)

2(42B)

Short Term Capital

Financial Capital Assets held for not more than 12 months

Asset (STCA)

Other Capital Assets Held for not more than 36 months

Short Term Capital

Capital Gains arising from transfer of STCA

Gains (STCG)
2(42C)

Slump sale

2(47)

Transfer

Sale, Exchange, relinquishment, Extinguishment of rights, Compulsory


acquisition, Conversion of CA into stock in trade, Maturity / Redemption
of zero coupon bond, Part Performance of contract, enjoyment of
immovable property.

2(48)

Zero Coupon

Bond issued by Infrastructure capital Company or Infrastructure Capital

Bond

Fund or Public Sector Company or w.r.e.f AY 2009-10, Scheduled Banks


on or after 01.06.2005 with no benefit before maturity or redemption

Capital Gains

10.1

IMPORTANT EXEMPTIONS IN CAPITAL GAINS


Section

Important Provisions

10(33)

Capital gain on transfer of US64

10(36)

Exemption in respect of long-term capital gains on transfer of listed shares

10(37)

Exemption on compulsory acquisition of agriculture land

10(38)

Exemption of LTCG from listed equity shares u/h capital gain

10(41)

Exemption of capital gain for power sector companies

10(43)

Exemption of reverse mortgage

Content

Computation

45(1)

Computation of capital gain

CG = SC CA or ICA

YOT

45(1A)

Insurance claim received on loss of assets

CG = CR CA or ICA

YOR

45(2)

Conversion of capital asset into stock-in-trade.

CG = FMV CA or ICA;

YOTCS

(Note: indexation on year of conversion)

BI = SC FMV

Sale of shares held as depository

CG = SC CA or ICA

YOT

CG = BV CA or ICA

YOI

CG = FMV CA or ICA

YOD

CG = NCR CA or ICA

YORI

45(2A)

(FIFO method shall be adopted)


45(3)

Introduction of capital asset by a partner


(member) into the firm (AOP or BOI)

45(4)

Distribution of capital asset to partners


(members) on dissolution of firm (AOP or BOI)

45(5)

Compulsory acquisition of capital asset by Govt.


/ Approved authority
(a) Normal Compensation
(Indexation upto year of compulsory acquisition)
(b) Enhanced Compensation

CG = ECR - Expenses incurred

(c) Reduction of enhanced compensation

Re-compute (b)

45(6)

Repurchase of units (80CCB) by company

CG = BBP CA

46

Capital gains on distribution of assets


by companies in liquidation

46A

BBP Buy Back Price

FMV of assets received

Cash received

Dividend (S.2(22)(c))

CA or ICA

CG

CG = BBP CA or ICA

Buy back of shares or specified securities

YOR
YOR

YOB

NCR Normal Compensation Received

Income Tax

10.2

BV Book Value

SC Sale Consideration

CA Cost of Acquisition

YOD Year of Distribution

CG Capital Gain

YOI Year of Introduction

CI Cost of Improvement

YOR Year of Receipt

CR Claim Received

YORB Year of Repurchase or Buyback

ECR Enhanced Compensation Received

YORI Year of Receipt of I installment

ICA Indexed Cost of Acquisition

YOT Year of Transfer

ICI Indexed Cost of Improvement

YOTCS Year of Transfer of Converted Shares

S
47

Transactions not regarded as transfer

47A

Withdrawal of exemption in certain cases

48

Mode of computation

49

Cost with reference to certain modes of acquisition

S
50

50A

Computation of capital gains in case of

CG = SC less (Expenses on transfer + opening

depreciable assets

WDV + additions during the year)

Transfer of Depreciable Assets by Power

CG = SC less Original CA

YOT

Sale or undertaking as a going concern or

CG = Lump-sum SC Less Net Worth

YOT

Slump Sale

(No indexation for LTCG)

Transfer of Land or Building or both at less

CG = Value determined by Stamp Duty

than stamp duty value

Authority less CA or ICA

If Sale Consideration (SC) of capital asset is

SC = FMV on the date of transfer

not ascertainable

[from AY 2013-14]

Forfeiture of Advance Money received in

The advance is reduced from Cost of

respect of unfructified transfer

Acquisition / WDV/ FMV

YOT

Sector Units
50B
50C
50D
51

YOT

S
54

Profit on sale of property used for residence

54B

Capital gain on transfer of land used for agricultural purposes not to be charged in certain cases

54D

Capital gain on compulsory acquisition of lands and buildings not to be charged in certain cases

54EC

Capital gain not to be charged on investment in certain bonds

Capital Gains

10.3

54F

Capital gain on transfer of certain capital assets not to be charged in case of investment in
residential house

54G

Exemption of capital gains on transfer of assets in cases of shifting of industrial undertaking from
urban area

54GA

Exemption of capital gains on transfer of assets in cases of shifting of industrial undertaking from
urban area to any Special Economic Zone

54GB

Capital gain on transfer of residential property

54H

Extension of time-limit for acquiring new asset

55

Meaning of adjusted, cost of improvement and cost of acquisition for the purpose of S.48 & 49

55A

Reference to Valuation Officer

DEFINITIONS
[CMA RTP D11][CS EP D09 & J12, 5 marks & 3 marks]
Question: What are capital assets? What are the items not included in capital assets [S.2(14)]?
Answer: Capital assets mean property of any kind held by the assessee, whether or not connected with
his business or profession, but does not include.
1

Any stock in trade, consumable stores or raw materials held for the purpose of business or profession.

Personal effects of the assessee. It means moveable property including wearing apparel and furniture
held for his personal use or for the use of any member of his family dependent upon him but exclude
(a) Jewellery (b) Painting (c) Archaeological Collection (d) Drawing (e) Sculptures (f) any work of art
even though it is meant for personal use of assessee

Rural agricultural land in India.

6% Gold Deposit Bonds, 1977 or 7% Gold Bonds, 1980 or National Defense Gold Bonds, 1980 issued
by Central Government.

Special Bearer Bonds, 1991

Gold Deposit Bonds issued under Gold Deposit Scheme, 1999

Important Notes:
1.
(a)

"Jewellery" is a capital asset. For the purpose of this chapter, Jewellery includes the following:Ornaments make of silver, gold, platinum, or any other metal or any alloy containing one or more of
such precious metal, whether or not containing any precious or semi-precious stones and whether or
not worked or sewn into any wearing apparel.

(b) Precious or semi-precious stones, whether or not set in any furniture, utensil of other article or
worked or sewn into any wearing apparel.
2.

Agricultural land situated in rural area is not a capital asset: For this chapter, rural area means the
land situated:

(a) In any area within the territorial jurisdiction of a municipality having a population of 10,000 or more;

Income Tax

10.4

(b) In any area specified by the government.


3.

Examples of personal effect are furniture, car, scooter etc. Silver bars, rupee coins for pooja, festivals
etc is capital asset and not personal effect.

4.

Personal effects of movable assets are not capital assets are not a capital asset. However personal
effects of immovable assets are capital assets and hence capital gain arises on the sale of residential
houses etc.

[CMA RTP J12 & D12]


Question: Ritu received a gift, from her mother, 6 % Gold Bonds of the value of 5 lakhs in 1980. These
bonds were redeemed by the Government on 1.10.2003 and he received gold of equivalent value,
weighing 5,000 grams approximately of fair market value of 10 lakhs. The gold was sold by him on
1.7.2015 for 24 lakhs. Examine the impact of the transactions in Ritus assessment.
Answer: Vide Circular No.415 dated 14.3.1985 [152 ITR (St.) 205], exchange of gold bonds for gold on
redemption does not attract Capital gains because it is not a capital asset. In case of subsequent sale of
such gold, capital gain is chargeable to tax. For this purpose:
Date of acquisition = date of redemption of such gold bonds;
Cost of acquisition of gold = market value of the gold on the date of redemption.

Computation of Capital Gains on sale of gold

Less

Consideration for transfer of gold

24,00,000

Indexed Cost of Acquisition [10,00,000 1081/463]

23,34,773

Long Term Capital Gains

65,227

[CMA RTP J12 & D12]


Question: A farmer resident of Bikaner sold his rural agricultural land in Nepal and received 5 lacs
over the cost of acquisition of this land. Explain the taxability of sale.
Answer: U/s 2(14), only rural agricultural lands in India are not a capital asset. In this given case, the
farmer has sold rural agricultural lands in Nepal and therefore, the transaction attracts the provisions of
capital gains.

[CMA RTP J12 & D12]


Question: A Plantation company, holding several acres of land, sold trees of spontaneous growth. The
Assessing Officer is of the opinion that the capital gain arises. Discuss
Answer: Sale proceeds of spontaneous growth will not result in capital gains, as they do not bring in any
profit or gain [Suman Tea & Plywood Industries Pvt. Ltd (1997) 226 ITR 34 (SC)].

Question: Write short note on long term capital asset as defined u/s 2(29A)?

Capital Gains

10.5

Answer: A capital asset which is not a Short Term Capital Asset is a Long term Capital Asset. In other
words, if the asset is held for more than 36 months or 12 months, as the case may be, such an asset will be
treated as Long Term Capital Assets.

Question: Write Short note on Short Term Capital Assets as defined u/s 2(42A)?
Answer: A capital asset held by an assessee for not more than 36 months immediately preceding the date
of its transfer is known as a short term capital asset.
However, the following assets shall be treated as short term capital assets if the period held is not more
than 12 months immediately preceding the date of its transfer.
a.

Equity or Preference shares in a company (listed in a recognized stock exchange in India)

b.

Any other securities listed in the recognised stock exchange in India.

c.

Units of UTI or Units of an equity oriented mutual fund (whether quoted or not)

d. Zero Coupon Bonds whether quoted or not.

Question: Write Short note on Capital Gains?


Answer: Since there are two types of capital assets, there will be two types of capital gains.
a.

Short Term Capital Gain as defined u/s 2(42B) of the Income Tax Act, 1961-It implies that any profit
or gains arising from the transfer of short term capital assets is called short term capital gain.

b.

Long Term Capital Gain as defined u/s 2(29B) of the Income Tax Act, 1961-it implies that any profit or
gains arising from the transfer of long term capital assets is called long capital gain.

Question: Define slump sale


Answer: S.2(42C) "slump sale" means the transfer of one or more undertakings as a result of the sale for a
lump sum consideration without values being assigned to the individual assets and liabilities in such
sales.

[CS EP1, D00, 5 marks]


Question: Write short note on definition of transfer u/s 2(47).
Answer: "Transfer" in relation to capital assets includes:a.

The sale, exchange, or relinquishment of the asset; or

b.

The extinguishment of any rights therein; or

c.

The compulsory acquisition thereof under any law; or

d. In a case where the asset is converted by the owner thereof into, or is treated by him as stock-in-trade
of a business carried on by him, such conversion or treatment; or
e.

Any transaction involving the allowing of the possession of any immovable property to be taken or
retain in the part performance of the contract as referred in Section 53A of the Transfer of Property
Act, 1882.

Income Tax

10.6

f.

Allotment or lease under a housing building scheme of society, company or other association.

g.

The maturity or redemption of Zero Coupon bonds.

Important Points:
a.

Surrender of preference shares on redemption thereof amount to 'transfer' as there is


relinquishment by the shareholder of his rights in preference shares. Decided in the case of Anarkali
Sarabhai (SC)

b.

Reduction of face value of share and consequent payment to the shareholder towards such
reduction amount to transfer as it results in extinguishments of right in the shares held by the
shareholder. Decided in the case of Kartikeya Sarabhai (SC)

Question: When does the transfer complete and effective?


Answer:
1.

Movable property: title to a movable property passes at the time when property is delivered
pursuant to a contract to sell. Entries in the books of account are not relevant for determining date of
transfer.

2.

Immovable property:
(a) immovable property when documents are registered: transfer is compete on registration.
(b) immovable property when documents are not registered: transfer is complete on satisfaction of
conditions laid u/s 53A of the Transfer of Property Act, which are listed below.
1.

there should be a contract in writing

2.

the transferee has paid consideration or is willing to perform his part of the contract and

3.

the transferee should have taken possession of the property.

[CMA INTER SY12,D13, 2 Marks]


Question: Is the right of management in an Indian company a capital asset? On relinquishment directly
or indirectly, is it liable to tax?
Answer: Yes, it is a capital asset in view of the Explanation to section 2(14) inserted by the Finance Act,
2012. It is a 'transfer' in view of Explanation 2 to section 2(47). Hence the relinquishment is liable to capital
gains tax.

[CA INTER N06, 1 Mark]


Question: Fill up the blanks: As per section 2(47), ______ or ________ of a zero coupon bond will be
treated as transfer for the purpose of capital gains tax. 1

Answer: maturity, redemption.

Capital Gains

10.7

[CMA RTP J12 & D12]


Question: Amit owns a plot of land acquired on 1.7.2005 for a consideration of 4 lakhs. He enters into an
agreement to sell the property on 23.3.2016 for a consideration of 11 lakhs. In part performance of the
contract, he handed over the possession of land on 25.3.2016 on which date, he received the full
consideration. As on 31.3.2016, the sale was pending registration. Discuss liability of capital gains for the
assessment year 2016-17 (no computation is required)
Answer: U/s 2(47), transfer includes part performance of a contract of the nature specified in S.53A of the
Transfer of Property Act. In the given case, consideration was received by Amit and the possession was
handed over on 25.3.2016 hence, the part performance condition is satisfied. Capital gain on the above
transaction is chargeable to tax as income for the assessment year 2016-17.

[CS EP D09 & J10, 3 Marks & 3 Marks]


Question: Write Short note on taxability of Zero Coupon Bonds?
Answer: Meaning of Zero Coupon Bond: Section 2(48) defines the expression Zero Coupon Bond as
bond issued by any infrastructure capital company or infrastructure capital fund or a public sector
company on or after 1st June, 2005, in respect of which no payment and benefit is received or receivable
before maturity or redemption from such issuing entity.
Tax Implications: The income of transfer of a ZCB (not being held as stock-in-trade) is to be treated as
capital gains. S.2(47)(iva) provides that maturity or redemption of ZCB shall be treated as a transfer for
the purposes of capital gains tax
Nature of Capital Gain: ZCBs held for not more than 12 months would be treated as short term capital
assets. Where the period of holding of ZCBs is more than 12 months, the resultant long term capital gains
arising on maturity or redemption would be treated in the same manner as applicable to capital gains
arising from the transfer of other listed securities or units covered by section 112.

EXEMPTIONS
Question: Explain capital gain on transfer of US64 [S.10(33)]
Answer: Any income arising from the transfer of a capital asset being a unit of US64 is not chargeable to
tax where the transfer of such assets takes place on or after April 1, 2002. This rule is applicable whether
the capital asset (US64) is long term capital asset or short term capital asset.

[CA INTER N04, 4 Marks]


Question: State the provisions relating the exemption in respect of long-term capital gains on transfer
of listed Equity Shares.
Answer: Section 10(36), inserted by Finance Act, 2003 w.e.f. A.Y. 2004-05, provides that any income
arising from the transfer of a long-term capital asset, being an eligible equity share in a company shall be

Income Tax

10.8

exempt provided that these were acquired on or after 1.3.2003 but before 1.3.2004 and held for a period of
12 months or more; Eligible equity share means:
(i) Any equity in a company being a constituent of BSE-500 Index of the Stock exchange. Mumbai as on
the 1.3.2003 and the transactions of purchase and sale of such equity share are entered into on a
recognized stock exchange in India.
(ii) Any equity share in a company allotted through a public issue on or after the 1.3.2003 and listed in a
recognised stock exchange in India before 1.3.2004 and the transaction of sale of such share is entered
into or a recognizes stock exchange in India.

Question: Write Short note on exemption on compulsory acquisition of agriculture land u/s 10(37)?
Answer: In the case of an individual or HUF, an income from the transfer of urban agricultural land shall
be exempt if
(a) such land was being used for agricultural purposes by such Individual or HUF or his parents, during
two years immediately preceding the date of transfer.
(b) transfer is by way of compulsory acquisition under any law, or consideration is determined by
Central Government or RBI
(c) such income has arisen from the compensation received by assessee on or after 1 st April, 2004.

[CA INTER M06, 4 Marks]


Question: X is in possession of agricultural land situated within urban limits, which is used for
agricultural purposes during the preceding 3 years by his father. On 4.4.2015 this land is compulsorily
acquired by the Government of India on a compensation fixed and paid by it 10 lakhs. Advise X as to the
tax consequences, assuming that the entire amount is invested in purchase of shares.
Answer: Section 10(37) exempts the capital gains arising to an individual or a Hindu Undivided
Family from transfer of agricultural land by way of compulsory acquisition, or a transfer, the
consideration for which is determined or approved by the RBI or the Central Government.
Such exemption is available where the compensation or the enhanced compensation or
consideration, as the case may be, is received on or after 1 st April, 2004 and the land has been used
for agricultural purposes during the preceding two years by such individual or a parent of his or by
such Hindu undivided family.
Since all the above conditions are fulfilled in this case, X is entitled to exemption u/s 10(37) of the entire
capital gains arising on sale of agricultural land.

[CA INTER N05, 2 Marks]


Question: State whether True or False: Where an urban agricultural land owned by an individual,
continuously used by him for agricultural purposes for a period of two years prior to the date of transfer,
is compulsorily acquired under law and the compensation is fixed by the State Government, resultant
capital gain is exempt.

Capital Gains

10.9

Answer: False. As per Section 10(37), where an individual owns urban agricultural land which has been
used for agricultural purposes for a period of two years immediately preceding the date of transfer, and
the same is compulsorily acquired under any law and the compensation is determined or approved by
the Central Government or the Reserve Bank of India, resultant capital gain will be exempt. In this case,
however, the compensation has been fixed by the State Government and hence the exemption will not be
available.

Question: Write Short note on exemption of LTCG from listed equity shares u/s 10(38) u/h capital
gain?
Answer: Long term capital gain from transfer of equity shares or units of equity - oriented mutual fund
shall be exempt if i.

They are sold through recognized stock exchange.

ii. Security transaction tax is chargeable on such transaction.


iii. Transfer took place on or after 01.10.2004
"Equity oriented fund" means a setup under mutual fund scheme specified u/s 10(23D) and whose
investible funds are invested in the equity shares in the domestic companies to the extent of more than
65% of the total proceeds of such fund.

[CA INTER N09, 3 Marks][CMA RTP J12]


Question: Write short note on reverse mortgage? [S. 10(43)]
Answer: Exemption of income received in a transaction of reverse mortgage [S.10(43)]. This scheme is for
the benefit of senior citizens, who own a residential house property. In order to supplement their existing
income, they can mortgage their house property with a scheduled bank or housing finance company, in
return for a lump-sum amount or for a regular monthly / quarterly / annual income.
The senior citizens can continue to live in the house and receive regular income without the botherations
of having to pay back the loan. The loan will be given up to say, 60% of the value of residential house
property mortgaged. Also, the bank / housing finance company would undertake a revaluation of the
property once every 5 years. The borrower can use the loan amount for extension of residential property,
family's medical and emergency expenditure etc. amongst other. However, he cannot use the amount for
speculative or trading purposes. The bank will recover the loan along with the accumulated interest by
selling the house after the death of the borrower. The excess amount will be given to the legal heirs.
However, before resorting to sale of the house, preference will be given to the legal heirs to repay the loan
and interest and get the mortgaged property released.

[CA INTER M09, 3 Marks]


Question: Mr. Abhiks father, who is a senior citizen had pledged his residential house to a bank under a
notified reverse mortgage scheme. He was getting loan from bank in monthly installments. Mr. Abhiks
father did not repay the loan on maturity and given possession of the house to the bank to discharge his
loan. How will the treatment of long-term capital gain be made on such reverse mortgage transaction?

Income Tax

10.10

Answer: Reverse Mortgage - At the time of pledging the property to the bank, capital gain is not
chargeable to tax. There is no capital gain tax liability in the hands of Abhiks Father when bank gives
loan or monthly installment. When possession of the property is given to the bank, there is no transfer
and, consequently, capital gain is not chargeable to tax. However, when bank transfers the property to
realize the amount of loan and interest, capital gain will be taxable as the transaction is treated as
transfer. If this transfer takes place during the lifetime of Abhiks Father (generally in the case reversed
mortgage, bank transfers the property after the death of the person who has pledged the property),
capital gain would be taxable in the hand of Abhiks Father.
If transfer by the bank takes place after the death of Abhiks father then capital gain will be taxable in the
hands of legal heirs of Abhiks father.

S.48: Mode of Computation


Question: Write short note on method of calculation of capital gains u/s 48?
Answer:
Computation of Short Term Capital Gain
Consideration Received

Expenses of Transfer

Net Consideration

Less

Cost of Acquisition

Less

Cost of Improvement

Short Term Capital Gain

Exemption u/s 54B, 54D, 54G, 54GA

Taxable STCG

Less

Less

Computation of Long Term Capital Gain


Consideration Received
Less

Expenses of Transfer
Net Consideration

Less

Indexed Cost of Acquisition

Less

Indexed Cost of Improvement

Long Term Capital Gain

Exemption u/s 54 to 54GA

Less

Taxable LTCG

Question: Write short note on concept of indexation in case of long term capital gains?
Answer: If the capital gain arises from the transfer of a long term capital asset, then for the purpose of
computing capital gains:
i.

'Indexed Cost of Acquisition' is taken instead of 'Cost of Acquisition (COA)'.

ii. 'Indexed Cost of Improvement' is taken instead of 'Cost of Improvement (COI)'.

Capital Gains

10.11

Note: No deduction will be allowed in respect of payment of securities transaction tax in computing
income under the head capital gains.
In the following cases benefit of indexation is not allowed:Bond or Debentures (Other than capital indexed bond issued by Government)
Share or Debenture of Indian Co. acquired by utilizing convertible foreign exchange (first proviso of
section 48)
Depreciable assets
Slump Sale under Selection 50B
Transfer of units purchased in foreign currency by an assessee covered u/s 115AB
Transfer of GDR's purchased in foreign currency by an assessee covered u/s 115ACA
Transfer of Securities by Foreign Institutional Investors (FII) u/s 115AD

Question: Write Short note on cost of improvement u/h capital gain?


Answer: Cost of improvement incurred before 01.04.1981 shall be ignored for all cases.
All expenditure incurred on improvement of assets by the assessee or the previous owner on or after
01.04-1981 shall be treated as cost of improvement.

S.45(1): General Computation of Capital Gains


[CMA RTP J12]
Question: What conditions must be satisfied for the purpose of taxing capital gain u/s 45(1)?
Answer: Any profits or gains arising from the transfer of capital assets effected in the previous year, shall
be chargeable to income tax under the head 'capital gain' and shall be deemed to be the income of the
previous year in which the transfer took place unless such capital gain is exempt u/s 54, 54B, 54D, 54EC,
54ED, 54F, 54G and 54GA
In other words, capital gains tax liability arises only when the following conditions are satisfied
1

There should be Capital Asset.

The capital asset is transferred by the assessee.

Such transfer takes place during the previous year

Any profit or gain arises from the result of transfer

Such profit or gain is not exempt from tax u/s 54, 54B, 54D, 54EC, 54ED, 54F, 54G and 54GA

All of the above conditions must be satisfied at a one point of time for the purpose of taxing capital gains.

[CMA INTER SY08, D13, 1 Mark]


Question: In case of transfer of a capital asset by an assessee who acquired the same from his father by
way of gift, the cost of acquisition of such asset for the purpose of computing capital gain is1
1

Answer: (b)

Income Tax

10.12

a) Nil
b) Cost of acquisition of the asset to father
c)

Fair market Value of the asset on the date of gift

d) Arm's length price of the asset on the date of gift

S.45(1A): Computation of Capital Gains [Insurance Claim]


Question: Write short note on insurance claim received u/s 45(1A) u/h capital gains?
Answer: If any person receives any money or other assets on account of insurance claim from the
insurance claim from the insurer on account of damage to, or destruction of, any capital asset from
Flood, typhoon, hurricane, earthquake & other convulsion of nature
Riot or civil disturbance
Accidental fire or explosion
Action by enemy or action taken in combating an enemy
Then any profit from the receipt of such money shall chargeable to capital gain and shall be deemed to be
the income of such person of the previous year in which such money or asset was received and money
received or FMV of the asset received, shall be deemed to be full consideration.
i.

Full value of consideration = Money received or FMV of asset on date of receipt.

ii. Total period of Holding = Date of acquisition to date of destruction.


Capital Gain = Insurance Money Received or FMV of Asset minus COA, COI or ICOA and ICOI.
If compensation received for depreciable asset, then provision of S.50 shall apply.

[CA INTER N06, 7 Marks]


Question: Mr. A, is an individual carrying on business. His stock and machinery were damaged and
destroyed in a fire accident. The value of stock lost (totally damaged) was 6,50,000. Certain portion of
the machinery could be salvaged. The opening WDV of the block as on 1-4-2015 was 10,80,000.
During the process of safeguarding machinery and in the firefighting operations, Mr. A lost his gold
chain and a diamond ring, which he had purchased in April, 2009 for 1,20,000. The market value of these
two items as on the date of fire accident was 1,80,000.
Mr. A received the following amounts from the insurance company:
1

Towards loss of stock

4,80,000

Towards damage of Machinery

6,00,000

Towards gold chain and diamond ring

1,80,000

You are requested to briefly comment on the tax treatment of the above three items under the provisions
of the income-tax Act, 1961.
Answer: Tax Treatments:

Capital Gains

10.13

Stock
Sale Consideration (Compensation)

4,80,000

Less

Cost of acquisition

6,50,000

Less

WDV of the block (assumed)

Less

Indexed cost of acquisition *

Machinery

Gold & Diamond

6,00,000

1,80,000

10,80,000
+

Business income

2,05,253
(1,70,000)

Short-term capital gain (loss)

(4,80,000)

Long-term capital gain (loss) [S.45(1A)]

(25,253)

S.45(2): Computation of Capital Gains [Conversion of CA into Stock]


[CA INTER M00, M07 & N12, 5, 6 & 4 Marks][CS EP J10, 4 Marks]
Question: Discuss the tax implications arising consequent to conversion of a capital asset into
stock-in-trade of business and its subsequent sale.
Answer: Capital Gain from conversion of capital asset into Stock in Trade shall be charged to tax in the
previous year in which the Stock in Trade is sold or otherwise transferred by the assessee
In the above case, FMV of the assets on the date of conversion of capital assets into stock in trade shall be
taken to the account.
Capital Gain = FMV of the asset on the date of transfer Indexed cost of acquisition and
improvement.
PGBP = Sale consideration FMV of the asset on the date of transfer.
Indexation shall apply on the basis of the year in which conversion takes place.
Section 45(2) is applicable only if the asset has been converted into stock-in-trade w.e.f. 01.04.1984
onwards. If the conversion is prior to 01.04.1984, no capital gains shall be computed as per Supreme
Court decision in Bai Shirinbai K.Kooka v. CIT (1962) 46 ITR 86 (SC).

[CA INTER M08, 5 Marks]


Question: Aarav converts his plot of land purchased in July, 2009 for 80,000 into stock-in-trade on
31 st March, 2015. The fair market value as on 31.3.2015 was 1,90,000. The stock-in-trade was sold for
2,25,000 in the month of January, 2016.
Find out the taxable income, if any, and if so under which head of income and for which
Assessment Year?
Answer:
Computation of taxable income of Mr. Aarav for A.Y.2016-17
1

Long term capital gains


Full value of consideration (Fair market value on the date of conversion)

Income Tax

1,90,000

10.14

Less

Indexed cost of acquisition *

Profits & Gains of Business or Profession

Less

1,29,620

Sale price of stock-in-trade

2,25,000

Fair market value on the date of conversion

1,90,000

60,380

35,000
95,380

S.45(2A): Computation of Capital Gains [Shares]


Question: Write short note on dematerialization u/s 45(2A) u/h capital gain?
Answer: Where any person had, at any time during the previous year, any beneficial interest in any
security, then any profit and gain arising from transfer made by depositary shall be chargeable to tax as
an income of the beneficial owner of the previous year in which the transfer took place.
Cost of Acquisition (COA) and period of holding (POH) shall be determined on the FIFO basis.

[CMA RTP J11]


Question: The depository account shows the following details of Ms holdings:
Date of Credit
10.11.2004

Particulars
Shares of XYZ Ltd. purchased in physical form on

Quantity
300

10.11.2004 @ 20 per share


30.11.2005

Purchased dematerialised shares of XYZ Ltd.

500 on 25.11.2005
@ 70 per share

06.12.2007

Shares of XYZ LTD. held in physical form, were got dematerialised on 01.12.2007

M sold 600 dematerialised shares on 6th June 2015 @ 200 per share. Brokerage is paid @ 2% of sale price.
Compute capital gains.
Answer:
a) Person Liable: The sale of shares held under Dematerialized format with a depository is chargeable
to tax as the income of the beneficial owner.
b) Cost of Acquisition and period of holdings: The cost of acquisition and the period of holding shall
be determined on FIFO Method. [Circular No. 768 dated 24.6.1998]
i.

FIFO method will be applied for each account independently.

ii. When physical stock is dematerialised, the date of credit into the depository account shall be
considered for the purpose of FIFO method, but indexed cost of acquisition shall be computed on
the basis of year of acquisition.

Capital Gains

10.15

Consideration for Transfer


600 Share @ 200 per share
Less :

1,20,000

Brokerage 2%

2,400

Indexed Cost of Acquisition


(i)

(500 70 1081 / 497)

76,127

(ii)

(100 20 1081 / 551)

3,924

Long Term Capital Gain

80,051
37,549

S.45(3): Computation of Capital Gains [Partner to Firm]


Question: Write short note on transfer of capital asset by a partner to firm u/s 45(3) u/h capital gain?
Answer: Capital gain arising from the transfer of a capital asset by way of contribution to the firm or
otherwise, shall be chargeable to tax in the previous year in which such transfer takes place.
For the purpose of sale consideration, the amount recorded in the books of the firm as a value of capital
asset shall be taken

[CMA RTP D10]


Question: Vijay has three motor cars which are used by him exclusively for his personal purposes. The
cost of the cars was 6,50,000, 8,00,000 and 10,00,000. The first car was transferred by him on 15.1.2016
to firm in which he is a partner as his capital contribution. The market value of the car as on 15.1.2016 is
5,00,000, but it was recorded in the books of account of the firm at 6,00,000. Compute the capital gain if
any, chargeable for the AY 2016-17.
Answer: Since the car is a moveable property and was used by Mr. Ayan for his personal purposes only,
it will be treated as a personal effect.
W.e.f. A.Y. 2008-09, Personal effect means moveable property including wearing apparel and furniture
held for personal use by the assessee or any member of his family dependent on him but excludes:
(i) Jewellery (ii) Archaeological collections (iii) Drawings (iv) Paintings (v) Sculptures (vi) Any work or
art.

[CMA RTP J11]


Question: Nisith acquired a property by way of gift from his father in the year 1991-92 when its FMV was
3 lacs. His father had acquired the property during 1984-85 for 4 lacs. This property was introduced as
capital contribution to a partnership firm in which Nisith became a partner on 15.6.2015. The market
value of the asset as on that date was 40 lacs, but it was recorded in the books of account of the firm at
34 lacs. Is there any capital gain chargeable in the hands of Mr. Nisith?

Income Tax

10.16

Answer:

Computation of Capital Gains

Less:

Consideration for Transfer

34,00,000

Indexed Cost of Acquisition(4,00,000 1081/125)

34,59,200

Long Term Capital Loss

59,200

a) Full value of consideration is taken as the value at which it is recorded in the books of accounts of the
firm.
b) Market value of the asset on the date of transfer is not relevant.

S.45(4): Computation of Capital Gains [Dissolution of Firm]


Question: Write short note on dissolution of firm u/s 45(4) under the head capital gain?
Answer: Capital Gains arising from the transfer of the capital assets by a firm at the time of dissolution of
the firm or otherwise, shall be chargeable to tax as an income of the firm in which such distribution takes
place. For the purpose of the sale consideration, FMV of the capital assets on the date of distribution shall
be taken.
Capital Gain = FMV on the date of transfer - Cost or Indexed COA

[CMA RTP J11]


Question: PQR & Co. is a partnership firm, consisting 3 partners P, Q and R. the firm is dissolved on
31.12.15. The assets of the firm were distributed to the partners as under:
Particulars

Block of machinery

Stock

Land

(given to P)

(given to Q)

(given to R)

Year of acquisition

1995-96

2007-08

1978-79

Cost of acquisition ()

7,20,000

4,00,000

10,000

Market value as on 31.12.15

15,00,000

6,00,000

30,00,000

WDV as on 31.12.15

10,40,000

Value at which given to partners as per agreement

10,00,000

4,50,000

18,00,000

2,70,000

Market value as on 1.4.81

Compute the income taxable in the hands of the firm for the assessment year 2016-17. What shall be the
cost of acquisition of such assets to the partners of the firm?
Answer:
Computation of Short Term Capital Gains on block of Machinery

Less

Sale consideration (i.e. the market value)

15,00,000

Cost of Acquisition (WDV of the block)

10,40,000

Capital Gains

10.17

4,60,000

Short Term Capital Gains


Income from Business (on transfer of stock)

Less

Market value of stock

6,00,000

Cost of Acquisition

4,00,000

Business Income

2,00,000

Computation of Capital Gains on transfer of Land

Less

Consideration for transfer

30,00,000

Indexed cost of Acquisition: ( 2,70,000 1081/100)

29,18,700

Long term capital gains

81,300

Cost of acquisition of assets to the Partners


Partner P

10,00,000

Partner Q

4,50,000

Partner R

18,00,000

S.45(5): Computation of Capital Gains [Compulsorily Acquisition]


Question: Write short note on compulsorily acquisition u/s 45(5) u/h capital gain?
Answer: Where a Capital asset is compulsorily acquired by the Government of similar agency, under any
law then,
1.

Capital gain shall be computed in the year in which the asset was acquired.

2.

Capital gain for initial compensation / original compensation shall be taxable in the year in which
such compensation or part thereof is first received by the assessee.

3.

Any additional compensation, increased compensation received by the assessee is taxable in the year
in which such additional amount is received.

4.

For the purpose of additional compensation COA & COI Shall be zero.

5.

Any expense incurred for the purpose of realization of the additional compensation shall be deducted
from the sale consideration.

6.

Nature of additional compensation should be same as an initial compensation.

7.

Reduction of compensation: Where original compensation / enhanced compensation is reduced by


court, then the Capital gain shall be recomputed accordingly.

Income Tax

10.18

[CMA RTP D11, J12 & D12]


Question: Mr. B acquired a house property for 50,000 in 1969-70. On his death in October 1989 the house
was acquired by his son C. The market value of the house as on 01.04.1981 was 3,00,000. This house was
acquired by the Government on 15.3.2012 and a compensation of 32 lacs is paid to him on 25.3.2016. C
filed a suit against the Government challenging the quantum of compensation and the court ordered for
giving additional compensation of 14,00,000. He incurred an expenditure of 40,000 as expenditure in
connection with the suit. The additional compensation was received on 25.3.2017. Compute capital gains
chargeable to tax.
Answer:

Computation of Long Term Capital Gains for the A.Y. 2016-17

Less

Consideration for transfer (being the compensation)

32,00,000

Indexed Cost of Acquisition(3,00,000785/100)

23,55,000

Long Term Capital Gains / Loss

8,45,000

Computation of Long Term Capital Gains for the A.Y. 2017-18


Enhanced Compensation received
Less

14,00,000

Cost of Acquisition

NIL

Cost of Improvement

NIL

Legal Expenses

(40,000)

Long Term Capital Gains

13,60,000

Question: Assume from the previous problem, that the enhanced compensation is reduced by Supreme
Court from 14,00,000 to 10,00,000 on 7.7.2017 and the legal expenses incurred 20,000.
Answer:

Re-computation of Long Term Capital Gains for the A.Y. 2018-19


Revised enhanced compensation received
Less

14,00,000

Cost of Acquisition

NIL

Cost of Improvement

NIL

Legal Expenses [40,000 + 20,000]


Long Term Capital Gains

60,000
13,40,000

Note: the assessing officer can re-compute the income of the AY 2018-19 within 4 years from the end of
the year in which order of the Supreme Court, reducing the compensation, is passed (i.e. 31.03.2023).

S.45(6): Computation of Capital Gains [Repurchase of Units]


Question: State the provisions u/s 45(6) related to repurchase of units.

Capital Gains

10.19

Answer: Capital gain = Repurchase price less capital value of the units
Note: capital value means the amount invested in the units referred by the assessee u/s 80CCB(2).

S.46: Computation of Capital Gains [Liquidation of Company]


[CA INTER M02, 6 Marks]
Question: State the provisions relating to the computation of capital gains in the hands of shareholder
of a company on distribution of assets upon liquidation. (S.46)
Answer: In the hands of the company (S.46(1)): Where the assets of a company are distributed to its
shareholders on its liquidation, such distribution shall not be regarded as transfer by virtue of Section 47.
In the hands of the shareholders (S.46(2)): Where a shareholder on the liquidation of a company received
any money or other assets from the company, he shall be chargeable to income tax under this head for
such receipt and sale consideration of shares shall be
a.

Distribution in Cash: Amount received less dividend u/s 2(22)(c).

b.

Distribution in Kind: Fair market value (FMV) of assets on the date of liquidation less deemed
divided u/s 2(22)(c).

While computing the period of holding of such assets the period after the date of liquidation shall be
ignored.
Capital Gain on the sale of assets received at the time of liquidation: The cost of acquisition of the asset
received in liquidation, shall be its fair market value on the date of distribution, without deducting
deemed dividend u/s 2(22)(c).

[CMA RTP D11]


Question: Define accumulated profits for a company under liquidation.
Answer: Accumulated profits for a company in liquidation includes all profits of the company upto the
date of liquidation.
Accumulated profits should include the credit balance of profit and loss account, general reserves,
investment allowance, capitalised profits and profits of the year upto the date of distribution/liquidation.
However, provisions and reserves meant for specific liability, to the extent of the liability shall not be
included. Provision for income tax, provision for dividend, reserve for depreciation do not form part of
the accumulated profits.
Securities premium is not accumulated profits.
It may consist of exempted incomes, like agricultural income.
It will include current profits and all profits of the company till the date of liquidation, subject to the
exception provided therein.

Income Tax

10.20

[CA INTER M08, 8 Marks]


Question: Ms. Vasumathi purchased 10,000 equity shares of Rajesh Co. Pvt. Ltd. on 28.2.2010 for
120,000. The company was wound up on 31.07.2015. The following is the summarized financial position
of the company as on 31.07.2015.

Liabilities
60,000 Equity shares
General reserve
Provision for taxation

6,00,000
40,00,000

Assets

Agricultural lands

42,00,000

Cash at Bank

6,50,000

2,50,000
48,50,000

48,50,000

The tax liability (towards dividend distribution tax) was ascertained at 3,00,000 after considering refund
due to the company. The remaining assets were distributed to the shareholders in the proportion of their
shareholding. The market value of 6 acres of agricultural land (in an urban area) as on 31.07.2015 is
10,00,000 per acre.
The agricultural land received above was sold by Ms. Vasumathi on 29.2.2016 for 15,00,000.
Discuss the tax consequences in the hands of the company and Ms. Vasumathi.
Answer: In the hands of the company: As per section 46(1), in case of distribution of capital assets
amongst the shareholders on liquidation of the company is not regarded as transfer in the hands of the
company. Consequently, there will be no capital gains in the hands of the company.
In the hands of Ms. Vasumathi (shareholder)
Section 46(2) provides that such capital gains would be chargeable in the hands of the shareholder.
Ms. Vasumathi holds 1/6th of the shareholding of the company
Market value of agricultural land received (1 acre @ 10 lakhs)
Add

Cash at bank (1/6th of (6,50,000 3,00,000)

Amount ( )
10,00,000
58,333
10,58,333

Less

Less

Less

Deemed Dividend u/s 2(22) (c ) 1/6th of (40,00,000 50,000)

6,58,333

Consideration for computing Capital Gain

4,00,000

Indexed cost of acquisition of Shares

2,05,253

Long term capital gains

1,94,747

Sale consideration of agricultural land

15,00,000

Fair market value of the agricultural land on the date of distribution

10,00,000

Short term capital gain

5,00,000

Note: Dividend u/s 2(22) (c) 6,58,333 will be exempt under section 10(34).

Capital Gains

10.21

Note: Since the question states that there is refund due to the company, it is assumed that the provision
for taxation of 2,50,000 shown in Balance Sheet is in respect of dividend distribution tax. Therefore, the
tax liability in respect of dividend distribution tax ascertained at 3,00,000 has to be reduced from bank
balance while computing full value of consideration under section 46(2), 50,000, being the difference
between 3,00,000 and 2,50,000 has to be reduced from General Reserve for calculating deemed dividend
under section 2(22)(c ).

S.46A: Computation of Capital Gains [Buy-back Shares]


[CMA RTP D10]
Question: What is the tax treatment of consequence for repurchase or buy back of shares or specified
securities by a company?
Answer: As per the provisions in Sec. 46A:
1.

Where a shareholder receives any consideration from the company for purchase of its own shares or
other specified securities, it is a transfer chargeable under the head Capital Gains.

2.

The Capital Gains taxable in the previous year in which the shares or securities are purchased by the
Company.

3.

Capital Gains = Value of Consideration Received Less Cost of Acquisition or Indexed cost of
acquisition.

4.

No deemed dividend: In case of buy back of shares, there is no question of deemed dividend u/s
2(22)(d).

Reference Legal Decisions:


Shares held as

In case Shares treated as Stock-in-Trade are exchanged for Shares of other companies,

Stock-in-Trade

then, Business Profit = Market Value of Shares exchanged Less Book Value of Original
Shares. [Orient Trading Co. Ltd. 224 ITR 371 (SC)]

Reduction of

When there is a reduction in the Face Value of the Shares and consequent payment by

Capital

the Company to the Shareholders towards such reduction, the reduction of Share
Capital is charged to Capital Gains Tax. [Kartikeya vs. Sarabhai 228 ITR 163 (SC)]

[CMA RTP J12 & D12]


Question: X Ltd. having an issued capital of 50,00,000 in equity shares of 100 each. On March 2016,
company decided to buy-back equity shares to the extent of 20%. Tushar, holding 500 shares of the
company, has received the buy-back consideration on the shares bought-back, @ 130 per share. He had
purchased these shares 14 months earlier @ 105 per share. Discuss the taxability.
Answer: Where a shareholder receives any consideration from the company for purchase of its own
shares or other specified securities, it is a transfer chargeable under the head capital gains. Such capital

Income Tax

10.22

gain is chargeable to tax in the previous year in which the shares or securities are purchased by the
company.
Computation of Capital Gains

Less

Consideration for transfer of 100 equity shares [500 shares 20%] @ 130 per share

13,000

Indexed Cost of Acquisition [100 equity shares 105 1081/1024]

11,084

Long term Capital Gains

1,916

S.47: Transactions not regarded as transfer


[CA INTER N03, 6 Marks]
Question: What are the transactions not regarded as transfer as per section 47 under the Income
Tax Act, 1961. (List at least six of such transactions).
Answer: Transactions not regarded as transfer [Section 47]
The following transactions will not be regarded as transfer for the purposes of capital gains tax.
S

Provisions

(i)

Any distribution of capital asset on the total or partial partition of HUF.

(iii)

Any transfer of capital asset under a gift or will or an irrevocable trust. However, it would
not include transfer under gift or an irrevocable trust of a capital asset being shares or
debentures or warrants allotted by a company to the employees under the ESOP scheme in
accordance with the guidelines issued by the Central Government in this behalf.

(iv)

Transfer of capital asset to its wholly owned Indian subsidiary company by a holding
company

(v)

Transfer of capital asset by a wholly owned subsidiary company to its Indian holding
company

(vi)

Transfer of capital asset by the amalgamating company to the Indian amalgamated


company in a scheme of amalgamation

(via)

Transfer of shares held in an Indian company by the amalgamating foreign company in a


scheme of amalgamation to the amalgamated foreign company, if
(a) at least 25% of the shareholders of the amalgamating foreign company continue to
remain shareholders of the amalgamated foreign company, and
(b) such transfer does not attract tax on capital gains in the country in which the
amalgamating company is incorporated.

(viaa)

any transfer, in a scheme of amalgamation of a banking company with a banking institution


sanctioned and brought into force by the Central Government

(viab) 1

any transfer, in a scheme of amalgamation, of a capital asset, being a share of a foreign

Following clause (viab) shall be inserted by the Finance Act, 2015, w.e.f. 1-4-2016

Capital Gains

10.23

company, referred to in the Explanation 5 to clause (i) of sub-section (1) of section 9, which
derives, directly or indirectly, its value substantially from the share or shares of an Indian
company, held by the amalgamating foreign company to the amalgamated foreign
company, if
(A) at least twenty-five per cent of the shareholders of the amalgamating foreign company
continue to remain shareholders of the amalgamated foreign company; and
(B) such transfer does not attract tax on capital gains in the country in which the
amalgamating company is incorporated;
(vib)

Transfer of capital asset by the demerged company to the resulting Indian company in a
scheme of demerger

(vic)

Transfer of capital asset, being a share or shares held in Indian company, by the demerged
foreign company to the resulting foreign company, if
(a) the shareholders holding not less than three-fourths in the value of the shares of the
demerged foreign company continue to remain shareholders of the resulting foreign
company; and
(b) such transfer does not attract tax on capital gains in the country, i n which the demerged
foreign company is incorporated.

(vica)

any transfer in a business reorganisation, of a capital asset by the predecessor co -operative


bank to the successor co-operative bank;

(vicb)

any transfer by a shareholder, in a business reorganisation, of a capital asset being a share


or shares held by him in the predecessor co-operative bank if the transfer is made in
consideration of the allotment to him of any share or shares in the successor co -operative
bank.

(vicc)1

any transfer in a demerger, of a capital asset, being a share of a foreign company, referred to
in the Explanation 5 to clause (i) of sub-section (1) of section 9, which derives, directly or
indirectly, its value substantially from the share or shares of an Indian company, held by the
demerged foreign company to the resulting foreign company, if
(a) the shareholders, holding not less than three-fourths in value of the shares of the
demerged foreign company, continue to remain shareholders of the resulting foreign
company; and
(b) such transfer does not attract tax on capital gains in the country in which the demerged
foreign company is incorporated:

(vid)

any transfer or issue of shares by the resulting company, in a scheme of demerger to the
shareholders of the demerged company if the transfer or issue is made in consideration of
demerger of the undertaking;

Following clause (viab) shall be inserted by the Finance Act, 2015, w.e.f. 1-4-2016

Income Tax

10.24

(vii)

any transfer by a shareholder, in a scheme of amalgamation, of a capital asset being a share


or shares held by him in the amalgamating company, if
(a) the transfer is made in consideration of the allotment to him of any share or shares in the
amalgamated company except where the shareholder itself is the amalgamated company,
and
(b) the amalgamated company is an Indian company;

(viia)

any transfer of a capital asset, being bonds or Global Depository Receipts referred to in subsection (1) of section 115AC, made outside India by a non-resident to another non-resident;

(viib)

any transfer of a capital asset, being a Government Security carrying a periodic payment of interest,
made outside India through an intermediary dealing in settlement of securities, by a non -resident to
another non-resident.

(viii)

any transfer of agricultural land in India effected before the 1st day of March, 1970;

(ix)

any transfer of a capital asset, being any work of art, archaeological, scientific or art
collection, book, manuscript, drawing, painting, photograph or print, to the Government or
a University or the National Museum, National Art Gallery, National Archives or any s uch
other public museum or institution as may be notified by the Central Government in the
Official Gazette to be of national importance or to be of renown throughout any State or
States.

(x)

any transfer by way of conversion of bonds or debentures, debenture-stock or deposit


certificates in any form, of a company into shares or debentures of that company;

(xa)

any transfer by way of conversion of bonds referred to in clause ( a) of sub-section (1)


of section 115AC into shares or debentures of any company;

(xi)

any transfer made on or before the 31st day of December, 1998 by a person (not being a
company) of a capital asset being membership of a recognised stock exchange to a company
in exchange of shares allotted by that company to the transferor.

(xii)

any transfer of a capital asset, being land of a sick industrial company, made under a
scheme prepared and sanctioned under section 18 of the Sick Industrial Companies (Special
Provisions) Act, 1985 (1 of 1986) where such sick industrial company is being managed by
its workers' co-operative

(xiii)

any transfer of a capital asset or intangible asset by a firm to a company as a result of


succession of the firm by a company in the business carried on by the firm, or any transfer
of a capital asset to a company in the course of demutualisation or corporatisation of a
recognised stock exchange in India as a result of which an association of persons or body of
individuals is succeeded by such company :
Provided that
(a) all the assets and liabilities of the firm or of the association of persons or body of
individuals relating to the business immediately before the succession become the assets

Capital Gains

10.25

and liabilities of the company;


(b) all the partners of the firm immediately before the succession become the shareholders of
the company in the same proportion in which their capital accounts stood in the books of
the firm on the date of the succession;
(c) the partners of the firm do not receive any consideration or benefit, direc tly or indirectly,
in any form or manner, other than by way of allotment of shares in the company; and
(d) the aggregate of the shareholding in the company of the partners of the firm is not less
than fifty per cent of the total voting power in the company and their shareholding
continues to be as such for a period of five years from the date of the succession;
(e) the demutualisation or corporatisation of a recognised stock exchange in India is carried
out in accordance with a scheme for demutualisation or corporatisation which is approved
by the Securities and Exchange Board of India established under section 3 of the Securities
and Exchange Board of India Act, 1992 (15 of 1992);
(xiiia)

any transfer of a capital asset being a membership right held by a m ember of a recognised
stock exchange in India for acquisition of shares and trading or clearing rights acquired by
such member in that recognised stock exchange in accordance with a scheme for
demutualisation or corporatisation which is approved by the Securities and Exchange Board
of India established under section 3 of the Securities and Exchange Board of India Act, 1992
(15 of 1992);

(xiiib)

any transfer of a capital asset or intangible asset by a private company or unlisted public
company to a limited liability partnership or any transfer of a share or shares held in the
company by a shareholder as a result of conversion of the company into a limited liability
partnership in accordance with the provisions of section 56 or section 57 of the Limited
Liability Partnership Act, 2008 (6 of 2009):
Provided that
(a) all the assets and liabilities of the company immediately before the conversion become
the assets and liabilities of the limited liability partnership;
(b) all the shareholders of the company immediately before the conversion become the
partners of the limited liability partnership and their capital contribution and profit sharing
ratio in the limited liability partnership are in the same proportion as their shareholding in
the company on the date of conversion;
(c) the shareholders of the company do not receive any consideration or benefit, directly or
indirectly, in any form or manner, other than by way of share in profit and capital
contribution in the limited liability partnership;
(d) the aggregate of the profit sharing ratio of the shareholders of the company in the
limited liability partnership shall not be less than fifty per cent at any time during the
period of five years from the date of conversion;

Income Tax

10.26

(e) the total sales, turnover or gross receipts in the business of the company in any of the
three previous years preceding the previous year in which the conversion takes place does
not exceed sixty lakh rupees; and
(f) no amount is paid, either directly or indirectly, to any partner out of balance of
accumulated profit standing in the accounts of the company on the date of conversion for a
period of three years from the date of conversion.
(xiv)

[CMA INTER SY12, J15, 4 Marks]


Question: State the conditions to be satisfied when a sole proprietary concern is succeeded by a
company, to avail tax exemption in respect of capital gains.
Answer: where a sole proprietary concern is succeeded by a company in the business carried on
by it as a result of which the sole proprietary concern sells or otherwise transfers any capital
asset or intangible asset to the company:
Provided that
(a) all the assets and liabilities of the sole proprietary concern relating to the business
immediately before the succession become the assets and liabilities of the company;
(b) the shareholding of the sole proprietor in the company is not less than fifty per cent of
the total voting power in the company and his shareholding continues to remain as such for
a period of five years from the date of the succession; and
(c) the sole proprietor does not receive any consideration or benefit, directly or indirectly, in
any form or manner, other than by way of allotment of shares in the company;

(xv)

any transfer in a scheme for lending of any securities under an agreement or arrangement,
which the assessee has entered into with the borrower of such securities and which is
subject to the guidelines issued by the SEBI, in this regard;

(xvi)

any transfer of a capital asset in a transaction of reverse mortgage under a scheme made and
notified by the Central Government

(xvii)

any transfer of a capital asset, being share of a special purpose vehicle to a business trust in
exchange of units allotted by that trust to the transferor.
Explanation.For the purposes of this clause, the expression "special purpose vehicle" shall
have the meaning assigned to it in the Explanation to clause (23FC) of section 10.]

(xviii)1

any transfer by a unit holder of a capital asset, being a unit or units, held by him in the
consolidating scheme of a mutual fund, made in consideration of the allotment to him of a
capital asset, being a unit or units, in the consolidated scheme of the mutual fund:
Provided that the consolidation is of two or more schemes of equity oriented fund or of two
or more schemes of a fund other than equity oriented fund.

Following clause (viab) shall be inserted by the Finance Act, 2015, w.e.f. 1-4-2016

Capital Gains

10.27

Explanation. For the purposes of this clause,


(a) "consolidated scheme" means the scheme with which the consolidating scheme merges
or which is formed as a result of such merger;
(b) "consolidating scheme" means the scheme of a mutual fund which merges under the
process of consolidation of the schemes of mutual fund in accordance with the Securities
and Exchange Board of India (Mutual Funds) Regulations, 1996 made under the Securities
and Exchange Board of India Act, 1992 (15 of 1992);
(c) "equity oriented fund" shall have the meaning assigned to it in clause ( 38) of section 10;
(d) "mutual fund" means a mutual fund specified under clause (23D) of section 10.

[CMA RTP D11]


Question: What is a Foreign Exchange Asset?
Answer: Foreign Exchange Asset- means those specified asset which the assessee has acquired or
purchased with, or subscribed to in, convertible foreign exchange.
The following are the specified assets:
(i) shares in an Indian Company (public or private)
(ii) debentures issued by an Indian Company which is not a private company ;
(iii) deposits with an Indian Company which is not a private Company, it may be even deposit with SBI
or any other banking company;
(iv) any security of the Central Government ; and
(v) such other asset as the Central Government may specify in this behalf by notification in the Official
Gazette.

[CMA RTP J12 & D12]


Question: Mohini, a non-resident Indian, purchased 5,000 shares in Happy Days Ltd, an Indian
Company, in foreign currency for $50,000 in August 2013. She sold these shares to Mrigakshi, another
non-resident in Singapore, in March 2016 for $67,000. Discuss the impact of the given transactions, if any,
in the assessment of Mohini.
Answer: As per S.47(viia), where a Non-resident Indian acquires bonds in foreign currency, and the same
is transferred outside India to another non-resident in foreign currency, then the transaction is not a
transfer and hence not chargeable to capital gains.
However, if the asset was transferred in India, then provisions of S.115AC shall apply and the Capital
Gain shall be chargeable to tax @ 10%, as it is a Long-Term Capital Asset.

Income Tax

10.28

S.47A: Withdrawal of Exemptions


Question: Explain the withdrawal of exemption u/s 47A
Answer: Withdrawal of exemption in certain cases.
S
1

Not transfer &

Before

Withdrawal and taxable u/s 45,

not taxed u/s 45

Period

in the following situation

S.47(iv)

Transfer

by

subsidiary

/ (v)

company

to

wholly

or

holding

owned

holding

8 years

(a) Converting such capital asset


in to stock-in-trade

company or subsidiary company

(b) ceases to be wholly owned


subsidiary company

S.47(xi)

Transfer of membership of a recognised

3 years

stock exchange for shares

Transfer of shares allotted in


exchange of membership in a
recognised stock exchange

S.47(xiii)

Transfer of capital asset by a firm / sole

/ (xiv)

trade to a company as a result of

The requirements of S.47(xiii) /


(xiv) are not complied

succession of the firm / sole trade


4

S.47(xiiib)

Transfer of capital asset by a private

company or unlisted public company to a

The requirements of S.47(xiiib)


are not complied

LLP a result of succession of the company

S.49: Deemed Cost of Acquisition


Question: Write short note on deemed cost of acquisition u/h capital gain?
Answer: When the capital asset becomes the property of the assessee in any of the manner mentioned
below, the cost of the acquisition of the asset shall be deemed to be the cost for which the previous owner
of the property acquired it. Following are the examples of the cases:
a.

On the distribution of the asset on the total / partial partition of HUF

b.

under a gift or will

c.

By succession, inheritance or devolution.

d. On any distribution of assets on the liquidation of the company.


e.

Under a transfer to a revocable or irrevocable trust;

f.

On the transfer by a wholly owned India subsidiary company to its holding company or vice versa

g.

On any transfer in a scheme of amalgamation of two Indian companies subject to certain conditions
u/s 47.

Capital Gains

10.29

w.r.t S

49

Case

Cost of Acquisition Deemed

Cost with reference to certain


modes of acquisition

49(1)

Where the capital asset became the


property of the assessee

49(1)(i)

S.47(i)

On any distribution of assets on

CA = CAP

the total or partial partition of a


HUF
49(1)(ii)

S.47(iii)

49(1)(iii)(a)

under a gift or will

CA = CAP

by succession, inheritance or

CA = CAP

devolution, or
49(1)(iii)(b)

on any distribution of assets on the

CA = CAP

dissolution of a firm, BOI, or AOP


dissolution had taken place at any
time before the 1st day of April,
1987, or
49(1)(iii)(c)

on any distribution of assets on the

CA = CA of the previous owner

liquidation of a company, or
49(1)(iii)(d)

under a transfer to a revocable or

CA = CA of the previous owner

an irrevocable trust, or
49(1)(iii)(e)

under any such transfer as is

CA = CA of the previous owner

referred to in S.47(iv) / (v) / (vi) /


(via) / (viaa) / (vica) / (vicb) / (xiii) /
(xiiib) / (xiv)
49(1)(iv)

Such assessee being a HUF, by the

CA = CA of the previous owner

mode referred to in S.64(2) after


31.12.1969
49(2)

47(vii)

transfer by amalgamating

CA = CA of the amalgamating company

company
49(2A)

49(2AA)

47(x)

conversion of bonds / debentures

CA = CA of such original bond or

/(xa)

into share or debenture

debenture

17(2)(vi)

sweat equity shares or specified

CA = FMV

securities w.e.f AY2010-11


49(2AAA)

47(xiiib)

conversion of company into LLP

CA of asset = CA of the shares

49(2AB)

115WC

sweat equity shares or specified

CA = FMV

(1)(ba)

securities

Income Tax

10.30

49(2ABB)1

115AC

receiving shares for Global

CA shares = FMV of shares on redemption

(1)(b)

Depository Receipt (GDR) by NRI

of GDR

49(2AC)

47(xvii)

shares received for units of trust

CA shares = CA of units

49(2AD)2

47(xviii)

units received for units

CA units = CA of original units

49(2C)

47(vid)

Shares received in resulting


company in demerger

49(2D)

49(2E)

Shares held in demerged company

CA in the demerged company is reduced

after S.49(2C)

by CA u/s 49(2C)

S.49(2)/(2C)/(2D apply to business reorganisation of a co-operative bank as referred to


in section 44DB.

49(3)

47(iv) /

CG by S.47A

CA to the transferee company = CA of such asset

(v)
49(4)

was acquired

56(2)(vii)

Cost of Acquisition of

CA = Taxable value considered for valuing the

/(viia)

property taxable

gifts

[CMA INTER SY08, D12, 6 Marks]


Question: Excel Ltd. allotted 1,000 (sweat) equity shares of 10 each to Mr. Rao, General Manager. The
fair market value of the shares computed in accordance with the method prescribed under the Income-tax
Act / Rules was 500 per share, whereas it was allotted at 300 per share. What is the perquisite value of
sweat equity shares allotted to Mr. Rao? In case these shares are sold subsequently, what would be their
cost of acquisition in the hands of Mr.Rao?
Answer: As per S.17(2)(vi) the value of sweat equity shares chargeable as perquisite shall be the fair
market value of such shares on the date on which the option is exercised by the assesses as reduced by
the amount actually paid or recovered from him in respect of such shares.

Particulars

Less:

Amount( )

Fair market value of 1,000 equity shares @ 500 each

5,00,000

Amount recovered from Mr. Rao 1,000 x 300 each

3,00,000

Value of perquisite of sweat equity shares allotted to Mr. Rao

2,00,000

As per S.49(2AA) the cost of acquisition of such shares shall be adopted while computing the capital gain.
Hence, the cost of acquisition for Mr. Rao shall be 3,00,000.

1
2

Applicable w.e.f. 1.4.2016


Applicable w.e.f. 1.4.2016

Capital Gains

10.31

S.50: Computation of Capital Gains [Depreciable Asset]


[CA INTER M08, 5 Marks]
Question: Explain the computation of capital gain in case of depreciable asset u/s 50.
Answer: Computation of capital gains in case of a depreciable asset:
Section 50 provides for the computation of capital gains in case of depreciable assets. Where the f ull
value of consideration received or accruing as a result of the transfer of the asset plus the full value
of such consideration for the transfer of any other capital asset falling within the block of assets
during the previous year, exceeds the aggregate of the following amounts, namely:
1.

expenditure incurred wholly and exclusively in connection with such transfer;

2.

WDV of the block of assets at the beginning of the previous year;

3.

the actual cost of any assets falling within the block of assets acquired during the previous year,
such excess shall be deemed to be the capital gains arising from the transfer of short -term capital
assets.

Where all assets in a block are transferred during the previous year, the block itself will cease to
exist. In such a situation, the difference between the sale value of the assets and the WDV of the
block of assets at the beginning of the previous year together with the actual cost of any asset falling
within that block of assets acquired by the assessee during the previous year will be deemed to be
the capital gains arising from the transfer of short term capital assets.

[CA INTER M99 & N08, 7 & 10 Marks][Similar in M99 Kishore Industries]
Question: Singhania & Co. own six machines, put in use for business in March, 2012. The
depreciation on these machines is charged @ 15%. The written down value of these machines at the
end of the previous year relevant to assessment year 2016-17 was 8,50,000. A new plant was bought
for 8,50,000 on 30 th November, 2015.
Three of the old machines were sold on 10 th June, 2015 for 11,00,000.
You are required to:
1.

determine the claim of depreciation for Assessment Year 2016-17.

2.

compute the capital gains liable to tax for Assessment Year 2016-17.

3.

If Singhania & Co. had sold the three machines in June, 2015 for 21,00,000, will there be any
difference in your above workings? Explain.

Answer:
Computation of depreciation claim for A.Y.2016-17
Particulars
W.D.V. of the block as on 1.4.2015
Add

Purchase of new plant during the year

8,50,000
_8,50,000
17,00,000

Income Tax

10.32

Less

Sale consideration of old machinery during the year

11,00,000

W.D.V of the block as on 31.03.16

_6,00,000

Since the value of the block as on 31.3.16 comprises of a new asset which has been put to use for less
than 180 days, depreciation is restricted to 50% of the prescribed percentage of 15% i.e. depreciation
is restricted to 7%. Therefore, the depreciation allowable for the year is 45,000, being 7% of
6,00,000.
(2) The provisions u/s 50 for computation of capital gains in the case of depreciable assets can be
invoked only under the following circumstances:
a.

When one or some of the assets in the block are re-sold for consideration more than the value of
the block.

b. When all the assets are transferred for a consideration more than the value of the block.
c.

When all the assets are transferred for a consideration less than the value of the block.

Since in the first two cases, the sale consideration is more than the written down value of the block,
the computation would result in short term capital gains.
In the third case, since the written down value exceeds the sale consideration, the resultant figure
would be a short term capital loss.
In the given case, capital gains will not arise as the block of asset continues to exist, and some of the
assets are sold for a price which is lesser than the written down value of the block.
(3) If the three machines are sold in June, 2015 for 21,00,000, then short term capital gains would
arise, since the sale consideration is more than the aggregate of the written down value of the block
at the beginning of the year and the additions made during the year.
Particulars

Sale consideration
Less

21,00,000

W.D.V. of the machines as on 1.4.2015

8,50,000

Purchase of new plant during the year

_8,50,000

Short term capital gains

50A Transfer of depreciable assets by Power Sector


Units [S.32(1)(i) depreciated under SLM)

17,00,000
_4,00,000

CG = Consideration less
original cost

S.50B: Computation of Capital Gains [Slump Sale]


[CA INTER M03, M06 & N08, 5, 6 & 6 Marks][CMA INTER SY08, J14, 5 Marks]
Question: What is Slump sale? How is capital gain computed in case of a slump sale?

Capital Gains

10.33

Answer: Meaning of slump sale: Sale of undertaking for a lump sum consideration without determining
the value of the individual assets and liabilities [S.2(42C)].
If any person has transferred the entire unit or part of the unit for a lump-sum consideration, capital
gain shall be computed for the entire unit instead of individual asset. Any profit or gain from the
slump sale shall be chargeable as Long Term Capital Gain in the year of transfer, if undertaking is
owned by the assessee for more than 36 months immediately preceding the date of transfer.
Otherwise it shall be short term capital gain. Benefit of indexation is not allowed in the case of slump
sale.
For computing capital gain, cost of acquisition and cost of improvement shall be Net Worth.
Net Worth shall be the total value of assets of the undertaking as reduced by the liabilities appearing
in the books of account. Revaluation shall be ignored.
A report from of Chartered Accountant is required to be furnished certifying that net worth has been
computed correctly.
While computing the net worth in case of slump sale for the purpose of computing capital gain, in the
case of capital assets in respect of which the whole of the expenditure has been allowed or is
allowable as a deduction under 35AD, its cost shall be taken as Nil as the entire cost has been allowed
as a deduction (w.e.f. 01.04.2010)

[CMA INTER SY08, D13, 1 Mark]


Question: In case of slum sale of any undertaking indexation benefit is _______ (allowed / not allowed)
for the purpose of computation of capital gain1.

[CMA INTER D11, 8 Marks]


Question: VERTIKA Limited has two units - one engaged in manufacture of computer hardware and the
other involved in developing software. As a restructuring drive, the company has decided to sell its
software unit as a going concern by way of slump sale for 385 lakh to a new company called SUMEDHA
Limited, in which it holds 74% equity shares.
The balance sheet of VERTIKA Limited as on 31 st March, 2016 being the date on which software unit has
been transferred, is given hereunder
Balance Sheet as on 31.3.2016
Liabilities

in lakh

Assets

in lakh

Paid up Share Capital

300

Fixed Assets

General Reserve

150

Hardware unit

170

Share Premium

50

Software unit

200

Revaluation Reserve
Current Liabilities

in lakh

120

in lakh

370

Debtors
Hardware unit

140

Answer: not allowed

Income Tax

10.34

Hardware unit

40

Software unit

90

Software unit
130

110

250

Inventories
Hardware unit

95

Software unit

35

130

750

750

Following additional information are furnished by the management.


(i) The Software unit is in existence since May, 2012.
(ii) Fixed assets of software unit include land which was purchased at 40 lakh in the year 2009 and
revalued at 60 lakh as on March 31, 2016.
(iii) Fixed assets of software unit mirrored at 140 lakh (200 lakh minus land value 60 lakh) is written
down value of depreciable assets as per books of account. However, the written down value of these
assets u/s 43(6) of the Income Tax Act is 90 lakh.
Ascertain the tax liability, which would arise from slump sale to SUMEDHA Limited.
Answer: As per section 50B, any profits and gains arising from the slump sale effected in the previous
year shall be chargeable to income-tax as capital gains arising from the transfer of capital assets and shall
be deemed to be the income of the previous year in which the transfer took place.
If the assessee owned and held the undertaking transferred under slump sale for more than 36 months
before slump sale, the capital gain shall be deemed to be long-term capital gain. Indexation benefit is not
available in case of slump sale as per section 50B(2).
Ascertainment of tax liability from slump sale of software unit
Particulars

Less

( in lakh)

Sale consideration for slump sale of Software Unit

385

Cost of acquisition being the net worth of Software Unit

185

Long term capital gains arising on slump sale

200

(The capital gains is long-term as the Software Unit is heldfor more than 36 months)
Tax liability on LTCG u/s 112 @ 20% on 200 lakhs
Add

40.00

Surcharge 7%

3.00
43.00

Add

Education cess @ 2% and SHEC @ 1% i.e. totaling 3%

1.29
44.29

Working Note:
Computation of net worth of Software Unit
(1)

Book value of non-depreciable assets

(i)

Land (Revaluation not to be considered)

Capital Gains

( in lakh)

40

10.35

(ii)

Debtors

110

(iii)

Inventories

35

(2)

Written down value of depreciable assets u/s 43(6) (Note 1)

90

Aggregate value of total assets


Less

Current liabilities of software unit


Net worth of software unit

275
90
185

Note 1: For computing net worth, the aggregate value of total assets in the case of depreciable assets shall
be the written down value of the block of assets as per section 43(6).

S.50C: SALE AT LESS THAN STAMP DUTY VALUE


[CA INTER N04, 6 Marks][CMA RTP D10]
Question: Write short notes on capital gains on sale of property at less than Government value.
Answer: [S.50C] The following are the details analysis on chargeability of Capital Gains if a property is
sold at a price which is lower than Govt. Value.
1.

Nature of Asset: The assessee transfers Land, or Building, or both.

2.

Applicability: Sale Consideration is less than the value adopted or assessed or assessable by the
State Government Authority (referred to as Stamp Valuation Authority) for the purpose of
payment of Stamp Duty.
Note: The word Assessable means the price which the Stamp Valuation Authority would have
adopted or assessed, if it were referred to such authority for the purpose of the payments of Stamp
Duty (W.e.f. 01.10.2009).

3.

Consideration adopted for Capital Gains: Value adopted by the Stamp Valuation Authority.

4.

Under the following conditions, a reference is made to the valuation officer :


a.

The assessee can claim that the value adopted or assessed by the Stamp Valuation Authority
exceeds the Fair Market Value of the property as on the date of transfer.

b.

Value adopted by the Stamp Valuation Authority is not disputed before any authority or Court.

c.

Where the value determined by the Valuation Officer exceeds the value adopted by the Stamp
Valuation Authority, the Capital Gain shall be considered as follows

Capital Gains = Value adopted by Stamp Valuation Authority Less Cost or Indexed Cost of Acquisition.

[CMA INTER SY12, D14, 4 Marks]


Question: From the following information, compute the income taxable under the head Capital gains and
Income from other sources in the hands of Sachin:
1.

Sehwag gifted a vacant site to his friend Sachin on 23.05.2015 on the occasion of latters birthday.

2.

Sehwag had acquired the said vacant site in May, 2011 for 30,00,000.

Income Tax

10.36

3.

The fair market value of the site for stamp duty purposes on the date of gift i.e., on 23.05.2015 was
60,00,000.

4.

Sachin sold the vacant site on 15.03.2016 for a consideration of 70 lakhs when its stamp duty value
on the date of sale was 90 lakhs.

For capital gains, state with reason, whether it is short-term or long-term. Also compute the capital gains
chargeable to tax in the hands of Sehwag.
Answer: Computation of total income in the hands of Sachin A.Y. 2016-17

Particulars
Capital gains:

Less

Sale consideration Stamp duty valuation, as per section 50C

90,00,000

Cost of acquisition

60,00,000

Short term capital gain

30,00,000

Income from other sources:


Stamp duty value of the property on the date of receipt of gift i.e., 3.05.15

60,00,000

Total income

90,00,000

Note: Where a property is received without consideration, the FMV i.e., stamp duty valuation is the value
taxable under the head income from other sources in the hands of Sachin. The holding period in the
hands of Sachin shall be from the date of receipt of property and not from the date of acquisition by the
previous owner. Since it was received on 23.05.2015 and sold on 15.03.2016 it is a short-term capital asset
and the resultant gain is short-term capital gain. The donor Sehwag is not subject to capital gains as the
property was not transferred but was gifted which is not regarded as transfer u/s 47.

50D If consideration received or accruing as a result of

SC = FMV on the date

transfer of a capital asset is not ascertainable or

of transfer [from AY

cannot be determined

2013-14]

S.51: Forfeiture of Advance Money


Question: Write Short note on advance money forfeited u/s 51 under the head Capital Gain:
Answer: If any advance money is forfeited by the assessee, then the amount so forfeited shall be
deducted from the cost for which the asset was acquired or the fair market value or written down value,
as the case may be for computing cost of acquisition.
This provision is applicable only when the transfer as per the original agreement does not take place and
the advance money is received and forfeited by the assessee as per the agreement
If advance money forfeited is more than the cost of acquisition, then such excess shall not be taxable.

Capital Gains

10.37

Key Note: Only the amount forfeited by the assessee is deducted, the amount forfeited by the previous
owner shall not be considered.
Note: If any advance money is forfeited on after the previous year 2014-15 is included in the total income
u/s 56(2)(ix)

[CA INTER M11, 8 Marks][CMA INTER D11, 7 Marks (Aggarwal instead of Rakesh)]
Question: Mr. Rakesh purchased a house property on 14th April, 1979 for 1,05,000. He entered into an
agreement with Mr. B for the sale of house on 15th September, 1983 and received an advance of 25,000.
However, since Mr. B did not remit the balance amount, Mr. Rakesh forfeited the advance.
Later on, he gifted the house property to his friend Mr. A on 15th June, 1987.
Following renovations were carried out by Mr. Rakesh and Mr. A to the house property:
Amount ( )
By Mr. Aggarwal during FY 1979-80

10,000

By Mr. Aggarwal during FY 1984-85

50,000

By Mr. A during FY 1994-95

1,90,000

The fair market value of the property as on 1.4.1981 is 1,50,000.


Mr. A entered into an agreement with Mr. C for sale of the house on 1 st June, 2000 and received an
advance of 80,000. The said amount was forfeited by Mr. A since Mr. C could not fulfill the terms of the
agreement.
Finally, the house was sold by Mr. A to Mr. Sanjay on 2 nd January, 2016 for a consideration of 24,00,000.
Compute the capital gains chargeable to tax in the hands of Mr. A for the assessment year 2016-17. Cost
inflation indices are as under:
Answer: Computation of capital gains chargeable to tax in the hands of Mr. A
Particulars

Amount ( )

Sale consideration

24,00,000

Less

Indexed cost of acquisition (

Less

Indexed cost of improvement in 1984-85 (

Less

Indexed cost of improvement in 1994-95 (

7,56,700
)

4,32,400
)

Long term capital gain

7,93,012
4,17,888

Question: Assume from the previous problem, Mr. A entered into an agreement with Mr. C for sale of the
house on 1st June, 2015 (instead of 1st June, 2000) and received an advance of 80,000. The said amount
was forfeited by Mr. A since Mr. C could not fulfill the terms of the agreement.

Income Tax

10.38

Answer: Computation of capital gains chargeable to tax in the hands of Mr. A


Particulars

Amount ( )

Sale consideration

24,00,000

Less

Indexed cost of acquisition (

Less

Indexed cost of improvement in 1984-85 (

Less

Indexed cost of improvement in 1994-95 (

16,21,500
)

4,32,400
)

Long term capital gain (loss)

7,93,012
(4,46,912)

Income from other sources u/s. 56(2)(ix)

80,000

S.55: Cost of Acquisition, Improvement


Question: Write short note on cost of acquisition u/s 55 u/h capital gain?
Answer:
S

Meaning of "adjusted", "cost of improvement" and "cost of acquisition"

55(1)

"cost of any improvement" for the purposes of S.48 and S.49

55(1)(b)

(1) goodwill of a business or a right to manufacture, produce or process any article or

Nil

thing or right to carry on any business


(2) other capital asset after 01.04.81 [before 31.03.81 is nil]

Actual

55(2)

Cost of acquisition for the purposes of S.48 and S.49

55(2)(a)

goodwill of a business or a trade mark or brand name associated with a business or a


right to manufacture, produce or process any article or thing or right to carry on any
business, tenancy rights, stage carriage permits or loom hours
(i) in case of purchase

Actual

(ii) self-generated

Nil

Question: Write short note on cost of acquisition of different types of Shares (S.55) u/h capital gains?
Types of Shares

Actual Cost of acquisition

Period of holding

55(2)(aa)

Share or any other securities

(i)

Shares Originally Purchased

Actual payment

a.

Primary market

Allotment Price

From allotment

b.

Secondary market
Amount paid + Brokerage Charges +

Date

Adjustment for ex. & com. Dividend /

note

Transaction
broker

through

share

Capital Gains

of

brokers

10.39

interest
Transaction between parties

As above (excluding Brokerage)

Date of contract of
sale

(ii)

Renouncing the shares

Nil

(iii)

Right Shares purchased by the

Price paid

From Allotment

Nil (but acquired before 01.04-1981,

From Allotment

original shareholder
(iiia)

Bonus shares

FMV as on 01-04-1981 if available)


(iv)

55(2)(ab)

Person purchasing such offer

Price paid to the seller + price paid to

from original shareholder

the company

Shares under corporatisation

CA of original membership of the

From Allotment

exchange
55(2)(b)

Cost of Acquisition of assets is the value for which it was acquired by the assessee.
Expenses of capital nature for completing or acquiring the capital assets are includible in the
cost of Acquisition. It will be determined as below:

(i)

If the asset is acquired before 01.04.1981, cost of acquisition shall be the expenditure incurred
by the assessee for acquiring the asset or its fair market value as on 01.04.1981, whichever is
higher.

(ii)

If the asset has been acquired with effect from 01.04.1981 onwards, cost of acquisition shall be
the expenditure incurred by the assessee for acquiring the asset.

(iii)

Where the capital asset became the property of the assessee on the distribution of the capital
assets of a company on its liquidation and the assessee has been assessed to the income-tax
under the head "capital gains" in respect of that asset u/s 46, means the fair market value of the
asset on the date of distribution.

(v)

Where the capital asset, being a share or a stock of a company, become the property of the
assessee on
(a) the consolidation
(b) conversion of shares in to stocks
(c) re-conversion of any stock of the company into shares
(d) sub-division
(e) conversion of one kind of shares into another kind of shares
In all the above cases, CA = CA of the original stock or shares

55(3)

If CA of previous owner is not ascertained, CA = FMA on the date of acquisition

[CA INTER N06, 6 Marks]


Question: Write short notes on cost of acquisition of self-generated assets, in the context of capital

Income Tax

10.40

gains, with reference to the provisions of the Income-tax Act, 1961:


Answer: Cost of acquisition of self-generated assets, in the context of capital gains
1.

Cost of acquisition of a capital asset, being goodwill of a business or a trade mark or brand name
associated with a business or a right to manufacture, produce or process any article or thing, or
right to carry on any business, tenancy rights, stage carriage permits and loom hours [Section
55(2)(a)]
(i) If the above capital assets have been purchased by the assessee, the cost of acquisition is the
amount of the purchase price. For example, if Mr. A purchases a stage carr iage permit from
Mr. B for 2 lacs, that will be the cost of acquisition for Mr. A.
(ii) If the above capital assets are self-generated, the cost of acquisition shall be taken as nil.
(iii) In case the capital asset is acquired by any mode given under clauses (i) to (iv) of section
49(1), the cost of acquisition will be the cost to the previous owner if the previous owner paid
for it. However, if it was self-generated by the previous owner, the cost of acquisition will be
taken as nil.

2.

Cost of acquisition of other self-generated assets not covered u/s 55(2)(a) In respect of selfgenerated goodwill of a profession and other self-generated assets not specifically covered u/s
55(2)(a), the decision of the Supreme Court in CIT v. B.C. Srinivasa Setty [1981] 128 ITR 294 will
apply. In that case, the Supreme Court held that if the cost of acquisition of a self -generated asset
is incapable of determination, then transfer of such asset is not taxable and consequently the
gains thereon cannot be brought to charge.

[CMA INTER SY08,D12, 1 Mark]


Question: When a person retires from a profession and receives any amount towards self-generated
goodwill, it is1
a) Taxable as income from profession
b) Exempt income u/s 10
c)

Not taxable since there is no cost of acquisition

d) Taxable as capital gain

[CMA RTP J12 & D12]


Question: (a) P commenced a business on 10.5.95. The said business is sold by P on 25.8.15 and he
received 12 lacs towards goodwill.
(b) What will be your answer in the above case, if P had acquired the goodwill for this business for a
consideration of 3,00,000.

Answer: (d) taxable as capital gain

Capital Gains

10.41

Answer:

Computation of Long Term Capital Gains for the A.Y. 2016-17


(a)

Consideration for transfer

12,00,000

Less

Indexed Cost of Acquisition (Self-Generated)

NIL

Long term Capital Gains

12,00,000

(b)

Consideration for transfer

12,00,000

Less

Indexed Cost of Acquisition [3,00,000 1081/281]

11,54,093

Long term Capital Gains

45,907

[CMA RTP J12 & D12]


Question: R has been living in a rented accommodation since August 1986, and he is paying a rent of
4,000 per month. The landlord got the house vacated from R on 16.7.2015 and paid a sum of 15 lacs for
vacating the house. Compute Capital Gains, if any, in the hands of R.
Answer:

Computation of Long Term Capital Gains for the A.Y. 2016-17


Consideration for transfer
Less

15,00,000

Indexed Cost of Acquisition (Self-Generated asset)


Long term Capital Gains

NIL
15,00,000

[CA INTER M00, 7 Marks]


Question: A is a shareholder of X & Co Ltd., holding 1,000 shares of the face value of 10 each, allotted at
the time of the companys incorporation in May, 1997. The company made a right issue in the ratio of 1:1
on 15.7.2015 at a premium of 40 per share. Instead of taking up the right, he renounced it in favour of B
at a price of 10 per share. What is the capital gain chargeable in the hands of A. What will be the cost of
the shares in the hands of B?
Answer:
Capital gain in the hand of A

Section

Sale consideration (10 x 1,000)


Less

Cost of acquisition of right entitlement


Short term Capital gain

10,000

S.55(2)(aa)(ii)

Nil
10,000

The cost of shares in the hands of B will be the aggregate of


Cost of acquisition of right shares
Price paid to A for acquiring the right

Income Tax

Section

10,000

10.42

Add

55

Amount paid to X & Co. Ltd. towards 1000 shares (1000 x 50)

50,000
60,000

[CA IPCC M13, 8 Marks]


Question: Ms. Neelima had purchased 500 equity shares in A Ltd. at a cost of 30 per share (brokerage
1%) in February 1979.
She got 50 bonus shares in September 1980.
She again got 550 bonus shares by virtue of her holding on March, 1985.
Fair market value of the shares of A Ltd. on April, 1981 is 50.
In January 2016, she transferred all her shares @ 240 per share (brokerage 2%)
Compute the capital gains taxable in the hands of Ms. Neelima for the Assessment Year 2016-17
assuming.
(A) A Ltd. is an unlisted company and securities transaction tax was not applicable at the time of sale.
(B) A Ltd. is a listed company and the shares are sold in a recognised stock exchange and securities
transaction tax was paid at the time of sale.
In case A Ltd. is an unlisted company and STT was not applicable at the time of sale
Answer: Computation of capital gains of Ms. Neelima for the A.Y. 2016-17

(A)
500 equity shares

Cost of acquisition
Sale proceeds (500 x 240)
Less

Bonus Share

Bonus Share

Feb 1979

Sep 1980

Mar 85

WEH (Cost

WEH (Cost

Nil

or FMV 1981)

or FMV 1981)

1,20,000

12,000

1,32,000

2,400

240

2,640

1,17,600

11,760

1,29,360

2,70,2501

27,0252

NIL

(1,52,650)

(15,265)

1,29,360

Brokerage paid (2% of 120000)


Net sale consideration

Less

Original Share

Indexed cost of acquisition


Long term capital gain (loss)
Total Long term capital gain (loss)

22,880

(B) In case the company is listed & shares are sold in recognised stock exchange where STT is paid,
capital gain is exempt from tax. So capital loss computed shall be ignored.

+
+

Capital Gains

10.43

S.55A: Reference to Valuation Officer


[CA INTER N98, N01, M98 & N09, 5, 5, 6 & 3 Marks][CMA RTP D11]
Question: Write short note on reference to valuation officer u/s 55A u/h capital gain?
Answer: For the purpose of ascertaining the fair market value of a capital asset, the assessing officer may
refer the valuation of capital asset to valuation of capital asset to valuation officer in the following cases:
1.

If the report of the registered valuer is attached - If the value of the asset claimed by the assessee, is as
per the estimate made by a registered valuer and assessing officer is of opinion that the value so
claimed is less than its fair market value.

2.

If the report of the registered valuer is not attached - If assessing officer is of the opinion that the
market value of the asset exceeds the value of the asset as claimed by the assessee and the difference
is more than 15% of the value claimed by assessee or more than 25,000/- of the value so claimed.

3.

The assessing officer is of opinion that, having regard to the nature of asset and other relevant
circumstances, is necessary to make the reference.

[CA INTER N07, 9 Marks]


Question: Mr. Thomas inherited a house in Jaipur under will of his father in May, 2008. The house was
purchased by his father in January 1980 for 2,50,000. He invested an amount of 7,00,000 in construction
of one more floor in this house in June 2010. The house was sold by him in November 2015 for 37,50,000.
The valuation adopted by the registration authorities for charge of stamp duty was 47,25,000 which was
not contested by the buyer, but as per assessees request, the Assessing officer made a reference to
Valuation office. The value determined by the Valuation officer was 47,50,000. Brokerage @ 1% sale
consideration was paid by Mr. Thomas to Mr. Sunil. The market value of house as on 01.04.1981 was
2,70,000.
You are required to compute the amount of capital gain chargeable to tax for A.Y. 2016-17.
Answer: Computation of capital Gain in the Hands of Mr. Thomas for the AY. 2016-17.
Particulars

Amounts ( )

Sales consideration (Being value determined by the Stamp Duty Authority)

47,25,000

Less

Indexed cost of acquisitions: (

29,18,700

Less

Indexed cost of improvement: (

Less

Cost of transfer (i.e. 1% of 37,50,000)

)
)

Long Term Capital Gains

10,64,276
37,500
7,04,524

Income Tax

10.44

S.115A: Computation of Capital Gain [Non-resident]


Question: Write short note on capital gains of non-residents and the method of conversion under Rule
115A u/h capital gain?
Answer: In case of a non-resident assessee, the capital gains arising from the transfer of shares or
debentures in an India company, shall be computed by converting
1.

The cost of acquisition of the asset.

2.

The expenditure incurred wholly and exclusively in connection with such transfer and

3.

Sale consideration received or accruing as a result of transfer of capital asset into the same foreign
currency as was initially utilized in the purchase of such shares or debentures. The capital gain so
computed in the foreign currency shall be reconverted into Indian Currency.
Indexation is not applicable

The method of conversion under Rule 115A is a follows:


(a) The Cost of Acquisition shall be converted at the average of the Telegraphic Transfer Buying Rate
(TIBR) and the Telegraphic Transfer Selling Rate (TTSR) as on the date of acquisition of shares /
debentures.
(b) The expenditure in connection with the transfer shall be converted at the average of TTBR and TTSR
as on the date of transfer of shares / debentures.
(c) The sale consideration shall be converted at the average of TTBR and TTSR as on the date of transfer
of shares / debentures.
(d) The capital gain computed in the foreign currency shall be converted into Indian currency by
applying TTBR as on the date of transfer of shares / debentures.

Question: Singar, a non-resident remits $35,000 to India on 9th October 2005. The amount is partly utilized
on 18th October 2005 for purchasing 50,000 shares in XYZ Ltd, an Indian Company, @ 30. These shares are
sold for 50 per share on 10th June 2015. Find out the capital gains chargeable to tax.
Date

Buying 1$

Selling $1

9th October, 2005

45

46

18th October, 2005

46

47

10th June, 2015

65

66

Answer:

Less

CR

Converted

Sale consideration

Average rate

65.5

25,00,000

38,168

Cost of acquisition

Average rate

46.5

15,00,000

32,259

Long term capital gain

Buying rate

65

3,84,085

Capital Gains

5,909

10.45

S.111A: Taxation on STCG from listed equity shares


Question: Write short note on tax on STCG from listed equity shares u/s 111A u/h capital gain?
Answer: Short Term Capital Gain from transfer of equity shares or units or equity oriented mutual fund
shall be taxable @ 15% if:
They are sold through recognized stock exchange.
Security transaction tax is chargeable on such transaction.
Transfer took place on or after 01.10.2004
No Deduction under VI-A shall be allowed from such income.
If normal income is less than basic exemption deficiency shall be allowed from short term capital
gain.

Income Tax

10.46

S.54, 54B, 54D, 54EC, 54F, 54G, 54GA & 54GB: Exemptions
[CA INTER N05, 4 Marks][CS EP D10 & D11, 2 marks & 3 marks]
Question: Briefly discuss about the provisions relating to claiming of exemption in respect of short -term capital gains, in order to
reduce tax liability. [54B, 54D and 54G]
Answer:
Asset
transferred

54
Residential
house,
whether stay or not

54B
Agri.
Land

54D
Land / Build. of
industrial
undertaking
(compulsorily
acquired)

54EC
Any assets

54F
Any assets other
than
Residential
House

Assessee
Capital Gain

I+HUF
LTCA

I+HUF
LTCA /
STCA
Used by
him or
his
parents
for
2
years
Agri.
Land
may be
in rural
or urban
Area

All
LTCA / STCA

All
LTCA

I+HUF
LTCA

Land / Building
for
industrial
undertaking

Specific Security
i.e., bond issued
by
Rural
Electrification
Corp. Ltd. (RECL)
and
National
Highway
Authority
(max
upto 50 lacs)

One
house

Within 2
years
after the
date of
transfer

Within 3 years
after the date
of the receipt
of
compensation

6 months from
the
date
of
transfer [provided
S.80C shall not be
allowed]

Prior use

New Assets

One residential house

Period for
investment

Purchase
one
residential house in
India, 1 year earlier or
2 year after the date
of
transfer
or

Used
years

for

54G
Plant
/
Machinery
or
Land / Building
for
industrial
undertaking in
urban area
All
LTCA / STCA

54GA
Plant
/
Machinery
or
Land / Building
for
industrial
undertaking in
urban area
All
LTCA / STCA

54GB
Residential house
[during 1.4.12 to
31.3.17]

residential

Plant
/
Machinery
or
Land / Building
for
Industrial
Undertaking in
Non-Urban Area
and
meeting
expenses of the
shifting

Equity shares in
eligible company

Purchase, 1 year
earlier or 2 year
after the date of
transfer
or
constructed within 3

Within 1 year
before or 3
years
after
transfer.

Plant
/
Machinery
or
Land / Building
for
Industrial
Undertaking in
Special
Economic Zone
and
meeting
expenses of the
shifting
Within 1 year
before or within
3 years after
transfer

May not own more


than one residential
house

1 "eligible company" means a company which fulfils the following conditions, namely:

Capital Gains

I+HUF

10.47

Shares acquired
within the due
date of filling of
return and the
1
eligible company

Withdrawal
of
exemption

Exemption
revoked

constructed within 3
years from the date of
transfer. In the case of
compulsory
acquisition from the
date of the receipt of
compensation
Transfer the new
asset within 3 years

STCG

years from the date


of transfer. In case
Compulsory
acquisition from the
date of the receipt of
compensation

should purchase
1
new asset within
one year from the
date
of
subscription
of
shares

Transfer
the new
asset
within 3
years

Transfer the
new asset
within 3 years

Transfer the new


asset within 3
years or borrows
securing such
securities

Transfer the new


asset within 3 years,
purchases any
residential house
within 2 years or
constructs any
residential house
within 3 years

Transfer the
new asset
within 3 years

Transfer the
new asset within
3 years

STCG

STCG

LTCG

LTCG

STCG

STCG

If the equity
shares sold within
5 years or if the
new asset not
purchased within
1 year or sold
within 5 years by
the eligible
company
LTCG

Note 1: Any amount remains unutilised, deposit in Capital Gain A/c scheme, on or before the date of furnishing the return of income (except S.54EC). If the deposited amount not
used for specific purpose within the due date will be charged u/s 45.
Note 2: Amount of exemption (except S.54F & S.54GB): WEL (amount invested or capital gain)
and for S.54F and S.54GB, exemption = WEL (

(i) it is a company incorporated in India during the period from the 1st day of April of the previous year relevant to the assessment year in which the capital gain
arises to the due date of furnishing of return of income under sub-section (1) of section 139 by the assessee;
(ii) it is engaged in the business of manufacture of an article or a thing;
(iii) it is a company in which the assessee has more than fifty per cent share capital or more than fifty per cent voting rights after the subscription in shares by the
assessee; and
(iv) it is a company which qualifies to be a small or medium enterprise under the Micro, Small and Medium Enterprises Act, 2006 (27 of 2006);
1
"new asset" means new plant and machinery but does not include
(i) any machinery or plant which, before its installation by the assessee, was used either within or outside India by any other person;
(ii) any machinery or plant installed in any office premises or any residential accommodation, including accommodation in the nature of a guest-house;
(iii) any office appliances including computers or computer software;
(iv) any vehicle; or
(v) any machinery or plant, the whole of the actual cost of which is allowed as a deduction (whether by way of depreciation or otherwise) in computing the
income chargeable under the head "Profits and gains of business or profession" of any previous year.

Income Tax

10.48

S.54H: Time Extension for Acquiring New Asset


Question: Write a note on extension of time for acquiring new asset or depositing or investing amount
of capital gain. [S.54H]
Answer: the period of acquiring new asset [for claiming benefits u/s 54] is calculated from the date of
receipt of the compensation, if the original asset compulsorily acquired under any law

[CA INTER M00, 7 Marks]


Question: Raghavan owned a residential house at Madurai, the original cost of which was 1,00,000. It
was acquired on 1.9.1999. He sold the house on 1.6.2015 for 8,00,000 and purchased another house on
30.5.2016 at Tiruchi for 6,00,000. The second house at Tiruchi was sold by him for 8,00,000 on 30.6.2018.
Discuss the impact of these transactions with regard to assessment to capital gains.
Answer:
Assessment year 2016-17
Sale consideration of Madurai house
Less

Less

Indexed cost of acquisition *

8,00,000

2,77,892

Long term capital gain

5,22,108

Exemption u/s 541 [WEL of (6,00,000 or 5,22,108)]

5,22,108

Long term capital gain

Nil

Assessment Year 2019-2000


Sale consideration of Tiruchi house
Less

Cost of acquisition [600000 5,22,108] being exemption availed u/s 54

8,00,000
77,892

as Tiruchi house is sold within 3 years


Short term Capital gain

7,22,108

[CMA RTP J12 & D12]


Question: Tina was the owner of two residential houses. On 5 th April, 2015, she disposed one of the
houses and utilized the entire sale proceeds to construct first floor on her second house which he
completed by 15th March, 2016. She seeks your advice as to the taxability of transaction to capital gains
under the provisions of Income Tax Act, 1961.
Answer: Vide S.54, where an assessee transfers a residential house being a Long-term Capital Asset and
the Long-term capital gain on such transaction is utilized for construction of another residential house,
within a period of 3-years from the date of transfer, is entitled for exemption. Construction of first floor in

Assumed 6,00,000 was deposited in capital gain deposited account scheme

Capital Gains

10.49

the existing building should be treated as independent residential unit entitled for exemption u/s 54 / 54F.
[P.V.NARASIMHAN 181 ITR 101]

[CMA RTP J11]


Question: On 16th January 2016, Suman sold agricultural land for 22 lacs. He incurred selling
expenses for 50,000. Compute capital gains:
If the land sold, was purchased on 1 st February 1999 for 4 lacs, and the land was used for agricultural
purposes by his mother. He again purchased agricultural land of 8 lacs on 25th January 2016.
Amount deposited in a scheduled bank under Capital Gains Deposit Scheme is 4 lacs on 6th April, 2016.
Answer:
Computation of Capital Gains for the A.Y. 2016-17
Consideration for transfer
Less

Less

22,00,000

Expenses on transfer

50,000

Net Consideration

21,50,000

Indexed Cost of Acquisition ( 4,00,000 1081/351)

12,31,909

Long-term Capital Gains

9,18,091

Less

Exemption u/s 54B Cost of New Land purchased

8,00,000

Less

Amount deposited in Capital Gains Account Scheme

4,00,000

4,00,000

before due date of furnishing return specified u/s 139(1)


Taxable long term Capital Gains

NIL

[CMA INTER D1, 1 Mark]


Question: On 17th January, 2016, M/s. Naina sold a house property and earned a long term capital gain
of1,02,50,000. She invested a sum of 50,00,000 in bonds specified in section 54EC on 8 th March, 2016. She
further invested a sum of 50,00,000 in same bonds on 8th May, 2016. Taxable Income of M/s. Naina for
the Assessment Year 2016-17 will be1:
a) 50,00,000
b) 52,50,000
c)

1,00,50,000

d) 2,50,000
[CMA RTP J12 & D12]
Question: Saptarshi acquired shares of G Ltd. on 15.12.2001 for 5 lacs which were sold on 14.6.15 for 19
lacs. Expense on transfer of shares is 40,000. He invests 8 lacs in the bonds of Rural Electrification
Corporation Ltd., on 16.10.2015.
1

Ans: B [52,50,000]

Income Tax

10.50

a.

Compute capital gain for the assessment year 2016-17.

b.

State the period for which the bonds should be held by the assessee. What will be the consequences if
such bonds are sold within the specified period?

c.

What will be the consequences if Saptarshi takes a loan against the security of such bonds?

Answer:
(a)

Consideration for transfer


Less

Less

Less

Computation of Capital Gains for the A.Y. 2016-17

Expenses on Transfer

19,00,000
40,000

Net Consideration

18,60,000

Indexed Cost of Acquisition [5,00,000 1081/426]

12,68,779

Long-term Capital Gains

5,91,221

Exemption u/s 54EC

5,91,221

Taxable long-term Capital Gain

NIL

(b) Saptarshi should not transfer or convert (otherwise than transfer) into money such bonds within 3
years from the date of their acquisition. If these bonds are transferred or converted into money within 3
years, capital gain exempted earlier shall attract taxability towards long-term capital gain of the previous
year in which such asset is transferred or converted into money.
(c) If any loan is taken against security of such bonds, it shall be taxable as long-term capital gains of the
previous year in which such loan is taken against the security of such bonds.

[CA IPCC M 14, 8 Marks]


Question: Mr. Roy, aged 55 years owned a Residential House in Ghaziabad. It was acquired by Mr. Roy
on 10-10-1986 for 6,00,000. He sold it for 53,00,000 on 4-11-2015. The stamp valuation authority of the
state fixed value of the property at 71,00,000. The Assessee paid 2% of the sale consideration as
brokerage on the sale of the said property.
Mr. Roy Acquired a Residential House property at Kolkata on 10-12-2015 for 10,00,000 and deposited
7,00,000 on 10-4-2016 and 5,00,000 on 15-6-2016 in the capital gains bonds of Rural Electrification
Corporation Ltd. He deposited 4,00,000 on 6-7-2016 and 3,00,000 on 1-11-2016 in the capital gain
deposit scheme in a Nationalized Bank for construction of an additional floor on the residential house
property in Kolkata.
Compute the Capital Gain chargeable to Tax for the Assessment Year 2016-17 and Income Tax chargeable
thereon assuming Mr. Roy has no other income.
Answer: Computation of taxable capital gains of Mr. Roy AY 2016-17

Sale consideration (S.50C WEH of sale value or stamp duty value)

Capital Gains

71,00,000

10.51

Less:

Expenses on transfer (2% of 53 lacs)

1,06,000

Net sale consideration


Less:

69,94,000
(

Indexed cost of acquisition

46,32,857

Long term Capital Gains


Less:

23,61,143

Exemption u/s 54

14,00,000

Exemption u/s 54 EC

7,00,000
21,00,000

Long Term Capital loss

2,61,143

Since Mr. Roy has no other income, he can set off 2,50,000 (the basic exemption limit) against his LTCG
income.

Less:

Tax is calculated as follows

On the first 2,50,000

Nil

On balance 11,143, @ 20%

2,229

Rebate u/s 87A

2,000

Balance
Add:

229

3% Education Cess

Total tax liability

236

Rounded off

240

Notes:
1.

5 lacs worth of REC bonds purchased on 15-6-2016 are not considered for exemption u/s 54EC, as
they are purchased more than 6 months after the date of transfer.

2.

3 lacs deposited in Capital Gains Deposit Scheme on 1-11-2016 is not considered for exemption u/s
54 since the same is deposited after 31-7-2016, the due date of filling return of income.

[CA INTER N06, 10 Marks]


Question: Mr. Malik owns a factory building on which he had been claiming depreciation for the
past few years. It is the only asset in the block. The factory building and land appurtenant thereto
were sold during the year. The following details are available.

Building completed in September, 2008 for

10,00,000

Land appurtenant thereto purchased in April, 2006 for

12,00,000

Income Tax

10.52

Advance received from a prospective buyer for land in May, 2007,

50,000

forfeited in favour of assessee, as negotiations failed


WDV of the building block as on 1.4.2015

8,74,800

Sale value of factory building in November, 2015

8,00,000

Sale value of appurtenant land

25,00,000

The assessee is ready to invest in long-term specified assets u/s 54EC, within specified time.
Compute the amount of taxable capital gain for the assessment year 2016-17 and the amount to be
invested u/s 54EC for availing the maximum exemption.
Answer:
Computation of taxable capital gain of Mr. Malik for A.Y.2016-17

Particulars

Factory building

Less

Sale price of building

8,00,000

WDV as on 1.4.2015

8,74,800

Short-term capital loss on sale of building (S.50)

(-) 74,800

Land appurtenant to the above building

Less

Sale value of land

25,00,000

Indexed cost of acquisition [(12,00,000 50,000 u/s 51)1081/519]

23,95,279

Long-term capital gains on sale of land

1,04,721

Chargeable long term capital gain 1 [amount required

29,921

to invest for availing exemption u/s 54EC]

[CA INTER M06 & M12, 8 & 8 Marks (similar in M12 Anusha instead of Mr. A)]
Question: Mr. A who transfers land and building on 2.1.2016, furnishes the following information:
1.

Net consideration received 17 lakhs.

2.

Value adopted by stamp valuation authority, which was not contested by Mr. A 19 lakhs.

3.

Value ascertained by Valuation Officer on reference by the Assessing Officer 20 lakhs.

4.

This land was distributed to Mr. A on the partial partition of his HUF on 1.4.1981. Fair market
value of the land as on 1.4.81 was 1,10,000.

5.

A residential building was constructed on the above land by Mr. A at a cost o f 3,20,000
(construction completed on 1.12.2012) during the financial year 2012-2013.

As per S.74, short term capital loss can be set-off against any income under the head Capital gains, longterm or short-term. Therefore, in this case, short-term capital loss of 74,800 can be set-off against long-term
capital gain of 1,31,021

Capital Gains

10.53

6.

Short-term capital loss incurred on sale of shares during the financial year 201 2-2013 2,05,000.

Mr. A seeks your advice as to the amount to be invested in NABARD bonds so as to be exempt from
clutches of capital gain tax.
Answer: Computation of Capital Gains of Mr. A for the Assessment Year 2015-16.
Particulars

(a)

Sale consideration

17,00,000

(b)

Stamp Duty Value

19,00,000

(c)

Whichever is higher of (a) or (b) [S.50C(1)]

19,00,000

(d)

Value as per valuation officer

20,00,000

Whichever is lower of (c) or (d) [S.50C(3)]

19,00,000

Full value of consideration (deemed)


Less

Less

Indexed cost of land (1,10,000 1081/100)

11,89,100

Indexed cost of building (3,20,000 1081/852)

4,06,009

Long-term capital gain

3,04,891

Brought forward short-term capital loss set off

2,05,000

Amount to be invested in NHAI / REC bonds [S.54EC]

99,891

[CA INTER N04, 6 Marks]


Question: Mr. `X furnishes the following data for the previous year ending 31.3.2016.
(a) Equity Shares of AB Ltd., 10,000 in number were sold on 31.5.2015, at 350 for each share.
(b) The above shares of 10,000 were acquired by `X in the following manner:
(i) Received as gift from his father on 1.6.1980 (5,000 shares) the market price on 1.4.81 50 per
share.
(ii) Bonus shares received from AB Ltd. on 21.7.1996 (2,000 shares).
(iii) Purchased on 1.2.2005 at the price of 125 per share (3,000 shares).
(c) Purchased one residential house at 25 lakhs, on 1.9.2016 from the sale proceeds of shares.
(d) `X already owns a residential house, even before the purchase of above house.
You are required to compute the taxable capital gain. He has no other source of income chargeable to
tax.

Income Tax

10.54

Answer: It is also assumed that the transaction of sale of shares is not exempt under section 10(38).
Computation of taxable capital gain of Mr. X for A.Y. 2016-17

Sale consideration received on sale of 10,000 shares @ 400 each


Less

Indexed cost of acquisition

(a)

5,000 shares (gift from father on 1.6.1980) *

(b)

2,000 bonus shares received from AB Ltd

(c)

3000 shares purchased on 1.2.2005 *

Less

40,00,000

27,02,500
Nil

8,44,531

35,47,031

Long term capital gain

4,52,969

Exemption u/s 54F

2,83,106

Taxable long term capital gain

1,69,863

[CMA RTP J11]


Question: Bipasha purchased jewellery worth 2,20,000 during 1986-87. During the year 1991-92, she
further purchased jewellery worth 3,50,000. All the jewellery was sold by her on 15.6.15. The jewellery
purchased in 1986-87 was sold for 20 lacs and that purchased in 1991-92 was sold for 32 lacs. On 26.6.15
she deposited 50 lacs in capital gains scheme account. On 21.10.15 withdrawing from the Deposit
Account, she utilized 48 lacs for purchase of a residential house property in Kolkata. On the date of
transfer, she owns only one residential house.
Answer:
Computation of Capital Gains for the A.Y. 2016-17

a)

On transfer of jewellery purchased during 86-87 Consideration for transfer

20,00,000

Less

Indexed Cost of Acquisition (

16,98,714

3,01,286

Long-term Capital Gains


b)

On transfer of jewellery purchased during 1991-92 Consideration for transfer

32,00,000

Less

Indexed Cost of Acquisition (

19,01,256

Long term capital gains

12,98,744

In order to avail the maximum benefit u/s 54F, the exemption should be computed as follows:
Total long-term Capital Gain
Less

Exemption u/s 54F *

16,00,030
+

Taxable Long-term Capital Gains

15,38,490
61,540

Capital Gains

10.55

[CMA RTP J11]


Question: P Ltd., owns an industrial undertaking manufacturing chemicals in Bangalore owns the
following assets
a) Plant and machinery (WDV 5 lacs) sold for 15 lacs.
b) Building (WDV 12 lacs) sold for 60 lacs.
c)

Furniture and fixtures (WDV 50,000) sold for 1,80,000

d) Land cost of acquisition 5,00,000 during 1987-88 and sold for 30 lacs
The industrial undertaking was shifted as per policy of the Government from urban area to other area.
The new assets acquired during the period 1.1.16 to 31.3.16 are plant and machinery 20 lacs; buildings
40 lacs.
Compute capital gain chargeable to tax for the assessment year 2016-17.
Answer:

Short-term Capital Gains on Depreciable assets


(i)

Plant & Machinery (15,00,000 5,00,000)

10,00,000

(ii)

Buildings (60,00,000 12,00,000)

48,00,000

(iii)

Furniture & Fixtures (1,80,000 50,000)

1,30,000

59,30,000

Long-term capital gains on industrial land:

Less

Consideration for transfer

30,00,000

Indexed Cost of Acquisition (5,00,000 1081/150)

36,03,333

Total Capital Gains


Less

6,03,333
53,26,667

Exemption u/s 54G


Plant & Machinery

20,00,000

Building

40,00,000

but restricted to 5,91,221 [53,26,667 1,30,000, being STCG on furniture,

65,00,000
5,91,221

not eligible for the purpose of claiming exemption u/s 54G


Short term Capital Gains (on furniture)

1,30,000

[CA INTER N07, 8 Mark]


Question: Mrs. Malini Hari shifted her industrial undertaking located in corporation limits of Faridabad,
to a Special Economic Zone (SEZ) on 1.12.2015.
The following particulars are available:
(a)

Land: Purchased on 20.01.2010

4,26,000

Sold for
(b)

22,00,000

WDV of Building as on 01.04.2015 [Construction completed on 14.03.2010]


Sold for

8,20,000
11,89,000

Income Tax

10.56

(c )

WDV of cars as on 01.04.2015

7,40,000

Sold for

6,00,000

(d )

Expenses on shifting the undertaking

1,15,000

(e )

Assets acquired for the undertaking in the SEZ (on or before 25.06.2016)

(i)

Land

3,00,000

(ii)

Building

5,00,000

(iii)

Computers

1,00,000

(iv)

Car

4,20,000

(v)

Machinery (Second hand)

2,00,000

(vi)

Furniture

50,000

There is no intention of investing in any other asset in this undertaking. Compute the exemption available
under Section 54GA for the assessment year 2016-17.
Answer: Computation of exemption available u/s 54GA of the Capital Gain:
Particular
Sales consideration
Less

Building

22,00,000

11,89,000

6,00,000

(8,20,000)

(7,40,000)

14,08,753

3,69,000

(1,40,000)

(LTCG)

(STCG)

(STCL)

12,66,000

3,69,000

Nil

1,42,753

Nil

(1,40,000)

Cost of acquisition
Indexed cost of requisition (Land) (

Cars

7,91,247

Gross Gain / Loss

Less

Land

Exemption u/s 54 GA:


Total investment except furniture (include cost of shifting)
[3,00,000+5,00,000+1,00,000+4,20,000+2,00,000+1,15,000] = 16,35,000

Net Gain / Loss

LTCG

STCL

Income chargeable under the head Capital Gain:


LTCG (on Land)
Less

STCL (on Car)


LTCG

Capital Gains

1,42,753
(1,40,000)
2,753

10.57

[CMA RTP D10]


Question: R, an individual resident in India bought 1,000 equity shares of 10 each of A Ltd. at 50 per
share on 30.5.2015. He sold 700 equity shares at 35 per share on 30.9.2015 and the remaining 300 shares
at 25 per share on 20.12.2015. A Ltd. declared a dividend of 50%, the record date being 10.8.2015. R sold
on 01.02.2016 a house from which he derived a long term capital gain of 75,000. Compute the capital
gain arising to R for the Assessment Year 2016-17.
Answer:
Assessee : Mr. R

Previous Year : 2015-2016

Assessment Year : 2016-17

Analysis as per Section 94(7):


1.

In the given case, shares were purchased on 30.05.2015 within 4 months prior to Record Date i.e.
10.08.2015.

2.

Shares are sold on 30.09.2015 and on 20.12.2015. The first sale is within 3 months and the next one is
after that period. Dividend has been received @ 50% on Face Value 10 per share on the entire 1,000
shares.

3.

Loss arising out of transfer of the first 700 shares should be ignored to the extent of such dividend.
Computation of Taxable Capital Gains
Particulars
Date of Transfer

30.09.15

20.12.15

Number of Shares

700

300

Sale price / share

35

25

() 24,500

7,500

Cost of Acquisition

() (35,000)

(15,000)

Short Term Capital Loss

() (10,500)

(7,500)

3,500

Nil

(7,000)

(7,500)

Consideration for transfer


Less

Add: Dividend (700 shares of Face Value 10 per share @ 50%) ()


[Transferred within 3 Months from Record Date]
Short Term Capital Loss [after considering S.94(7)] (Note) ()
Total Short Term Capital Loss ()

(14,500)

Long Term Capital Gains (Given) ()

75,000

Taxable LTCG (after set off u/s 71) ()

60,000

Note: Since 300 shares are transferred on 20.12.15 which is 3 months after the reorder date, dividend
received need not be adjusted against Short Term Capital Loss.

[CMA RTP J12 & D12]


Question: Explain the tax treatment of income from Deep Discount Bonds (DDBs).
Answer: Deep discount bonds as clarified vide Circular No. 2/2002 as follows:

Income Tax

10.58

1.

Income based on market value


i.

Income treated as interest for investors

ii. Income treated as business income for traders


2.

For original subscribers, Income = difference between market value on 31 st March of the previous year
and 1st April of the previous year
For subsequent purchases, income = difference between market value on 31 st March of the previous
year and cost of purchase of the bond

3.

If there is a transfer before maturity:


i.

For the Investor, Short term Capital Gains = Sale Price less Cost of Bond;

ii. For Traders, Business Income = Sale price less cost of bond.
4.

Cost of bond = Cost of acquisition + Income already taxed up to the date of transfer.

5.

If there is a redemption on maturity:


i.

For the Investor, Interest Income = Redemption Price less market value as on the last valuation
date, immediately preceeding the maturity date.
In case of a trader, this interest income would be construed as Business Income.

ii. For subsequent purchasers, Interest Income = Redemption price less cost of the bond to such
purchaser.
In case of a trader, this interest income would be construed as Business Income.
Where, Cost of bond = cost of acquisition + income already taxed by the bond holder up to the
date of redemption

[CMA RTP J10]


Question: X received a vacant site under his fathers will. The value of the site on 31.3.2016 is 15 Lakhs.
As per terms of the Will in the event X wants to sell the site he should offer it to his brother for sale at
10 Lakhs. X, therefore, claims that the value of the site should be taken at 10 Lakhs as at 31.3.2016. Is
the claim correct?
Answer:
1.

As per Rule 21 of Schedule III to the Act, the price or other consideration for which any property
may be acquired by or transferred to any person under the terms of a deed of trust or through or
under any restrictive agreement in any instrument of transfer shall be ignored for the purpose of
determining the value under the provisions of the Schedule.

2.

In view of the above, the value of the site should be taken as 15 Lakhs and not as 10 Lakhs.

3.

Therefore, claim of X is incorrect.

Capital Gains

10.59

[CMA INTER D12, 1Mark]


Question: Unabsorbed loss under the head Capital gains shall be carried forward for a period of ______
assessment years immediately following the assessment year in which such loss was incurred 1.

[CMA INTER SY08, D13, 1 Mark]


Question: When entire net consideration has been invested by an individual towards subscription of
shares of an eligible company the exemption u/s. 54GB of the Income Tax Act 1961 would be2
a) NIL
b) 10% of capital gain
c)

50% of capital gain

d) 100% of capital gain

[CMA INTER SY08, D13, 1 Mark]


Question: Chapter VIA deduction _______ (shall / shall not) be allowed in respect of income from short
term capital gain.3

[CMA INTER SY08, D13, 1 Mark]


Question: A short-term capital loss in the current year can be set-off only against:4
a) Any capital gain in the current year
b) Any short-term capital gain only
c)

Any long-term capital gain only

d) Any income under other heads of income

[CMA RTP J12 & D12]


Question: Well Wishers & Associates, a partnership firm, is holding land. This firm is not engaged in real
estate business. The land was sold during the year. Discuss taxability, whether, this would be assessed to
tax as business income or capital gain.
Answer: Land held by partnership firm, which is not engaged in real estate business, would be treated as
fixed asset of the firm. Transfer of the same is assessable as capital asset, hence capital gains and not as
business income. [Mohakampur Ice & Storage 281 ITR 354 (All.)]

Answer: 8 years
Answer: (d) 100% of capital gain
3 Answer: shall. However, in case it is short term capital gain in respect of transaction in equity shares in a company
chargeable to STT (u/s 111A) deduction under Chapter VIA shall not be allowed. The question does not mention the
type of transaction. Hence, both answers would be possible.
4
Answer: (a)
2

Income Tax

10.60

[CMA INTER SY12, D13, 1 Mark]


Question: Long-term capital gain arising from sale of listed shares in a recognized stock exchange (STT
paid) is exempt u/s _________ of the Income-tax Act, 19611:
a) 10(35)
b) 10(37)
c)

10(38)

d) 10(36)

Answer: (c)

Capital Gains

10.61

Additional CA Exam Questions


[CA INTER M08, 2 Marks]
Question: State true or false: Ms. Vasudha contends that sale of a work of art held by her is not
eligible to capital gains tax; is she correct?
Answer: No., Ms. Vasudhas contention is not correct. As per the amendment made under the Finance
Act, 2007 the definition of capital asset [U/s 2(14) has got widen. Now, archaeological collections,
drawings, paintings, sculptures or any other work of an art are included within the meaning of the term
capital asset.

[CA INTER N08, 4 Marks]


Question: Answer the following with regard to the provisions of the Income-tax Act, 1961:
Mrs. X an individual resident woman wanted to know whether Income-tax is attracted on sale of gold
and jewellery gifted to her by her parent at the occasion of her marriage in the year 1979 which was
purchased at a total cost of 2,00,000?
Answer: Capital gains in the hands of Mrs. X - Long-term capital gains arising on transfer of gold and
jewellery is chargeable to tax in the hands of Mrs. X. Cost of acquisition will be taken as cost of the gold
and jewellery to her parents or the Fair Market Value of the asset on April 1, 1981, whichever is higher
Long-term capital gain will be calculated after deducting indexed cost of acquisition.

[CA INTER N10, 4 Marks]


Question: How will you calculate the period of holding in case of the following assets?
(1) Shares held in a company in liquidation
(2) Bonus shares
(3) Flat in a co-operative society
(4) Transfer of a security by a depository (i.e., demat account)
Answer: Period of Holding S.2(42A) of the following asset:
1.

Shares held in a company in liquidation: Period of holding will be taken from the date of acquisition
of shares to the date on which the company goes into liquidation:

2.

Bonus Shares: The period of holding shall be taken from the date of allotment of shares till the date
preceding the date of transfer of shares.

3.

Flat in a co-operative society: The period of holding shall be taken from the date of allotment of the
flat from the society to the date preceding the date of transfer of the flat.

4.

Transfer of a security by a depository: Period of holding shall be calculated on FIFO basis. For
deciding FIFO technique the date of entry in Demat Account is significant and the date of purchase
of security is irrelevant.

Income Tax

10.62

[CA INTER M08, 1 Mark]


Question: Fill up the blanks: Zero Coupon Bond means a bond on which no payment and benefits
are received or receivable before maturity or redemption.
Answer: True - As per section 2(48), Zero Coupon Bond means a bond issued by any infrastructure
capital company or infrastructure capital fund or a public sector company on or after 1 st June 2005, in
respect of which no payment and benefit is received or receivable before maturity or redemption
from such issuing entity and which the Central Government may notify in this behalf.

[CA INTER J09, 2 Marks]


Question: State True or False, with reasons: Alienation of a residential house in a transaction of
reverse mortgage under a scheme made and notified by the Central Government is treated as
"transfer" for the purpose of capital gains.
Answer: False. As per clause (xvi) inserted by the Finance Act, 2008 in section 47, such alienation in a
transaction of reverse mortgage under a scheme made and notified by the Central Government is not
regarded as "transfer" for the purpose of capital gains.

[CA INTER M06, 2 Marks]


Question: State True or False, with reasons: Zero coupon bonds of eligible corporation, held for
more than 12 months, will be long-term capital assets.
Answer: True. U/s 2(42A), zero coupon bond held for not more than 12 months will be treated as a
short-term capital asset. Consequently, such bond held for more than 12 months will be a long-term
capital asset.

[CA INTER M06, 2 Marks]


Question: State True or False, with reasons: In the case of a dealer in shares, income by way of
dividend is taxable under the head "Profits and gains of business or profession".
Answer: False. In the case of all assesses, dividend income is taxable under the head "Income from
other sources", in view of the provisions of section 56(2)(i).

[CA INTER N08, 2 Marks]


Question: State with reason, true or false: capital gain of 75 lacs arising from transfer of long term capital
assets will be exempt from tax if such capital gain is invested in the bonds redeemable after three years, issued by
NHAI u/s 54EC.
Answer: True. as per Sec. 54EC
1.

The assets transfer should be a long term assets.

2.

Within a period of 6 months the capital gain must be invested in long term asset for a locking period
of 3 years in a National highway authority of India.

Capital Gains

10.63

[CA INTER M05, 1 Mark]


Question: Choose the correct answer: In respect of shares held as investment, while computing the
capital gains, securities transaction tax paid in respect of sale of listed shares sold in a recognized stock
exchange,1
(a) Is deductible upto 1,00,000
(b) Is deductible upto 2,00,000
(c) Is deductible where capital gains is below 5,00,000
(d) Is not deductible at all.

[CA INTER M05, 1 Marks]


Question: Choose the correct answer: For an individual who has derived short-term capital gains of
40,000 from transfer of listed equity shares after 1.10.2010, with other income of 20,000 (these two items
above making up his total income), income-tax payable in respect of short-term capital gains is2
(a) 4000
(b) 8000
(c) 1000
(d) None of the above.

[CA INTER N05, 1 Mark]


Question: Fill in the blanks having regard to the provisions of the Income-tax Act, 1961: Where a person
transfers capital asset to a firm in which he becomes partner, the full value of consideration in the context
of capital gain computation, will be _______
Answer: The amount recorded in the books of account of the firm as the value of the capital asset.

[CA INTER N10, 5 Marks]


Question: Mr. A is a proprietor of Akash Enterprises having 2 units. He transferred on 1.4.2015 his unit 1
by way of slump sale for a total consideration of 25 Lacs. The expenses incurred for this transfer were
28,000/-. His Balance Sheet as on 31.3.2015 is as under:

Liabilities
Own Capital

1
2

Total
15,00,000

Assets
Building

Unit 1

Unit 2

Total

12,00,000

2,00,000

14,00,000

Revaluation Reserve (for building of unit 1)

3,00,000

Machinery

3,00,000

1,00,000

4,00,000

Bank loan (70% for unit 1)

2,00,000

Debtors

1,00,000

40,000

1,40,000

D - Is not deductible at all


D - None of the above

Income Tax

10.64

Trade creditors (25% for unit 1)


Total

1,50,000
21,50,000

Other assets
Total

1,50,000

60,000

2,10,000

17,50,000

4,00,000

21,50,000

Other information:
1.

Revaluation reserve is created by revising upward the value of the building of unit 1.

2.

No individual value of any asset is considered in the transfer deed.

3.

Other assets of unit 1 include patents acquired on 1.7.2013 for 50,000/- on which no depreciation has
been charged.

Compute the capital gain for the assessment year 2016-17.


Answer: Capital gain on transferee of unit 1 by way of slump sale u/s 50B
Amount ( )
Sale proceeds
Less

Less

25,00,000

Expenses of transfer

28,000

Net consideration proceeds

24,72,000

Cost of acquisition or improvement (See Note 1)

12,50,625

Long term Capital Gain

12,21,375

Note 1: Cost of acquisition or improvement = Net worth of the undertaking


Net worth of the undertaking
Aggregate value of total assets
Depreciable Assets
Building excluding revaluation reserve

9,00,000

Machinery

3,00,000

Patents [Note 2]

28,125

12,28,125

Other Assets
Debtors

1,00,000

Other assets except patent

1,00,000

2,00,000
14,28,125

Value of Liabilities
Bank loan (200000 x 70%)
Trade creditors (150000 x 25%)
Net worth

140,000
37,500

1,77,500
12,50,625

Note 2: Since patents have been acquired 1.7.2013 therefore depreciation @ 25% p.a. will be charged for
two years on WDV basis. Therefore book values as on 31.3.2015 = 50000 x .75 x .75 = 28125.

Capital Gains

10.65

Note 3: For the purpose of computation of net worth, the WDV determined as per section 43(6) has to be
considered in the case of depreciable asset. The said problem has been solved assuming that B/S value of
3 lakhs and 9 lakhs (12 lakhs 3 lakhs) represents the written down value of machinery & building
respectively, of unit 1.

[CA INTER N09, 6 Marks]


Question: Compute the net taxable capital gains of Smt. Megha on the basis of the following information:
A house was purchased on 1.05.2003 for 4,50,000 and was used as a residence by the owner. The owner
had contracted to sell this property in June, 2013 for 10 lacs and had received an advance of 70,000
towards sale. The intending purchaser did not proceed with the transaction and the advance was
forfeited by the owner. The property was sold in April, 2015 for 15,00,000. The owner, from out of sale
proceeds, invested 4 lacs in a new residential house in January, 2016.
Answer: Computation of net taxable capital gains of Smt. Megha for the A.Y. 2016-17

Particulars
Full value of consideration
Less

15,00,000

Expenses on transfer

Nil

Net sale consideration


Less

Less

15,00,000

Indexed cost of acquisition (

8,87,214

Long term capital gain

6,12,786

Exemption under S.54

4,00,000

Taxable long term capital gain

2,12,786

[CA INTER M02, 8 Marks]


Question: Mr. Sunder furnishes the following particulars for the previous year ending 31.3.2015, and
requests you to compute the taxable capital gain:
(i) He had a residential house, inherited from father in 1996 the fair market value of which as on 1.4.81 is
5 lakhs.
(ii) In the year 1997-98, further construction and improvements cost 6 lakhs.
(iii) On 10.5.2015, the house was sold for 80 lakhs. Expenditure in connection with transfer 50,000.
(iv) On 20.12.2015, he purchased a residential house for 15 lakhs.
Answer: Computation of Capital Gains:

Sale proceeds

80,00,000

Less

Indexed cost of acquisition *

Less

Indexed cost of improvement *

Income Tax

54,05,000
+

19,59,517

10.66

Less

Expenses of transfer

50,000
6,35,483

Less

Exemption u/s 54

6,35,483

Long term capital gain

NIL

The following assumption has been made:


(a) Mr. Sunders father purchased the property before April 1, 1981.
(b) Cost of the house to Xs father was less than 5,00,000.

[CA INTER N03, 4 Marks]


Question: Ms.Vimla sold a residential building at Jodhpur for 15,00,000 on 01-07-2015.
The building was acquired for 1,50,000 on 01-06-2000. She paid brokerage @ 2% at the time of sale of
the building. She invested 7 lakhs in purchase of a residential building in December 2014 and
deposited 2 lakhs in NABARD Capital Gains Bond in March, 2016. Compute her taxable capital
gain.
Answer: Computation of taxable capital gain of Ms. Vimla for A.Y.2016-17
Particulars
Sale price of residential building
Less

Brokerage @ 2%

15,00,000
30,000

Net consideration
Less

14,70,000

Indexed cost of acquisition [1,50,000 x 1081 / 406]

3,99,384
10,70,616

Less

Deduction u/s.54 for purchase of new residential house in Dec. 2013

7,00,000

Taxable long term capital gain

3,70,616

Note One of the conditions for claiming deduction u/s 54EC for the investment in REC/NHAI
Capital Gains bonds is that the deposit should be made within 6 months from the date of transfer. In
this case, the transfer took place on 1.7.15 and the 6 months period within which the deposit should
be made for the purpose of section 54EC would expire by 1.1.16. The investment in REC/NHAI
Capital Gains bonds was made only in March 2016. Therefore, the assessee is not eligible for section
54EC deduction.

[CA INTER M04, 4 Marks]


Question: Bala sold his vacant site on 21.09.2015 for 7,00,000. It was acquired by him on 01.10.1999
for 1,50,000. The State stamp valuation authority fixed the value of the site at the time of transfer @
13,00,000. Compute capital gains in the hands of Bala and give your reasons for computation.

Capital Gains

10.67

Answer:

Computation of capital gains of Shri Bala for the A.Y.2016-17


Deemed sale consideration as per section 50B
Less

Indexed cost of acquisition (

13,00,000

4,16,838

Taxable long term capital gain

8,83,162

Note: According to section 50C(1), where the consideration received or accruing as a result of the
transfer of land or building or both is less than the value adopted or assessed by the State stamp
valuation authority for the purpose of payment of stamp duty in respect of such transfer, then the
value so adopted or assessed by the State stamp valuation authority shall be deemed to be the full
value of the consideration received or accruing as a result of the transfer.
In this case, since the consideration of 7 lacs received on transfer of land is less than the value of 13
lacs fixed by the State stamp valuation authority, the value adopted by the State stamp valuation
authority is deemed to be the full value of consideration and capital gains is calculated accordingly.

[CA INTER N01, 6 Marks]


Question: Arjun furnishes the following particulars and requests your advise as to the liability to capital
gains for the Assessment Year 2016-17:
(i) Jewellery purchased by him on 10.3.2001 for 1,05,000 was sold by him for a consideration of
3,15,000 on 2.11.2015.
(ii) He incurred expenses:
(a) At the time of purchase 2,000
(b) At the time of sale (for brokerage) 4,000
(iii) He invested 50,000 in bonds with National Highway Authority of India out of sale consideration.
On these facts:
(a) Compute the capital gains chargeable to tax.
(b) Whether Arjun would be entitled to any exemption?
Answer: Computation of Capital Gains for Mr. Arjun for Assessment Year 2016-17.

Particulars
Sale Consideration

3,15,000

Less

Indexed Cost of Acquisition *(

Less

Expense on transfer

Less

2,84,894
4,000

Long Term Capital Gain

26,106

Exemption u/s 54EC Investment in bonds of NHAI

26,106

Taxable Long Term Capital Gain

Income Tax

Nil

10.68

[CA INTER M01, 10 Marks]


Question: Mr. A sold shares of a public limited company for 5,00,000 on 1.10.2015, which had been
acquired by him in October, 1990 for 50,000. He wants to utilize the said amount of sale consideration
for purchase or construction of a new residential house. He already owns one residential house at the
time of sale of the shares i.e., on 1.10.2015. He has deposited 4,00,000 under the Capital Gains Deposit
Account scheme with a specified Bank on 30.4.2015. Ascertain the Capital gain taxable in As hands for
Assessment Year 2016-17 and advice him as to what further action he has to take; to avail of the
exemption.
Answer: If A has transferred these shares in a recognized stock exchange in India he will be subject to
securities transaction tax and consequently, the long term capital gain will be exempt from tax. Otherwise
if these shares are transferred outside a recognized stock exchange the manner of income computation
will be as under:

Sales Consideration
Less

5,00,000

Indexed cost of acquisition *

3,14,244

Long term capital gain


Less

1,85,756

Exemption under Section 54F *

Long term capital gain

1,48,605
37,151

[CA INTER N08, 12 Marks]


Question: Paulomi has transferred 1,000 shares of Hetal Ltd., (which she acquired at a cost of 10,000
in the financial year 2009-10) to Dhaval, her brother, at a consideration of 3,00,000 on 15.5.2015
privately.
During the financial year 2015-16, she has paid through e-banking 15,000 towards medical
premium, 50,000 towards L.I.P. and 25,000 towards PPF.
Assuming she has no other source of income, compute her total income and tax paya ble for the
Assessment Year 2016-17.
Answer: Computation of total income and tax liability of Paulomi for Assessment year 2016-17

Particulars
Sale consideration
Less

3,00,000

Indexed cost of acquisition *

17,104

Long term capital gain

2,82,896

Total income

2,82,896

Tax liability
Income-tax @ 20% on 32,896 [2,82,896 2,50,000]

Capital Gains

6,579

10.69

Less

Rebate u/s 87A

2,000
4,579

Add

Education Cess @ 2%

91

Secondary and Higher Education Cess @ 1%

46

Total tax liability

4,716

Notes
1.

As per section 112 of Income-tax Act, 1961, deductions under Chapter VI-A are not allowable
against long term capital gain. Therefore, Paulomi is not entitled to deduction u/s 80C in respect
of payment of LIP and contribution to PPF. She is also not entitled to deduction u/s 80D in
respect of medical insurance premium paid by her.

2.

Since Paulomi has not transferred her shares through the Stock Exchange and, therefore, has not
paid securities transaction tax, she is not entitled to claim exemption u/s 10(38) in respect of long
term capital gain.

3.

She is, however, entitled to reduce the long-term capital gain by the unexhausted basic
exemption limit and pay tax on the balance @20% as per section 112. In this case, since she has no
other source of income, the entire basic exemption limit of 2,50,000 can be reduced from the
long-term capital gain and rebate u/s 87A 2,000 also availed. It is presumed that Paulomi is a
resident assessee.

[CA IPCC N12, 8 Marks]


Question: Mr. C inherited from his father 8 plots of land in 1980. His father had purchased the plots in
1960 for 5 lakhs. The fair market value of the plots as on 1-4-1981 was 8 lakhs. (1 lakh for each plot)
On 1st June 2001, C started a business of dealer in plots and converted the 8 plots as stock in trade of his
business. He recorded the plots in his books and 45 lakhs being the fair market value on that date. In
June 2005, C sold the 8 plots for 50 lakhs.
In the same year he acquired a residential house property for 45 lakhs. He invested an amount of 5
lakhs in construction of one more floor in his house in June 2006. The house was sold by him in June 2015
for 63,50,000/The valuation adopted by the registration authorities for charges of stamp duty was 86,50,000. As per
the assessee request the assessing officer made a reference to a valuation officer. The value determined
by the valuation officer was 87,20,000. Brokerage of 1% of sale consideration was paid by C.
Give the tax computation for the relevant assessment years with reasoning.

Computation of total income and tax liability of Mr. C for A.Y. 2016-17

Capital Gains on sale of residential house property


Sale consideration (S.50C WEL of (WEH of Sale value or Stamp duty)

Income Tax

86,50,000

10.70

or value as per valuation officer)


Less

Brokerage @ 1% of sale consideration

63,500

Net Sale consideration


Less

85,86,500

Indexed cost of acquisition *

97,87,726
10,41,426

Indexed cost of improvement

1,08,29,152
Less

Long-term capital Loss (Total Income)

22,42,652

Tax on total income

Nil

[CA INTER M98, 6 Marks]


Question: In April 2003, S subscribed to the first issue of equity capital of a public limited company (face
value of each share was 100) to the extent of 25,000. In 2011, the company converted the face value of its
shares from 100 to 10 each. Half of the holdings of the shares held by S was sold by him in October,
2015 for 50,000. S had to pay a brokerage of 2% on sale.
What is the nature of gains realized and compute the same.
Answer:
Computation of Capital gain

Sale proceeds of half holding

50,000

Less

Indexed cost of acquisition [12,500 1081/463]

29,185

Less

Brokerage (2% of 50,000)

1,000

Long term Capital gain

19,815

[CA IPCC N11, 8 Marks]


Question: Mr. Selvan, acquired a residential house in January, 2001 for 10,00,000 and made some
improvements by way of additional construction to the house, incurring expenditure of 2,00,000 in
October, 2005. He sold the house property in October, 2015 for 75,00,000. The value of property was
adopted as 80,00,000 by the State stamp valuation authority for registration purpose. He acquired a
residential house in January, 2016 for 25,00,000. He deposited 20,00,000 in capital gains bonds issued by
National Highway Authority of India (NHAI) in June, 2016. Compute the capital gain chargeable to tax
for the assessment year 2016-17.
What would be the tax consequence and in which assessment year it would be taxable, if the house
property acquired in January, 2016 is sold for 40,00,000 in March, 2018 ?

Capital Gains

10.71

Answer: Computation of Capital Gains Chargeable to tax for A.Y. 2016-17.

Particulars

Less:

Sale consideration (S.50C WEH of CA or Stamp Duty Value)

80,00,000

Indexed Cost of Acquisition

26,62,562
4,35,010

Indexed Cost of Improvement

49,02,428
Less:

Capital Gains exemption u/s 54

Less

Capital gains exemption u/s 54EC is available only if deposited within 6 months.

25,00,000
Nil

In this case deposit made after 6 months is disqualified


Taxable Capital Gains

24,02,428

Taxable in AY 2018-19, if newly purchased

HP deducted u/s 54 sold within 3 years


Sale consideration
Less

40,00,000

Cost of acquisition

25,00,000

Capital gains exempted earlier

(25,00,000)

Short-term Capital Gains

Nil
40,00,000

[CA INTER M09, 8 Marks]


Question: Mr. Kumar is the owner of a residential house which was purchased in September 1997 for
5,00,000. He sold the said house on 5th August, 2015 for 24,00,000. Valuation as per stamp valuation
authority of the said plot of land was 45,00,000. He invested 8,00,000 in NHAI Bonds on 12 th January
2016. He purchased a residential house on 8th September, 2015 for 12,00,000. He gives other particulars
as follows:
Interest on Bank Deposit

32,000

Investment in public provident fund

12,000

You are requested to calculate the taxable income for the assessment year 2016-17 and the tax liability, if
any.
Answer: Computation of taxable income

Sales consideration
Less

45,00,000

Indexed cost of acquisition *


Long-term capital gain

Income Tax

16,32,931
28,67,069

10.72

Less

Exemption under section 54

12,00,000

Exemption u/s 54EC not available

Nil

as invested after 6 months from the date of sale


Taxable long-term capital gain

16,67,069

Income from other sources

32,000

Gross total income


Less

16,99,069

Deduction under section 80C

12,000

Taxable income (rounded off)

16,87,069

[CA IPCC M15, 8 Marks]


Question: Ms. Mohini transferred a house to her friend Mr. Ragini for 35,00,000 on 01-10-2015. The Sub
Registrar valued the land at 48,00,000. Ms. Mohini contested the valuation and the matter was referred
to Divisional Revenue Officer, who valued the house at 41,00,000. Accepting the said value, differential
stamp duty was also paid and the transfer was completed.
The total incomes of Mohini and Ragini for the assessment year 2016-17, before considering the transfer
of said house are 2,80,000 and 3,45,000, respectively. Ms. Mohini had purchased the house on 15 th May
2010 for 25,00,000 and registration expenses were 1,50,000.
You are required to explain provisions of Income-tax Act, 1961 applicable to present case and also
determine the total income of both Ms. Mohini and Ms. Ragini taking into account the above said
transactions.
Answer:
Total income of Ms. Mohinin

Less:

Sale Consideration

41,00,000

Indexed COA

40,29,044
70,956

Other Income

2,80,000

Capital Gain

3,50,956

Total Income of Ms. Ragini


Income from other Sources
Less

Immovable Property for inadequate consideration (41 lac 35 lac)

6,00,000

Other Income

3,45,000
9,45,000

Capital Gains

10.73

[CA INTER M05, 6 Marks]


Question: Mr. Y submits the following information pertaining to the year ended 31 st March, 2016:
(1) On 30.11.2015, when he attained the age of 60, his friends in India gave a flat at Surat as a gift, each
contributing a sum of 20,000 in cash. The cost of the flat purchased using the various gifts was 3.4
lacs.
(2) His close friend abroad sent him a cash gift of 25,000 through his relative, for the above occasion.
(3) Mr. Y sold the above flat on 30.1.2016 for 3.6 lacs. The Registrars valuation for stamp duty purposes
was 3.7 lacs. Neither Mr. Y nor the buyer, questioned the value fixed by the Registrar.
(4) He had purchased some equity shares in X Pvt. Ltd. on 5.2.2014 for 3.5 lacs. These shares were sold
on 15.3.2016 for 2.8 lacs.
You are requested to calculate the total income of Mr. Y for the assessment year 2016-17.
Answer: Computation of Total Income

Gift of Surat Flat (WN 1)


Add

Cash gift from Relative (WN 2)

Add

Short term capital gain (WN3)

Add

Long term capital loss (WN4)

3,40,000
-----30,000
(1,22,929)
3,70,000

Less

Deduction under chapter VI-A


Total Income

Nil
3,70,000

Notes:
1.

It is now taxable as it is a gift-in-kind received on or after 1st October 2013.

2.

Gift received from unrelated person is not taxable if the amount received from each such unrelated
person is less than 50000 per annum.

3.

Computation of short term capital gains [on Sale of Flats]

Less

Sale Consideration S.50C (WEH of Stamp duty value or sale consideration)

3,70,000

Cost of acquisition [cost to Previous owner as per Sec. 49(1)(iii)]

3,40,000

Short term capital gains


4.

30,000

Computation of long term capital gains [on sale of Equity Shares of X Private ltd.]
Sale consideration
Less

Indexed cost of acquisition *


Long term capital loss

2,80,000
+

4,02,929
1,22,929

Note: STT is not applicable as shares are of a private co.

Income Tax

10.74

Shares held for not more than 12 months immediately preceding the date of transfer are short-term
capital assets as per Section 2(42A). In this case, shares have been held for more than 12 months and
therefore it constitute a LTCA. The loss arising from sale of such shares, is therefore a long-term capital
loss. As per Section 70(3), long term capital loss can be set-off only against long-term capital gains. So,
long-term capital loss cannot be set-off against short-term capital gains. But, such long-term capital loss
can be carried forward to the next year for set-off against long-term capital gains arising in that year &
they can be carried forward for a maximum of 8 assessment years.
[CA INTER M10 & N13, 6 & 4 Marks (Similar in N13 Vaibhav & Dhanush instead of Rajkumar &
Dhuruv)]
Question: Mr. Rajkumar sold a house to his friend Mr. Dhuruv on 1 st November 2015 for a consideration
of 25,00,000. The Sub-Registrar refused to register the document for the said value, as according to him,
stamp duty had to be paid on 45,00,000, which was the Government guideline value. Mr. Raj Kumar
preferred an appeal to the Revenue Divisional Officer, who fixed the value of the house as 32,00,000
(22,00,000 for land balance for building portion). The differential stamp duty was paid, accepting the
said value determined. Assuming that the fair market value is 32,00,000, what are the tax implications in
the hands of Mr. Raj Kumar and Mr. Dhuruv for the assessment year 2016-17? Mr. Raj Kumar had
purchased the land on 1st June 2012 for 5,19,000 and completed the construction of house on 1 st October,
2013 for 14,00,000.
Answer: In the hands of the seller, Mr. Raj Kumar
As per section 50C(1), where the consideration received or accruing as a result of transfer of land or
building or both, is less than value adopted or assessed or assessable by the stamp valuation authority,
the value of adopted or assessed or assessable by the stamp valuation authority shall be deemed to be the
full value of consideration received or accruing as a result of transfer.
Where the assessee appeals against the stamp valuation and the value is reduced in appeal by the
appellate authority (Revenue Divisional Officer, in this case), such value will be regarded as the
consideration received or accruing as a result of transfer.
In the given problem, land has been held for a period exceeding 36 months and building for a period less
than 36 months immediately preceding the date of transfer. So land is a long-term capital asset, while
building is a short-term capital asset.

Particulars
Long term capital gain on sale of land
Consideration received or accruing as a result of transfer of land
Less

Indexed cost of acquisition (

Long-term capital gain (A)

22,00,000
6,58,496
21,99,352

Short-term capital loss on sale of building


Consideration received or accruing from transfer of building

Capital Gains

10,00,000

10.75

Less

Cost of acquisition

14,00,000

Short term capital loss (B)

4,00,000

As per section 70, short-term capital loss can be set-off against long term capital gains. Therefore, the net
taxable long-term capital gains would be 17,99,342 (i.e.) [21,99,352 4,00,000].
In the hands of the buyer Mr. Dhuruv.
As per Section 56 (2)(vii), inserted by the Finance Act, 2009, where an individual receives from a nonrelative, any immovable property for inadequate consideration, and the difference between the stamp
value (or the value reduced by the appellate authority, as in this case) and the consideration exceeds
50,000, such difference is chargeable to tax as income from other sources. The problem states that the
immovable property in question was received on 1 st November, 2014 from a non- relative. Hence the
above provisions will be attracted. The inadequate consideration to the tune of 7 lacs (32 lacs less 25 lacs)
will be assessed as income from other sources.
[CA IPCC N15, 8 Marks]
Question: Mr. Martin sold his residential house property on 08-06-2015 for 70 lakhs which was purchased
by him for 20 lakhs on 05-05-2005. He paid 1 lakh as brokerage for the said property. The stamp duty
valuation assessed by sub registrar was 80 lakhs.
He bought another house property on 25-12-2015 for 15 lakhs.
He deposited 10 lakhs on 10-11-2015 in the capital gain bond of National Highway Authority of India
(NHAI).
He deposited another 10 lakhs on 10-07-2016 in the capital gain deposit scheme with SBI for construction
of additional floor of house property.
Compute income under the head Capital Gains for A.Y. 2016-17 as per Income Tax Act, 1961 and also
Income tax payable on the assumption that he has no other income chargeable to tax.

Income Tax

10.76

Additional CMA Exam Questions


CMA INTER SY12, D13, 5 Marks
Question: Chirag, an individual, purchased 5,000 shares of X. Limited at 50 per share and 4,000 shares of
Y. Limited at 60 per share in the previous year 2010-11 and held them as capital assets. In the previous
year 2012-13, he converted the shares into his stock-in-trade. The fair market value of the shares of both
the companies on the date of conversion was 300 per share.
In the previous year 2015-16, he sold the shares of the two companies at 380 per share. Shares were sold
by way of private sale and hence securities transactions tax was not payable. Ascertain chargeable capital
gain and business income from the above-noted transactions in the hands of Chirag.
Answer: Computation of Income for Assessment Year 2016- 2017

Capital Gain
Fair Market value of Y Ltd. on the date of conversion 9,000 x 300
Less

Indexed cost of Acquisition (2,50,000 + 2,40,000) x 852/711


Long term capital gain

27,00,000
5,87,173
21,12,827

Business Income

Less

Sale proceeds of Shares (9,000 x 380)

34,20,000

Fair market Value of Shares on the date of conversion

27,00,000
7,20,000

Business Income

Note: As securities transaction tax was not paid, exemption u/s 10(38) in respect of long term capital gain
is not available.

[CMA INTER SY08, J14, 1 Mark]


Question: The investment made in the long term specified asset by an assessee to claim exemption u/s
54EC from capital gain tax cannot exceed in lakhs1
a) 10
b) 15
c)

25

d) 50

[CMA INTER SY08, J14, 3 Marks]


Question: Mr. Sanjay running a proprietary business by name "Kannan Departmental Store" wants to
convert it into a company. State the conditions to be satisfied by Mr. Sanjay for availing exemption from
capital gains tax by treating the transaction not being regarded as "transfer".
1

Answer: (d)

Capital Gains

10.77

Answer: Mr. Sanjay must satisfy the conditions contained in section 47(xiv) in order to be eligible for
exemption in respect of conversion of proprietary concern into company. The conditions are
1.

All the assets and liabilities of the sole proprietary concern relating to the business immediately
before the succession must become the assets and liabilities of the successor company.

2.

The shareholding of the proprietor relating to the business must not be less than 50% of the total
voting power and it should continue to remain so for a period of five years from the date of
succession; and

3.

The sole proprietor (Sanjay in this case) must not receive any consideration or benefit, directly or
indirectly, in any form or manner, other than by way of allotment of shares in the company.

[CMA INTER SY08, J14, 1 Mark]


Question: Where any land is located within aerial distance of 7 kms from municipal limits, to be regarded
as capital asset u/s 2(14), the population of the municipality as per last census done before 1.3.201 should
be more than1
a) 9 lacs
b) 8 lacs
c)

10 lacs

d) None of these

[CMA INTER SY08, D14, 1 Mark]


Question: In order to avail exemption u/s 54EC of the Income-tax Act, 1961, the time limit for making
investment in specified bonds from the date of transfer of long-term capital asset is2
a) 12 months
b) 36 months
c)

60 months

d) 6 months

[CMA INTER SY08, D14, 1 Mark]


Question: When an asset owned by father for 10 years is inherited by son in June, 2015 and is
subsequently sold in December, 2015, the resultant capital gain is _________ in nature 3.

[CMA INTER SY08, D14, 1 Mark]


Question: When a factory building owned by an individual used for business purposes is sold after 48
months from the date of purchase, no other building being owned by the assessee, the resultant capital
gain is _________ in nature1.
1

Answer: (c)
Answer: (d)
3
Answer: Long term
2

Income Tax

10.78

[CMA INTER S12 & SY08, J14, 9 Marks]


Question: Mr. Ahuja purchased one plot of land in 1996-97 at a cost of 1,00,000 in Delhi. The land was
held by him as capital asset. He converted the plot into his stock in trade on 1 st April, 2013, on which date
the fair market value of the plot was 15,50,000.
He started constructing a building consisting of eight flats of equal size and dimension on the plot on 1 st
April, 2013. Cost of construction of each flat is 6,00,000. Construction was completed in June 2014. He
sold five flats at 12,00,000 per flat from June, 2015 to March, 2016. The remaining three flats were held as
stock on 31st March, 2016.
Compute Capital Gain and Business Income arising from above transactions for Assessment year 2016-17.
Answer: Computation of Capital Gain for Assessment Year 2016-17

Particulars
Proportionate Fair market value of land-deemed consideration u/s 45(2): 15,50,000 x 5 /
8 (Note 1)
Less

9,68,750

Proportionate indexed cost of acquisition (

) (Note 2)

1,92,418

Long term capital gain

7,76,332

Computation of Business Income for Assessment Year 2016-17

Particulars

Sale proceeds of 5 flats : 12,00,000 x 5


Less

Proportionate fair market value of land


Cost of construction (6,00,000 x 5)
Business Income

60,00,000

9,68,750
30,00,000

39,68,750
20,31,250

Notes:
1. Capital gain arising on conversion of capital asset into stock-in-trade is taxed in the year in which the
converted asset is transferred. Therefore, capital gain in respect of proportionate land for 5 flats is
chargeable to tax in assessment year 2016-17.
2.Although tax liability for capital gain is taxed in the year of actual transfer, cost inflation index of the
year in which capital asset is converted is to be used for determining capital gain, as transfer u/s 2(47) is
recognized in the year of conversion.

[CMA INTER SY08, J14, 6 Marks]


Question: Dev is engaged in the business of real estate. During the previous year 2014-15, he sold a flat
forming part of his stock-in-trade to Jeet for 45 lakhs. The value assessed by the Stamp Valuation
Authority for stamp duty purpose is 55 lakhs. Jeet holds the flat as capital asset.

Answer: Short-term

Capital Gains

10.79

State with reasons the implications of the above transaction on taxable income of Dev and Jeet for the
Assessment Year 2015-16.
Answer: Implication in the hands of Dev: As the flat sold by Dev formed part of his stock-in-trade, the
provision of section 43CA shall apply.
As per section 43CA where consideration as a result of transfer by an assessee of any immovable asset
(other than a capital asset) is less than the stamp duty value, then the stamp duty value shall be deemed
to be the full value of the consideration for the purpose of computing business profit from transfer of
such asset.
Therefore, the difference between the stamp duty value and the actual consideration i.e. 10 lacs (55 lacs
45 lacs) shall be charged to tax under the head profits and gains from business or profession.
Implication in the hand of Jeet: As per section 56(2)(vii) if any individual receive any immovable
property (being capital asset) for a consideration which is less than its stamp duty value by more than
5000, then the excess of stamp duty value over the actual consideration shall be taxed under the head
income from other sources
Therefore, the amount of 10 lacs, being the excess of stamp duty value over the actual consideration
shall be taxed in the hands of Jeet under the head income from other sources.

[CMA INTER SY12, J14, 1 Mark]


Question: Choose the best option: To avail exemption u/s 54, an individual should purchase a new
residential house within ________ years from the date of sale.1
(A) 2 (B) 3 (C) 1 (D) 4
[CMA INTER SY12 & SY08, J14, 5 Marks]
Question: State the provisions relating to claiming of exemption in order to reduce tax liability on shortterm capital gains.
Answer: Provisions relating to claiming of exemption in order to reduce tax liability on short term capital
gains.
Applicable for

Exemption U/S

Transfer of Agricultural land

54B

Transfer by way of compulsory acquisition by Government

54D

Shifting of industrial undertaking from urban to Rural area

54G

Compulsory acquisition of Agricultural land by Central Government / RBI

10(37)

Transfer by companies engaged in power sector business

10(41)

Income Tax

10.80

[CMA INTER SY12, D14, 1 Mark]


Question: Fill up the blanks: Buy back of unlisted shares by a company is _______ even if it is incurred
in the course of shareholder1.

[CMA INTER SY08, D14, 1 Mark]


Question: Choose the best option: Basic rate of tax applicable to domestic company on distributed
income in the case of buyback of shares not listed in stock exchange is 2
(A) 15% on the distributed income (B) 20% on the distributed income (C) 30% on the distributed income
(D) Nil

[CMA INTER SY08, D14, 4 Marks]


Question: State with brief reason, whether the following assets are long-term or short-term capital assets
(Mere conclusion will not receive any credit):
1.

Renunciations of right to subscribe to right share, immediately after the offer was made by the
company.

2.

Sale of surgery equipments by a doctor on discontinuance of profession.

3.

Slump sale of undertaking offer operating for 4 years.

4.

Sale of bonus shares within 6 months after the allotment; original shares were acquired 24 menthes e
earlier.

Answer:
1.

As per clause (e) of the Explanation 1 to section 2(42A) right to subscribe to any financial asset when
renounced in favour of any other person, the period shall be reckoned from the date of offer of such
right by the company. Since the time between the offer made by the company and its renunciation in
no case could exceed 12 months, it is always a short-term capital asset.

2.

Surgery equipment used by a surgeon in profession might have been allowed depreciation under
section 32. Hence, in view of section 50 it shall deemed to be a short-term capital asset and the gain or
loss if any sale be short-term capital gain or loss.

3.

As per proviso to section 50B(1) an undertaking owned and held for not more than 36 months shall
be short-term capital asset and when the undertaking was operated for 4 years before slump sale, it
shall be a long-term capital asset.

4.

As per clause (f) of the Explanation 1 to section 2(42A), a financial asset allotted without payment
shall be treated as long-term or short-term by reckoning the time period from the date of allotment of
such financial asset. Thus bonus shares sold within 6 months after allotment will be short-term
capital asset.

1
2

Exempt
(B) 20% on the distributed income

Capital Gains

10.81

[CMA INTER SY12, J15, 1 Marks]


Question: Fill up the blanks: A Zero coupon bond is a long-term capital asset, if it is held for more than
______ months before transfer1.

[CMA INTER SY12, J15, 1 Marks]


Question: Choose the best option: Long-term capital gain on off-market sale of shares of a listed
company without availing of indexation benefit is taxed at 2
(A) 5% (B) 10% (C) 15% (D) 20%

[CMA INTER SY12, J15, 3 Marks]


Question: Can Mr. Ajit who has long-term capital gain from sale of vacant site in India buy a residential
house outside India to claim exemption under section 54F? Assume that he has no residential property in
India.
Answer: Exemption u/s 54F
When the assessee has no residential house property or has not more than one residential house
property, he is eligible to claim exemption under section 54F.
When a long-term capital asset other than a residential house is transferred, the exemption by way of
investment in residential house could be obtained by deploying the net consideration.
The exemption is subject to the condition that the assessee has within a period of one year before or
two years after the date of transfer of long-term capital asset acquires a residential house or three
years after transfer constructs a new residential house in India for the purpose of availing the
exemption under section 54F.
A person transferring a long-term capital asset in India cannot acquire a residential house outside
India and be eligible for exemption under section 54F.
Thus the person cannot claim exemption in respect of long-term capital gain on sale of vacant site in
India by acquiring a residential house outside India.

[CMA INTER SY12, J15, 5 Marks]


Question: Mr. Dhoni sold a residential building at Cochin for 65 lakhs in December 2015. The stamp
valuation authority determined the value at 80 lakhs which was not contested by Mr. Dhoni. The
property was acquired in April, 2005 for 6 lakhs. He acquired a residential flat at Ranchi for 55 lakhs
and another residential house at Cuttack for 25 lakhs before March, 2016. Compute the capital gain of
Mr. Dhoni for the assessment year 2016-17.

1
2

12 months
(B) 10%

Income Tax

10.82

Answer: You are required to plan in such a way that the incidence of tax is the least.
Particulars

Less

Less

Sale consideration of residential building of Cochin

65,00,000

Stamp valuation authority has determined the value of 80 lakhs

80,00,000

The higher of the two is to be adopted as deemed sale consideration u/s 50C

80,00,000

Indexed cost of acquisition

13,05,030

Capital gains before exemption

66,94,970

Exemption u/s 54
In respect of residential property acquired of Ranchi

55,00,000

Taxable long-term capital gain

11,94,970

Note: From the assessment year 2016-17 onwards, the exemption u/s. 54 is limited to one residential
house in India. Out of the two residential properties acquired by him, he can opt for exemption u/s. 54 in
respect of one property only. The higher of the value has been adopted for maximizing the benefit to the
assessee.

Capital Gains

10.83

Bare Act E.Capital gains


Capital gains.
45. (1) Any profits or gains arising from the transfer of a capital asset effected in the previous year shall, save as
otherwise provided in sections 54, 54B, 54D, 54E, 54EA, 54EB, 54F, 54G and 54H, be chargeable to income-tax
under the head "Capital gains", and shall be deemed to be the income of the previous year in which the transfer took
place.
(1A) Notwithstanding anything contained in sub-section (1), where any person receives at any time during any
previous year any money or other assets under an insurance from an insurer on account of damage to, or destruction
of, any capital asset, as a result of
(i) flood, typhoon, hurricane, cyclone, earthquake or other convulsion of nature; or
(ii) riot or civil disturbance; or
(iii) accidental fire or explosion; or
(iv) action by an enemy or action taken in combating an enemy (whether with or without a declaration of war),
then, any profits or gains arising from receipt of such money or other assets shall be chargeable to income-tax under
the head "Capital gains" and shall be deemed to be the income of such person of the previous year in which such
money or other asset was received and for the purposes of section 48, value of any money or the fair market value of
other assets on the date of such receipt shall be deemed to be the full value of the consideration received or accruing
as a result of the transfer of such capital asset.
Explanation.For the purposes of this sub-section, the expression "insurer" shall have the meaning assigned to it in
clause (9) of section 2 of the Insurance Act, 1938 (4 of 1938).
(2) Notwithstanding anything contained in sub-section (1), the profits or gains arising from the transfer by way of
conversion by the owner of a capital asset into, or its treatment by him as stock-in-trade of a business carried on by
him shall be chargeable to income-tax as his income of the previous year in which such stock-in-trade is sold or
otherwise transferred by him and, for the purposes of section 48, the fair market value of the asset on the date of
such conversion or treatment shall be deemed to be the full value of the consideration received or accruing as a
result of the transfer of the capital asset.
(2A) Where any person has had at any time during previous year any beneficial interest in any securities, then, any
profits or gains arising from transfer made by the depository or participant of such beneficial interest in respect of
securities shall be chargeable to income-tax as the income of the beneficial owner of the previous year in which such
transfer took place and shall not be regarded as income of the depository who is deemed to be the registered owner
of securities by virtue of sub-section (1) of section 10 of the Depositories Act, 1996, and for the purposes of
(i) section 48; and
(ii) proviso to clause (42A) of section 2,
the cost of acquisition and the period of holding of any securities shall be determined on the basis of the first-infirst-out method.
Explanation.For the purposes of this sub-section, the expressions "beneficial owner", "depository" and "security"
shall have the meanings respectively assigned to them in clauses (a), (e) and (l) of sub-section (1) of section 2 of the
Depositories Act, 1996.
(3) The profits or gains arising from the transfer of a capital asset by a person to a firm or other association of
persons or body of individuals (not being a company or a co-operative society) in which he is or becomes a partner
or member, by way of capital contribution or otherwise, shall be chargeable to tax as his income of the previous year
in which such transfer takes place and, for the purposes of section 48, the amount recorded in the books of account
of the firm, association or body as the value of the capital asset shall be deemed to be the full value of the
consideration received or accruing as a result of the transfer of the capital asset.

Income Tax

10.84

(4) The profits or gains arising from the transfer of a capital asset by way of distribution of capital assets on the
dissolution of a firm or other association of persons or body of individuals (not being a company or a co-operative
society) or otherwise, shall be chargeable to tax as the income of the firm, association or body, of the previous year
in which the said transfer takes place and, for the purposes of section 48, the fair market value of the asset on the
date of such transfer shall be deemed to be the full value of the consideration received or accruing as a result of the
transfer.
(5) Notwithstanding anything contained in sub-section (1), where the capital gain arises from the transfer of a capital
asset, being a transfer by way of compulsory acquisition under any law, or a transfer the consideration for which was
determined or approved by the Central Government or the Reserve Bank of India, and the compensation or the
consideration for such transfer is enhanced or further enhanced by any court, Tribunal or other authority, the capital
gain shall be dealt with in the following manner, namely :
(a) the capital gain computed with reference to the compensation awarded in the first instance or, as the case
may be, the consideration determined or approved in the first instance by the Central Government or the
Reserve Bank of India shall be chargeable as income under the head "Capital gains" of the previous year
in which such compensation or part thereof, or such consideration or part thereof, was first received; and
(b) the amount by which the compensation or consideration is enhanced or further enhanced by the court,
Tribunal or other authority shall be deemed to be income chargeable under the head "Capital gains" of the
previous year in which such amount is received by the assessee :
91

[Provided that any amount of compensation received in pursuance of an interim order of a court,
Tribunal or other authority shall be deemed to be income chargeable under the head "Capital gains" of
the previous year in which the final order of such court, Tribunal or other authority is made;]
(c) where in the assessment for any year, the capital gain arising from the transfer of a capital asset is computed
by taking the compensation or consideration referred to in clause (a) or, as the case may be, enhanced
compensation or consideration referred to in clause (b), and subsequently such compensation or
consideration is reduced by any court, Tribunal or other authority, such assessed capital gain of that year
shall be recomputed by taking the compensation or consideration as so reduced by such court, Tribunal or
other authority to be the full value of the consideration.
Explanation.For the purposes of this sub-section,
(i) in relation to the amount referred to in clause (b), the cost of acquisition and the cost of improvement shall
be taken to be nil;
(ii) the provisions of this sub-section shall apply also in a case where the transfer took place prior to the 1st day
of April, 1988;
(iii) where by reason of the death of the person who made the transfer, or for any other reason, the enhanced
compensation or consideration is received by any other person, the amount referred to in clause (b) shall
be deemed to be the income, chargeable to tax under the head "Capital gains", of such other person.
(6) Notwithstanding anything contained in sub-section (1), the difference between the repurchase price of the units
referred to in sub-section (2) of section 80CCB and the capital value of such units shall be deemed to be the capital
gains arising to the assessee in the previous year in which such repurchase takes place or the plan referred to in that
section is terminated and shall be taxed accordingly.
Explanation.For the purposes of this sub-section, "capital value of such units" means any amount invested by the
assessee in the units referred to in sub-section (2) of section 80CCB.
Capital gains on distribution of assets by companies in liquidation.

Capital Gains

10.85

46. (1) Notwithstanding anything contained in section 45, where the assets of a company are distributed to its
shareholders on its liquidation, such distribution shall not be regarded as a transfer by the company for the purposes
of section 45.
(2) Where a shareholder on the liquidation of a company receives any money or other assets from the company, he
shall be chargeable to income-tax under the head "Capital gains", in respect of the money so received or the market
value of the other assets on the date of distribution, as reduced by the amount assessed as dividend within the
meaning of sub-clause (c) of clause (22) of section 2 and the sum so arrived at shall be deemed to be the full value
of the consideration for the purposes of section 48.
Capital gains on purchase by company of its own shares or other specified securities.
46A. Where a shareholder or a holder of other specified securities receives any consideration from any company for
purchase of its own shares or other specified securities held by such shareholder or holder of other specified
securities, then, subject to the provisions of section 48, the difference between the cost of acquisition and the value
of consideration received by the shareholder or the holder of other specified securities, as the case may be, shall be
deemed to be the capital gains arising to such shareholder or the holder of other specified securities, as the case may
be, in the year in which such shares or other specified securities were purchased by the company.
Explanation.For the purposes of this section, "specified securities" shall have the meaning assigned to it
in Explanation to section 77A of the Companies Act, 1956 (1 of 1956).
Transactions not regarded as transfer.
47. Nothing contained in section 45 shall apply to the following transfers :
(i) any distribution of capital assets on the total or partial partition of a Hindu undivided family;
(ii) [***]
(iii) any transfer of a capital asset under a gift or will or an irrevocable trust :
Provided that this clause shall not apply to transfer under a gift or an irrevocable trust of a capital asset
being shares, debentures or warrants allotted by a company directly or indirectly to its employees under
any Employees' Stock Option Plan or Scheme of the company offered to such employees in accordance
with the guidelines issued by the Central Government in this behalf;
(iv) any transfer of a capital asset by a company to its subsidiary company, if
(a) the parent company or its nominees hold the whole of the share capital of the subsidiary company, and
(b) the subsidiary company is an Indian company;
(v) any transfer of a capital asset by a subsidiary company to the holding company, if
(a) the whole of the share capital of the subsidiary company is held by the holding company, and
(b) the holding company is an Indian company :
Provided that nothing contained in clause (iv) or clause (v) shall apply to the transfer of a capital asset
made after the 29th day of February, 1988, as stock-in-trade;
(vi) any transfer, in a scheme of amalgamation, of a capital asset by the amalgamating company to the
amalgamated company if the amalgamated company is an Indian company;
(via) any transfer, in a scheme of amalgamation, of a capital asset being a share or shares held in an Indian
company, by the amalgamating foreign company to the amalgamated foreign company, if
(a) at least twenty-five per cent of the shareholders of the amalgamating foreign company continue to
remain shareholders of the amalgamated foreign company, and

Income Tax

10.86

(b) such transfer does not attract tax on capital gains in the country, in which the amalgamating company
is incorporated;
(viaa) any transfer, in a scheme of amalgamation of a banking company with a banking institution sanctioned and
brought into force by the Central Government under sub-section (7) of section 45 of the Banking
Regulation Act, 1949 (10 of 1949), of a capital asset by the banking company to the banking institution.
Explanation.For the purposes of this clause,
(i) "banking company" shall have the same meaning assigned to it in clause (c) of section 5 of the Banking
Regulation Act, 1949 (10 of 1949);
(ii) "banking institution" shall have the same meaning assigned to it in sub-section (15) of section 45 of the
Banking Regulation Act, 1949 (10 of 1949);]
Following clause (viab) shall be inserted after clause (viaa) of section 47 by the Finance Act, 2015,
w.e.f. 1-4-2016 :
(viab) any transfer, in a scheme of amalgamation, of a capital asset, being a share of a foreign company, referred
to in the Explanation 5 to clause (i) of sub-section (1) of section 9, which derives, directly or indirectly, its
value substantially from the share or shares of an Indian company, held by the amalgamating foreign
company to the amalgamated foreign company, if
(A) at least twenty-five per cent of the shareholders of the amalgamating foreign company continue to
remain shareholders of the amalgamated foreign company; and
(B) such transfer does not attract tax on capital gains in the country in which the amalgamating company
is incorporated;
(vib) any transfer, in a demerger, of a capital asset by the demerged company to the resulting company, if the
resulting company is an Indian company;
(vic) any transfer in a demerger, of a capital asset, being a share or shares held in an Indian company, by the
demerged foreign company to the resulting foreign company, if
(a) the shareholders holding not less than three-fourths in value of the shares of the demerged foreign
company continue to remain shareholders of the resulting foreign company; and
(b) such transfer does not attract tax on capital gains in the country, in which the demerged foreign
company is incorporated :
Provided that the provisions of sections 391 to 394 of the Companies Act, 1956 (1 of 1956) shall not
apply in case of demergers referred to in this clause;
(vica) any transfer in a business reorganisation, of a capital asset by the predecessor co-operative bank to the
successor co-operative bank;
(vicb) any transfer by a shareholder, in a business reorganisation, of a capital asset being a share or shares held by
him in the predecessor co-operative bank if the transfer is made in consideration of the allotment to him of
any share or shares in the successor co-operative bank.
Explanation.For the purposes of clauses (vica) and (vicb), the expressions "business reorganisation",
"predecessor co-operative bank" and "successor co-operative bank" shall have the meanings respectively
assigned to them in section 44DB;
Following clause (vicc) shall be inserted after clause (vicb) of section 47 by the Finance Act, 2015,
w.e.f. 1-4-2016 :
(vicc) any transfer in a demerger, of a capital asset, being a share of a foreign company, referred to in
the Explanation 5 to clause (i) of sub-section (1) of section 9, which derives, directly or indirectly, its
value substantially from the share or shares of an Indian company, held by the demerged foreign company
to the resulting foreign company, if

Capital Gains

10.87

(a) the shareholders, holding not less than three-fourths in value of the shares of the demerged foreign
company, continue to remain shareholders of the resulting foreign company; and
(b) such transfer does not attract tax on capital gains in the country in which the demerged foreign
company is incorporated:
Provided that the provisions of sections 391 to 394 of the Companies Act, 1956 (1 of 1956) shall not
apply in case of demergers referred to in this clause;
(vid) any transfer or issue of shares by the resulting company, in a scheme of demerger to the shareholders of the
demerged company if the transfer or issue is made in consideration of demerger of the undertaking;
(vii) any transfer by a shareholder, in a scheme of amalgamation, of a capital asset being a share or shares held
by him in the amalgamating company, if
(a) the transfer is made in consideration of the allotment to him of any share or shares in the amalgamated
company except where the shareholder itself is the amalgamated company, and
(b) the amalgamated company is an Indian company;
(viia) any transfer of a capital asset, being bonds or Global Depository Receipts referred to in sub-section (1)
of section 115AC, made outside India by a non-resident to another non-resident;
92

[(viib) any transfer of a capital asset, being a Government Security carrying a periodic payment of interest, made
outside India through an intermediary dealing in settlement of securities, by a non-resident to another
non-resident.
Explanation.For the purposes of this clause, "Government Security" shall have the meaning assigned to
it in clause (b) of section 2 of the Securities Contracts (Regulation) Act, 1956 (42 of 1956);]
(viii) any transfer of agricultural land in India effected before the 1st day of March, 1970;
(ix) any transfer of a capital asset, being any work of art, archaeological, scientific or art collection, book,
manuscript, drawing, painting, photograph or print, to the Government or a University or the National
Museum, National Art Gallery, National Archives or any such other public museum or institution as may
be notified by the Central Government in the Official Gazette to be of national importance or to be of
renown throughout any State or States.
Explanation.For the purposes of this clause, "University" means a University established or
incorporated by or under a Central, State or Provincial Act and includes an institution declared under
section 3 of the University Grants Commission Act, 1956 (3 of 1956), to be a University for the purposes
of that Act;
(x) any transfer by way of conversion of bonds or debentures, debenture-stock or deposit certificates in any
form, of a company into shares or debentures of that company;
(xa) any transfer by way of conversion of bonds referred to in clause (a) of sub-section (1) of section
115AC into shares or debentures of any company;
(xi) any transfer made on or before the 31st day of December, 1998 by a person (not being a company) of a
capital asset being membership of a recognised stock exchange to a company in exchange of shares
allotted by that company to the transferor.
Explanation.For the purposes of this clause, the expression "membership of a recognised stock
exchange" means the membership of a stock exchange in India which is recognised under the provisions
of the Securities Contracts (Regulation) Act, 1956 (42 of 1956);
(xii) any transfer of a capital asset, being land of a sick industrial company, made under a scheme prepared and
sanctioned under section 18 of the Sick Industrial Companies (Special Provisions) Act, 1985 (1 of 1986)
where such sick industrial company is being managed by its workers' co-operative :
Provided that such transfer is made during the period commencing from the previous year in which the
said company has become a sick industrial company under sub-section (1) of section 17 of that Act and

Income Tax

10.88

ending with the previous year during which the entire net worth of such company becomes equal to or
exceeds the accumulated losses.
Explanation.For the purposes of this clause, "net worth" shall have the meaning assigned to it in clause
(ga) of sub-section (1) of section 3 of the Sick Industrial Companies (Special Provisions) Act, 1985 (1 of
1986);
(xiii) any transfer of a capital asset or intangible asset by a firm to a company as a result of succession of the
firm by a company in the business carried on by the firm, or any transfer of a capital asset to a company in
the course of demutualisation or corporatisation of a recognised stock exchange in India as a result of
which an association of persons or body of individuals is succeeded by such company :
Provided that
(a) all the assets and liabilities of the firm or of the association of persons or body of individuals relating
to the business immediately before the succession become the assets and liabilities of the company;
(b) all the partners of the firm immediately before the succession become the shareholders of the company
in the same proportion in which their capital accounts stood in the books of the firm on the date of
the succession;
(c) the partners of the firm do not receive any consideration or benefit, directly or indirectly, in any form
or manner, other than by way of allotment of shares in the company; and
(d) the aggregate of the shareholding in the company of the partners of the firm is not less than fifty per
cent of the total voting power in the company and their shareholding continues to be as such for a
period of five years from the date of the succession;
(e) the demutualisation or corporatisation of a recognised stock exchange in India is carried out in
accordance with a scheme for demutualisation or corporatisation which is approved by the Securities
and Exchange Board of India established under section 3 of the Securities and Exchange Board of
India Act, 1992 (15 of 1992);
(xiiia) any transfer of a capital asset being a membership right held by a member of a recognised stock exchange
in India for acquisition of shares and trading or clearing rights acquired by such member in that recognised
stock exchange in accordance with a scheme for demutualisation or corporatisation which is approved by
the Securities and Exchange Board of India established under section 3 of the Securities and Exchange
Board of India Act, 1992 (15 of 1992);
(xiiib) any transfer of a capital asset or intangible asset by a private company or unlisted public company
(hereafter in this clause referred to as the company) to a limited liability partnership or any transfer of a
share or shares held in the company by a shareholder as a result of conversion of the company into a
limited liability partnership in accordance with the provisions of section 56 or section 57 of the Limited
Liability Partnership Act, 2008 (6 of 2009):
Provided that
(a) all the assets and liabilities of the company immediately before the conversion become the assets and
liabilities of the limited liability partnership;
(b) all the shareholders of the company immediately before the conversion become the partners of the
limited liability partnership and their capital contribution and profit sharing ratio in the limited
liability partnership are in the same proportion as their shareholding in the company on the date of
conversion;
(c) the shareholders of the company do not receive any consideration or benefit, directly or indirectly, in
any form or manner, other than by way of share in profit and capital contribution in the limited
liability partnership;

Capital Gains

10.89

(d) the aggregate of the profit sharing ratio of the shareholders of the company in the limited liability
partnership shall not be less than fifty per cent at any time during the period of five years from the
date of conversion;
(e) the total sales, turnover or gross receipts in the business of the company in any of the three previous
years preceding the previous year in which the conversion takes place does not exceed sixty lakh
rupees; and
(f) no amount is paid, either directly or indirectly, to any partner out of balance of accumulated profit
standing in the accounts of the company on the date of conversion for a period of three years from
the date of conversion.
Explanation.For the purposes of this clause, the expressions "private company" and "unlisted public
company" shall have the meanings respectively assigned to them in the Limited Liability Partnership Act,
2008 (6 of 2009);
(xiv) where a sole proprietary concern is succeeded by a company in the business carried on by it as a result of
which the sole proprietary concern sells or otherwise transfers any capital asset or intangible asset to the
company :
Provided that
(a) all the assets and liabilities of the sole proprietary concern relating to the business immediately before
the succession become the assets and liabilities of the company;
(b) the shareholding of the sole proprietor in the company is not less than fifty per cent of the total voting
power in the company and his shareholding continues to remain as such for a period of five years
from the date of the succession; and
(c) the sole proprietor does not receive any consideration or benefit, directly or indirectly, in any form or
manner, other than by way of allotment of shares in the company;
(xv) any transfer in a scheme for lending of any securities under an agreement or arrangement, which the
assessee has entered into with the borrower of such securities and which is subject to the guidelines issued
by the Securities and Exchange Board of India, established under section 3 of the Securities and Exchange
Board of India Act, 1992 (15 of 1992) or the Reserve Bank of India constituted under sub-section (1) of
section 3 of the Reserve Bank of India Act, 1934 (2 of 1934), in this regard;
(xvi) any transfer of a capital asset in a transaction of reverse mortgage under a scheme made and notified by the
Central Government;
93

[(xvii) any transfer of a capital asset, being share of a special purpose vehicle to a business trust in exchange of
units allotted by that trust to the transferor.
Explanation.For the purposes of this clause, the expression "special purpose vehicle" shall have the
meaning assigned to it in theExplanation to clause (23FC) of section 10.]
Following clause (xviii) shall be inserted after clause (xvii) of section 47 by the Finance Act, 2015,
w.e.f. 1-4-2016 :

(xviii) any transfer by a unit holder of a capital asset, being a unit or units, held by him in the consolidating
scheme of a mutual fund, made in consideration of the allotment to him of a capital asset, being a unit or
units, in the consolidated scheme of the mutual fund:
Provided that the consolidation is of two or more schemes of equity oriented fund or of two or more
schemes of a fund other than equity oriented fund.
Explanation. For the purposes of this clause,
(a) "consolidated scheme" means the scheme with which the consolidating scheme merges or which is
formed as a result of such merger;

Income Tax

10.90

(b) "consolidating scheme" means the scheme of a mutual fund which merges under the process of
consolidation of the schemes of mutual fund in accordance with the Securities and Exchange Board
of India (Mutual Funds) Regulations, 1996 made under the Securities and Exchange Board of India
Act, 1992 (15 of 1992);
(c) "equity oriented fund" shall have the meaning assigned to it in clause (38)of section 10;
(d) "mutual fund" means a mutual fund specified under clause (23D)of section 10.
Withdrawal of exemption in certain cases.
47A. (1) Where at any time before the expiry of a period of eight years from the date of the transfer of a capital asset
referred to in clause (iv) or, as the case may be, clause (v) of section 47,
(i) such capital asset is converted by the transferee company into, or is treated by it as, stock-in-trade of its
business; or
(ii) the parent company or its nominees or, as the case may be, the holding company ceases or cease to hold the
whole of the share capital of the subsidiary company,
the amount of profits or gains arising from the transfer of such capital asset not charged under section 45 by virtue of
the provisions contained in clause (iv) or, as the case may be, clause (v) of section 47 shall, notwithstanding
anything contained in the said clauses, be deemed to be income chargeable under the head "Capital gains" of the
previous year in which such transfer took place.
(2) Where at any time, before the expiry of a period of three years from the date of the transfer of a capital asset
referred to in clause (xi) ofsection 47, any of the shares allotted to the transferor in exchange of a membership in a
recognised stock exchange are transferred, the amount of profits and gains not charged under section 45 by virtue of
the provisions contained in clause (xi) of section 47 shall, notwithstanding anything contained in the said clause, be
deemed to be the income chargeable under the head "Capital gains" of the previous year in which such shares are
transferred.
(3) Where any of the conditions laid down in the proviso to clause (xiii) or the proviso to clause (xiv) of section
47 are not complied with, the amount of profits or gains arising from the transfer of such capital asset or intangible
asset not charged under section 45 by virtue of conditions laid down in the proviso to clause (xiii) or the proviso to
clause (xiv) of section 47 shall be deemed to be the profits and gains chargeable to tax of the successor company for
the previous year in which the requirements of the proviso to clause (xiii) or the proviso to clause (xiv), as the case
may be, are not complied with.
(4) Where any of the conditions laid down in the proviso to clause (xiiib) of section 47 are not complied with, the
amount of profits or gains arising from the transfer of such capital asset or intangible assets or share or shares not
charged under section 45 by virtue of conditions laid down in the said proviso shall be deemed to be the profits and
gains chargeable to tax of the successor limited liability partnership or the shareholder of the predecessor company,
as the case may be, for the previous year in which the requirements of the said proviso are not complied with.
Mode of computation.
48. The income chargeable under the head "Capital gains" shall be computed, by deducting from the full value of the
consideration received or accruing as a result of the transfer of the capital asset the following amounts, namely :
(i) expenditure incurred wholly and exclusively in connection with such transfer;
(ii) the cost of acquisition of the asset and the cost of any improvement thereto:
94

Provided that in the case of an assessee, who is a non-resident, capital gains arising from the transfer of a capital
asset being shares in, or debentures of, an Indian company shall be computed by converting the cost of acquisition,
expenditure incurred wholly and exclusively in connection with such transfer and the full value of the consideration
received or accruing as a result of the transfer of the capital asset into the same foreign currency as was initially

Capital Gains

10.91

utilised in the purchase of the shares or debentures, and the capital gains so computed in such foreign currency shall
be reconverted into Indian currency, so, however, that the aforesaid manner of computation of capital gains shall be
applicable in respect of capital gains accruing or arising from every reinvestment thereafter in, and sale of, shares in,
or debentures of, an Indian company :
Provided further that where long-term capital gain arises from the transfer of a long-term capital asset, other than
capital gain arising to a non-resident from the transfer of shares in, or debentures of, an Indian company referred to
in the first proviso, the provisions of clause (ii) shall have effect as if for the words "cost of acquisition" and "cost of
any improvement", the words "indexed cost of acquisition" and "indexed cost of any improvement" had respectively
been substituted:
Provided also that nothing contained in the second proviso shall apply to the long-term capital gain arising from the
transfer of a long-term capital asset being bond or debenture other than capital indexed bonds issued by the
Government :
Provided also that where shares, debentures or warrants referred to in the proviso to clause (iii) of section 47 are
transferred under a gift or an irrevocable trust, the market value on the date of such transfer shall be deemed to be
the full value of consideration received or accruing as a result of transfer for the purposes of this section :
Provided also that no deduction shall be allowed in computing the income chargeable under the head "Capital
gains" in respect of any sum paid on account of securities transaction tax under Chapter VII of the Finance (No. 2)
Act, 2004.
Explanation.For the purposes of this section,
(i) "foreign currency" and "Indian currency" shall have the meanings respectively assigned to them in section 2
of 95[the Foreign Exchange Management Act, 1999 (42 of 1999)];
(ii) the conversion of Indian currency into foreign currency and the reconversion of foreign currency into
Indian currency shall be at the rate of exchange prescribed in this behalf;
(iii) "indexed cost of acquisition" means an amount which bears to the cost of acquisition the same proportion
as Cost Inflation Index for the year in which the asset is transferred bears to the Cost Inflation Index for
the first year in which the asset was held by the assessee or for the year beginning on the 1st day of April,
1981, whichever is later;
(iv) "indexed cost of any improvement" means an amount which bears to the cost of improvement the same
proportion as Cost Inflation Index for the year in which the asset is transferred bears to the Cost Inflation
Index for the year in which the improvement to the asset took place;
(v) "Cost Inflation Index", in relation to a previous year, means such Index as the Central Government may,
having regard to seventy-five per cent of average rise in the 96[Consumer Price Index for urban nonmanual employees] for the immediately preceding previous year to such previous year, by notification 97 in
the Official Gazette, specify, in this behalf.
Cost with reference to certain modes of acquisition.
49. (1) Where the capital asset became the property of the assessee
(i) on any distribution of assets on the total or partial partition of a Hindu undivided family;
(ii) under a gift or will;
(iii) (a) by succession, inheritance or devolution, or
(b) on any distribution of assets on the dissolution of a firm, body of individuals, or other association of
persons, where such dissolution had taken place at any time before the 1st day of April, 1987, or
(c) on any distribution of assets on the liquidation of a company, or
(d) under a transfer to a revocable or an irrevocable trust, or

Income Tax

10.92

(e) under any such transfer as is referred to in clause (iv) or clause (v) or clause (vi) or clause (via) 98[or
clause (viaa) or clause (vica) or clause (vicb)] or clause (xiii) or clause (xiiib) or clause (xiv)
of section 47;
(iv) such assessee being a Hindu undivided family, by the mode referred to in sub-section (2) of section 64 at
any time after the 31st day of December, 1969,
the cost of acquisition of the asset shall be deemed to be the cost for which the previous owner of the property
acquired it, as increased by the cost of any improvement of the assets incurred or borne by the previous owner or the
assessee, as the case may be.
Explanation.In this sub-section the expression "previous owner of the property" in relation to any capital asset
owned by an assessee means the last previous owner of the capital asset who acquired it by a mode of acquisition
other than that referred to in clause (i) or clause (ii) or clause (iii) or clause (iv) of this sub-section.
(2) Where the capital asset being a share or shares in an amalgamated company which is an Indian company became
the property of the assessee in consideration of a transfer referred to in clause (vii) of section 47, the cost of
acquisition of the asset shall be deemed to be the cost of acquisition to him of the share or shares in the
amalgamating company.
(2A) Where the capital asset, being a share or debenture of a company, became the property of the assessee in
consideration of a transfer referred to in clause (x) or clause (xa) of section 47, the cost of acquisition of the asset to
the assessee shall be deemed to be that part of the cost of debenture, debenture-stock, bond or deposit certificate in
relation to which such asset is acquired by the assessee.
(2AA) Where the capital gain arises from the transfer of specified security or sweat equity shares referred to in subclause (vi) of clause (2) ofsection 17, the cost of acquisition of such security or shares shall be the fair market value
which has been taken into account for the purposes of the said sub-clause.
(2AAA) Where the capital asset, being rights of a partner referred to in section 42 of the Limited Liability
Partnership Act, 2008 (6 of 2009), became the property of the assessee on conversion as referred to in clause (xiiib)
of section 47, the cost of acquisition of the asset shall be deemed to be the cost of acquisition to him of the share or
shares in the company immediately before its conversion.
(2AB) Where the capital gain arises from the transfer of specified security or sweat equity shares, the cost of
acquisition of such security or shares shall be the fair market value which has been taken into account while
computing the value of fringe benefits under clause (ba) of sub-section (1) of section 115WC.
(2ABB) Where the capital asset, being share or shares of a company, is acquired by a non-resident assessee on
redemption of Global Depository Receipts referred to in clause (b) of sub-section (1) of section 115AC held by such
assessee, the cost of acquisition of the share or shares shall be the price of such share or shares prevailing on any
recognised stock exchange on the date on which a request for such redemption was made.
Explanation.For the purposes of this sub-section, "recognised stock exchange" shall have the meaning assigned to
it in clause (ii) of theExplanation 1 to sub-section (5) of section 43.
99

[(2AC) Where the capital asset, being a unit of a business trust, became the property of the assessee in
consideration of a transfer as referred to in clause (xvii) of section 47, the cost of acquisition of the asset shall be
deemed to be the cost of acquisition to him of the share referred to in the said clause.]
Following sub-section (2AD) shall be inserted after sub-section (2AC) of section 49 by the Finance Act, 2015,
w.e.f. 1-4-2016 :
(2AD)Where the capital asset, being a unit or units in a consolidated scheme of a mutual fund, became the property
of the assessee in consideration of a transfer referred to in clause (xviii)of section 47, the cost of acquisition of the
asset shall be deemed to be the cost of acquisition to him of the unit or units in the consolidating scheme of the
mutual fund.
(2B) [***]

Capital Gains

10.93

(2C) The cost of acquisition of the shares in the resulting company shall be the amount which bears to the cost of
acquisition of shares held by the assessee in the demerged company the same proportion as the net book value of the
assets transferred in a demerger bears to the net worth of the demerged company immediately before such demerger.
(2D) The cost of acquisition of the original shares held by the shareholder in the demerged company shall be
deemed to have been reduced by the amount as so arrived at under sub-section (2C).
(2E) The provisions of sub-section (2), sub-section (2C) and sub-section (2D) shall, as far as may be, also apply in
relation to business reorganisation of a co-operative bank as referred to in section 44DB.
Explanation.For the purposes of this section, "net worth" shall mean the aggregate of the paid up share capital and
general reserves as appearing in the books of account of the demerged company immediately before the demerger.
(3) Notwithstanding anything contained in sub-section (1), where the capital gain arising from the transfer of a
capital asset referred to in clause (iv) or, as the case may be, clause (v) of section 47 is deemed to be income
chargeable under the head "Capital gains" by virtue of the provisions contained in section 47A, the cost of
acquisition of such asset to the transferee-company shall be the cost for which such asset was acquired by it.
(4) Where the capital gain arises from the transfer of a property, the value of which has been subject to income-tax
under clause (vii) or clause (viia)of sub-section (2) of section 56, the cost of acquisition of such property shall be
deemed to be the value which has been taken into account for the purposes of the said clause (vii) or clause (viia).
Special provision for computation of capital gains in case of depreciable assets.
50. Notwithstanding anything contained in clause (42A) of section 2, where the capital asset is an asset forming part
of a block of assets in respect of which depreciation has been allowed under this Act or under the Indian Income-tax
Act, 1922 (11 of 1922), the provisions of sections 48 and 49 shall be subject to the following modifications :
(1) where the full value of the consideration received or accruing as a result of the transfer of the asset together
with the full value of such consideration received or accruing as a result of the transfer of any other capital
asset falling within the block of the assets during the previous year, exceeds the aggregate of the following
amounts, namely :
(i) expenditure incurred wholly and exclusively in connection with such transfer or transfers;
(ii) the written down value of the block of assets at the beginning of the previous year; and
(iii) the actual cost of any asset falling within the block of assets acquired during the previous year,
such excess shall be deemed to be the capital gains arising from the transfer of short-term capital assets;
(2) where any block of assets ceases to exist as such, for the reason that all the assets in that block are
transferred during the previous year, the cost of acquisition of the block of assets shall be the written down
value of the block of assets at the beginning of the previous year, as increased by the actual cost of any
asset falling within that block of assets, acquired by the assessee during the previous year and the income
received or accruing as a result of such transfer or transfers shall be deemed to be the capital gains arising
from the transfer of short-term capital assets.
Special provision for cost of acquisition in case of depreciable asset.
50A. Where the capital asset is an asset in respect of which a deduction on account of depreciation under clause (i)
of sub-section (1) of section 32has been obtained by the assessee in any previous year, the provisions of sections
48 and 49 shall apply subject to the modification that the written down value, as defined in clause (6) of section 43,
of the asset, as adjusted, shall be taken as the cost of acquisition of the asset.

Income Tax

10.94

Special provision for computation of capital gains in case of slump sale.


50B. (1) Any profits or gains arising from the slump sale effected in the previous year shall be chargeable to
income-tax as capital gains arising from the transfer of long-term capital assets and shall be deemed to be the
income of the previous year in which the transfer took place :
Provided that any profits or gains arising from the transfer under the slump sale of any capital asset being one or
more undertakings owned and held by an assessee for not more than thirty-six months immediately preceding the
date of its transfer shall be deemed to be the capital gains arising from the transfer of short-term capital assets.
(2) In relation to capital assets being an undertaking or division transferred by way of such sale, the "net worth" of
the undertaking or the division, as the case may be, shall be deemed to be the cost of acquisition and the cost of
improvement for the purposes of sections 48 and 49and no regard shall be given to the provisions contained in the
second proviso to section 48.
(3) Every assessee, in the case of slump sale, shall furnish in the prescribed form 1 along with the return of income, a
report of an accountant as defined in the Explanation below sub-section (2) of section 288, indicating the
computation of the net worth of the undertaking or division, as the case may be, and certifying that the net worth of
the undertaking or division, as the case may be, has been correctly arrived at in accordance with the provisions of
this section.
Explanation 1.For the purposes of this section, "net worth" shall be the aggregate value of total assets of the
undertaking or division as reduced by the value of liabilities of such undertaking or division as appearing in its
books of account :
Provided that any change in the value of assets on account of revaluation of assets shall be ignored for the purposes
of computing the net worth.
Explanation 2.For computing the net worth, the aggregate value of total assets shall be,
(a) in the case of depreciable assets, the written down value of the block of assets determined in accordance
with the provisions contained in sub-item (C) of item (i) of sub-clause (c) of clause (6) of section 43;
(b) in the case of capital assets in respect of which the whole of the expenditure has been allowed or is
allowable as a deduction undersection 35AD, nil; and
(c) in the case of other assets, the book value of such assets.
Special provision for full value of consideration in certain cases.
50C. (1) Where the consideration received or accruing as a result of the transfer by an assessee of a capital asset,
being land or building or both, is less than the value adopted or assessed or assessable by any authority of a State
Government (hereafter in this section referred to as the "stamp valuation authority") for the purpose of payment of
stamp duty in respect of such transfer, the value so adopted or assessed or assessable shall, for the purposes
of section 48, be deemed to be the full value of the consideration received or accruing as a result of such transfer.
(2) Without prejudice to the provisions of sub-section (1), where
(a) the assessee claims before any Assessing Officer that the value adopted or assessed or assessable by the
stamp valuation authority under sub-section (1) exceeds the fair market value of the property as on the
date of transfer;
(b) the value so adopted or assessed or assessable by the stamp valuation authority under sub-section (1) has
not been disputed in any appeal or revision or no reference has been made before any other authority,
court or the High Court,
the Assessing Officer may refer the valuation of the capital asset to a Valuation Officer and where any such
reference is made, the provisions of sub-sections (2), (3), (4), (5) and (6) of section 16A, clause (i) of sub-section (1)
and sub-sections (6) and (7) of section 23A, sub-section (5) of section 24, section 34AA, section 35 and section 37

Capital Gains

10.95

of the Wealth-tax Act, 1957 (27 of 1957), shall, with necessary modifications, apply in relation to such reference as
they apply in relation to a reference made by the Assessing Officer under sub-section (1) of section 16A of that Act.
Explanation 1.For the purposes of this section, "Valuation Officer" shall have the same meaning as in clause (r) of
section 2 of the Wealth-tax Act, 1957 (27 of 1957).
Explanation 2.For the purposes of this section, the expression "assessable" means the price which the stamp
valuation authority would have, notwithstanding anything to the contrary contained in any other law for the time
being in force, adopted or assessed, if it were referred to such authority for the purposes of the payment of stamp
duty.
(3) Subject to the provisions contained in sub-section (2), where the value ascertained under sub-section (2) exceeds
the value adopted or assessed or assessable by the stamp valuation authority referred to in sub-section (1), the value
so adopted or assessed or assessable by such authority shall be taken as the full value of the consideration received
or accruing as a result of the transfer.
Fair market value deemed to be full value of consideration in certain cases.
50D. Where the consideration received or accruing as a result of the transfer of a capital asset by an assessee is not
ascertainable or cannot be determined, then, for the purpose of computing income chargeable to tax as capital gains,
the fair market value of the said asset on the date of transfer shall be deemed to be the full value of the consideration
received or accruing as a result of such transfer.
Advance money received.
51. Where any capital asset was on any previous occasion the subject of negotiations for its transfer, any advance or
other money received and retained by the assessee in respect of such negotiations shall be deducted from the cost for
which the asset was acquired or the written down value or the fair market value, as the case may be, in computing
the cost of acquisition :
2

[Provided that where any sum of money, received as an advance or otherwise in the course of negotiations for
transfer of a capital asset, has been included in the total income of the assessee for any previous year in accordance
with the provisions of clause (ix) of sub-section (2) of section 56, then, such sum shall not be deducted from the cost
for which the asset was acquired or the written down value or the fair market value, as the case may be, in
computing the cost of acquisition.]
Profit on sale of property used for residence.
54. (1) Subject to the provisions of sub-section (2), where, in the case of an assessee being an individual or a Hindu
undivided family, the capital gain arises from the transfer of a long-term capital asset, being buildings or lands
appurtenant thereto, and being a residential house, the income of which is chargeable under the head "Income from
house property" (hereafter in this section referred to as the original asset), and the assessee has within a period of
one year before or two years after the date on which the transfer took place purchased, or has within a period of
three years after that date 3[constructed, one residential house in India], then, instead of the capital gain being
charged to income-tax as income of the previous year in which the transfer took place, it shall be dealt with in
accordance with the following provisions of this section, that is to say,
(i) if the amount of the capital gain is greater than the cost of the residential house so purchased or constructed
(hereafter in this section referred to as the new asset), the difference between the amount of the capital
gain and the cost of the new asset shall be charged undersection 45 as the income of the previous year; and
for the purpose of computing in respect of the new asset any capital gain arising from its transfer within a
period of three years of its purchase or construction, as the case may be, the cost shall be nil; or

Income Tax

10.96

(ii) if the amount of the capital gain is equal to or less than the cost of the new asset, the capital gain shall not
be charged under section 45; and for the purpose of computing in respect of the new asset any capital gain
arising from its transfer within a period of three years of its purchase or construction, as the case may be,
the cost shall be reduced by the amount of the capital gain.
(2) The amount of the capital gain which is not appropriated by the assessee towards the purchase of the new asset
made within one year before the date on which the transfer of the original asset took place, or which is not utilised
by him for the purchase or construction of the new asset before the date of furnishing the return of income
under section 139, shall be deposited by him before furnishing such return [such deposit being made in any case not
later than the due date applicable in the case of the assessee for furnishing the return of income under sub-section (1)
ofsection 139] in an account in any such bank or institution as may be specified in, and utilised in accordance with,
any scheme which the Central Government may, by notification in the Official Gazette, frame in this behalf and
such return shall be accompanied by proof of such deposit; and, for the purposes of sub-section (1), the amount, if
any, already utilised by the assessee for the purchase or construction of the new asset together with the amount so
deposited shall be deemed to be the cost of the new asset :
Provided that if the amount deposited under this sub-section is not utilised wholly or partly for the purchase or
construction of the new asset within the period specified in sub-section (1), then,
(i) the amount not so utilised shall be charged under section 45 as the income of the previous year in which the
period of three years from the date of the transfer of the original asset expires; and
(ii) the assessee shall be entitled to withdraw such amount in accordance with the scheme aforesaid.
Explanation.[Omitted by the Finance Act, 1992, w.e.f. 1-4-1993.]
Capital gain on transfer of land used for agricultural purposes not to be charged in certain cases.
54B. (1) Subject to the provisions of sub-section (2), where the capital gain arises from the transfer of a capital asset
being land which, in the two years immediately preceding the date on which the transfer took place, was being used
by the assessee being an individual or his parent, or a Hindu undivided family for agricultural purposes (hereinafter
referred to as the original asset), and the assessee has, within a period of two years after that date, purchased any
other land for being used for agricultural purposes, then, instead of the capital gain being charged to income-tax as
income of the previous year in which the transfer took place, it shall be dealt with in accordance with the following
provisions of this section, that is to say,
(i) if the amount of the capital gain is greater than the cost of the land so purchased (hereinafter referred to as
the new asset), the difference between the amount of the capital gain and the cost of the new asset shall be
charged under section 45 as the income of the previous year; and for the purpose of computing in respect
of the new asset any capital gain arising from its transfer within a period of three years of its purchase, the
cost shall be nil; or
(ii) if the amount of the capital gain is equal to or less than the cost of the new asset, the capital gain shall not
be charged under section 45; and for the purpose of computing in respect of the new asset any capital gain
arising from its transfer within a period of three years of its purchase, the cost shall be reduced, by the
amount of the capital gain.
(2) The amount of the capital gain which is not utilised by the assessee for the purchase of the new asset before the
date of furnishing the return of income under section 139, shall be deposited by him before furnishing such return
[such deposit being made in any case not later than the due date applicable in the case of the assessee for furnishing
the return of income under sub-section (1) of section 139] in an account in any such bank or institution as may be
specified in, and utilised in accordance with, any scheme which the Central Government may, by notification in the
Official Gazette, frame in this behalf and such return shall be accompanied by proof of such deposit; and, for the
purposes of sub-section (1), the amount, if any, already utilised by the assessee for the purchase of the new asset
together with the amount so deposited shall be deemed to be the cost of the new asset :

Capital Gains

10.97

Provided that if the amount deposited under this sub-section is not utilised wholly or partly for the purchase of the
new asset within the period specified in sub-section (1), then,
(i) the amount not so utilised shall be charged under section 45 as the income of the previous year in which the
period of two years from the date of the transfer of the original asset expires; and
(ii) the assessee shall be entitled to withdraw such amount in accordance with the scheme aforesaid.
Explanation.[Omitted by the Finance Act, 1992, w.e.f. 1-4-1993.]
Capital gain on compulsory acquisition of lands and buildings not to be charged in certain cases.
54D. (1) Subject to the provisions of sub-section (2), where the capital gain arises from the transfer by way of
compulsory acquisition under any law of a capital asset, being land or building or any right in land or building,
forming part of an industrial undertaking belonging to the assessee which, in the two years immediately preceding
the date on which the transfer took place, was being used by the assessee for the purposes of the business of the said
undertaking (hereafter in this section referred to as the original asset), and the assessee has within a period of three
years after that date purchased any other land or building or any right in any other land or building or constructed
any other building for the purposes of shifting or re-establishing the said undertaking or setting up another industrial
undertaking, then, instead of the capital gain being charged to income-tax as the income of the previous year in
which the transfer took place, it shall be dealt with in accordance with the following provisions of this section, that
is to say,
(i) if the amount of the capital gain is greater than the cost of the land, building or right so purchased or the
building so constructed (such land, building or right being hereafter in this section referred to as the new
asset), the difference between the amount of the capital gain and the cost of the new asset shall be charged
under section 45 as the income of the previous year; and for the purpose of computing in respect of the
new asset any capital gain arising from its transfer within a period of three years of its purchase or
construction, as the case may be, the cost shall be nil; or
(ii) if the amount of the capital gain is equal to or less than the cost of the new asset, the capital gain shall not
be charged under section 45; and for the purpose of computing in respect of the new asset any capital gain
arising from its transfer within a period of three years of its purchase or construction, as the case may be,
the cost shall be reduced by the amount of the capital gain.
(2) The amount of the capital gain which is not utilised by the assessee for the purchase or construction of the new
asset before the date of furnishing the return of income under section 139, shall be deposited by him before
furnishing such return [such deposit being made in any case not later than the due date applicable in the case of the
assessee for furnishing the return of income under sub-section (1) of section 139] in an account in any such bank or
institution as may be specified in, and utilised in accordance with, any scheme which the Central Government may,
by notification in the Official Gazette, frame in this behalf and such return shall be accompanied by proof of such
deposit; and, for the purposes of sub-section (1), the amount, if any, already utilised by the assessee for the purchase
or construction of the new asset together with the amount so deposited shall be deemed to be the cost of the new
asset:
Provided that if the amount deposited under this sub-section is not utilised wholly or partly for the purchase or
construction of the new asset within the period specified in sub-section (1), then,
(i) the amount not so utilised shall be charged under section 45 as the income of the previous year in which the
period of three years from the date of the transfer of the original asset expires; and
(ii) the assessee shall be entitled to withdraw such amount in accordance with the scheme aforesaid.
Explanation.[Omitted by the Finance Act, 1992, w.e.f. 1-4-1993.]

Income Tax

10.98

Capital gain not to be charged on investment in certain bonds.


54EC. (1) Where the capital gain arises from the transfer of a long-term capital asset (the capital asset so transferred
being hereafter in this section referred to as the original asset) and the assessee has, at any time within a period of six
months after the date of such transfer, invested the whole or any part of capital gains in the long-term specified
asset, the capital gain shall be dealt with in accordance with the following provisions of this section, that is to say,
(a) if the cost of the long-term specified asset is not less than the capital gain arising from the transfer of the
original asset, the whole of such capital gain shall not be charged under section 45;
(b) if the cost of the long-term specified asset is less than the capital gain arising from the transfer of the original
asset, so much of the capital gain as bears to the whole of the capital gain the same proportion as the cost of
acquisition of the long-term specified asset bears to the whole of the capital gain, shall not be charged
under section 45 :
Provided that the investment made on or after the 1st day of April, 2007 in the long-term specified asset by an
assessee during any financial year does not exceed fifty lakh rupees :
4

[Provided further that the investment made by an assessee in the long-term specified asset, from capital gains
arising from transfer of one or more original assets, during the financial year in which the original asset or assets
are transferred and in the subsequent financial year does not exceed fifty lakh rupees.]
(2) Where the long-term specified asset is transferred or converted (otherwise than by transfer) into money at any
time within a period of three years from the date of its acquisition, the amount of capital gains arising from the
transfer of the original asset not charged under section 45 on the basis of the cost of such long-term specified asset
as provided in clause (a) or, as the case may be, clause (b) of sub-section (1) shall be deemed to be the income
chargeable under the head "Capital gains" relating to long-term capital asset of the previous year in which the longterm specified asset is transferred or converted (otherwise than by transfer) into money.
Explanation.In a case where the original asset is transferred and the assessee invests the whole or any part of the
capital gain received or accrued as a result of transfer of the original asset in any long-term specified asset and such
assessee takes any loan or advance on the security of such specified asset, he shall be deemed to have converted
(otherwise than by transfer) such specified asset into money on the date on which such loan or advance is taken.
(3) Where the cost of the long-term specified asset has been taken into account for the purposes of clause (a) or
clause (b) of sub-section (1),
(a) a deduction from the amount of income-tax with reference to such cost shall not be allowed under section
88 for any assessment year ending before the 1st day of April, 2006;
(b) a deduction from the income with reference to such cost shall not be allowed under section 80C for any
assessment year beginning on or after the 1st day of April, 2006.
Explanation.For the purposes of this section,
(a) "cost", in relation to any long-term specified asset, means the amount invested in such specified asset out of
capital gains received or accruing as a result of the transfer of the original asset;
(b) "long-term specified asset" for making any investment under this section during the period commencing from
the 1st day of April, 2006 and ending with the 31st day of March, 2007, means any bond, redeemable after
three years and issued on or after the 1st day of April, 2006, but on or before the 31st day of March, 2007,
(i) by the National Highways Authority of India constituted under section 3 of the National Highways
Authority of India Act, 1988 (68 of 1988); or
(ii) by the Rural Electrification Corporation Limited, a company formed and registered under the
Companies Act, 1956 (1 of 1956),
and notified by the Central Government in the Official Gazette for the purposes of this section with such
conditions (including the condition for providing a limit on the amount of investment by an assessee in such
bond) as it thinks fit:

Capital Gains

10.99

Provided that where any bond has been notified before the 1st day of April, 2007, subject to the conditions
specified in the notification, by the Central Government in the Official Gazette under the provisions of clause
(b) as they stood immediately before their amendment by the Finance Act, 2007, such bond shall be deemed
to be a bond notified under this clause;
(ba) "long-term specified asset" for making any investment under this section on or after the 1st day of April, 2007
means any bond, redeemable after three years and issued on or after the 1st day of April, 2007 by the
National Highways Authority of India constituted under section 3 of the National Highways Authority of
India Act, 1988 (68 of 1988) or by the Rural Electrification Corporation Limited, a company formed and
registered under the Companies Act, 1956 (1 of 1956).
Capital gain on transfer of certain capital assets not to be charged in case of investment in residential house.
54F. (1) Subject to the provisions of sub-section (4), where, in the case of an assessee being an individual or a Hindu
undivided family, the capital gain arises from the transfer of any long-term capital asset, not being a residential
house (hereafter in this section referred to as the original asset), and the assessee has, within a period of one year
before or two years after the date on which the transfer took place purchased, or has within a period of three years
after that date 5[constructed, one residential house in India] (hereafter in this section referred to as the new asset),
the capital gain shall be dealt with in accordance with the following provisions of this section, that is to say,
(a) if the cost of the new asset is not less than the net consideration in respect of the original asset, the whole of
such capital gain shall not be charged under section 45 ;
(b) if the cost of the new asset is less than the net consideration in respect of the original asset, so much of the
capital gain as bears to the whole of the capital gain the same proportion as the cost of the new asset bears to
the net consideration, shall not be charged under section 45:
Provided that nothing contained in this sub-section shall apply where
(a) the assessee,
(i) owns more than one residential house, other than the new asset, on the date of transfer of the original
asset; or
(ii) purchases any residential house, other than the new asset, within a period of one year after the date of
transfer of the original asset; or
(iii) constructs any residential house, other than the new asset, within a period of three years after the date
of transfer of the original asset; and
(b) the income from such residential house, other than the one residential house owned on the date of transfer of
the original asset, is chargeable under the head "Income from house property".
Explanation.For the purposes of this section,
"net consideration", in relation to the transfer of a capital asset, means the full value of the consideration
received or accruing as a result of the transfer of the capital asset as reduced by any expenditure incurred
wholly and exclusively in connection with such transfer.
(2) Where the assessee purchases, within the period of two years after the date of the transfer of the original asset, or
constructs, within the period of three years after such date, any residential house, the income from which is
chargeable under the head "Income from house property", other than the new asset, the amount of capital gain
arising from the transfer of the original asset not charged under section 45 on the basis of the cost of such new asset
as provided in clause (a), or, as the case may be, clause (b), of sub-section (1), shall be deemed to be income
chargeable under the head "Capital gains" relating to long-term capital assets of the previous year in which such
residential house is purchased or constructed.
(3) Where the new asset is transferred within a period of three years from the date of its purchase or, as the case may
be, its construction, the amount of capital gain arising from the transfer of the original asset not charged

Income Tax

10.100

under section 45 on the basis of the cost of such new asset as provided in clause (a) or, as the case may be, clause
(b), of sub-section (1) shall be deemed to be income chargeable under the head "Capital gains" relating to long-term
capital assets of the previous year in which such new asset is transferred.
(4) The amount of the net consideration which is not appropriated by the assessee towards the purchase of the new
asset made within one year before the date on which the transfer of the original asset took place, or which is not
utilised by him for the purchase or construction of the new asset before the date of furnishing the return of income
under section 139, shall be deposited by him before furnishing such return [such deposit being made in any case not
later than the due date applicable in the case of the assessee for furnishing the return of income under sub-section (1)
of section 139] in an account in any such bank or institution as may be specified in, and utilised in accordance with,
any scheme which the Central Government may, by notification in the Official Gazette, frame in this behalf and
such return shall be accompanied by proof of such deposit; and, for the purposes of sub-section (1), the amount, if
any, already utilised by the assessee for the purchase or construction of the new asset together with the amount so
deposited shall be deemed to be the cost of the new asset :
Provided that if the amount deposited under this sub-section is not utilised wholly or partly for the purchase or
construction of the new asset within the period specified in sub-section (1), then,
(i) the amount by which
(a) the amount of capital gain arising from the transfer of the original asset not charged under section
45 on the basis of the cost of the new asset as provided in clause (a) or, as the case may be, clause
(b) of sub-section (1),
exceeds
(b) the amount that would not have been so charged had the amount actually utilised by the assessee for
the purchase or construction of the new asset within the period specified in sub-section (1) been the
cost of the new asset,
shall be charged under section 45 as income of the previous year in which the period of three years from the
date of the transfer of the original asset expires; and
(ii) the assessee shall be entitled to withdraw the unutilised amount in accordance with the scheme aforesaid.
Explanation.[Omitted by the Finance Act, 1992, w.e.f. 1-4-1993.]
Exemption of capital gains on transfer of assets in cases of shifting of industrial undertaking from urban
area.
54G. (1) Subject to the provisions of sub-section (2), where the capital gain arises from the transfer of a capital
asset, being machinery or plant or building or land or any rights in building or land used for the purposes of the
business of an industrial undertaking situate in an urban area, effected in the course of, or in consequence of, the
shifting of such industrial undertaking (hereafter in this section referred to as the original asset) to any area (other
than an urban area) and the assessee has within a period of one year before or three years after the date on which the
transfer took place,
(a) purchased new machinery or plant for the purposes of business of the industrial undertaking in the area to
which the said undertaking is shifted;
(b) acquired building or land or constructed building for the purposes of his business in the said area ;
(c) shifted the original asset and transferred the establishment of such undertaking to such area; and
(d) incurred expenses on such other purpose as may be specified in a scheme framed by the Central Government
for the purposes of this section,
then, instead of the capital gain being charged to income-tax as income of the previous year in which the transfer
took place, it shall be dealt with in accordance with the following provisions of this section, that is to say,

Capital Gains

10.101

(i) if the amount of the capital gain is greater than the cost and expenses incurred in relation to all or any of the
purposes mentioned in clauses (a) to (d) (such cost and expenses being hereafter in this section referred to as
the new asset), the difference between the amount of the capital gain and the cost of the new asset shall be
charged under section 45 as the income of the previous year; and for the purpose of computing in respect of
the new asset any capital gain arising from its transfer within a period of three years of its being purchased,
acquired, constructed or transferred, as the case may be, the cost shall be nil ; or
(ii) if the amount of the capital gain is equal to, or less than, the cost of the new asset, the capital gain shall not be
charged under section 45; and for the purpose of computing in respect of the new asset any capital gain
arising from its transfer within a period of three years of its being purchased, acquired, constructed or
transferred, as the case may be, the cost shall be reduced by the amount of the capital gain.
Explanation.In this sub-section, "urban area" means any such area within the limits of a municipal corporation or
municipality as the Central Government may, having regard to the population, concentration of industries, need for
proper planning of the area and other relevant factors, by general or special order, declare to be an urban area for the
purposes of this sub-section.
(2) The amount of capital gain which is not appropriated by the assessee towards the cost and expenses incurred in
relation to all or any of the purposes mentioned in clauses (a) to (d) of sub-section (1) within one year before the
date on which the transfer of the original asset took place, or which is not utilised by him for all or any of the
purposes aforesaid before the date of furnishing the return of income under section 139, shall be deposited by him
before furnishing such return [such deposit being made in any case not later than the due date applicable in the case
of the assessee for furnishing the return of income under sub-section (1) of section 139] in an account in any such
bank or institution as may be specified in, and utilised in accordance with, any scheme which the Central
Government may, by notification in the Official Gazette, frame in this behalf and such return shall be accompanied
by proof of such deposit ; and, for the purposes of sub-section (1), the amount, if any, already utilised by the
assessee for all or any of the purposes aforesaid together with the amount, so deposited shall be deemed to be the
cost of the new asset :
Provided that if the amount deposited under this sub-section is not utilised wholly or partly for all or any of the
purposes mentioned in clauses (a) to (d) of sub-section (1) within the period specified in that sub-section, then,
(i) the amount not so utilised shall be charged under section 45 as the income of the previous year in which the
period of three years from the date of the transfer of the original asset expires ; and
(ii) the assessee shall be entitled to withdraw such amount in accordance with the scheme aforesaid.
Explanation.[Omitted by the Finance Act, 1992, w.e.f. 1-4-1993.]
Exemption of capital gains on transfer of assets in cases of shifting of industrial undertaking from urban area
to any Special Economic Zone.
54GA. (1) Notwithstanding anything contained in section 54G, where the capital gain arises from the transfer of a
capital asset, being machinery or plant or building or land or any rights in building or land used for the purposes of
the business of an industrial undertaking situate in an urban area, effected in the course of, or in consequence of the
shifting of such industrial undertaking to any Special Economic Zone, whether developed in any urban area or any
other area and the assessee has within a period of one year before or three years after the date on which the transfer
took place,
(a) purchased machinery or plant for the purposes of business of the industrial undertaking in the Special
Economic Zone to which the said undertaking is shifted;
(b) acquired building or land or constructed building for the purposes of his business in the Special Economic
Zone;
(c) shifted the original asset and transferred the establishment of such undertaking to the Special Economic Zone;
and

Income Tax

10.102

(d) incurred expenses on such other purposes as may be specified in a scheme framed by the Central Government
for the purposes of this section,
then, instead of the capital gain being charged to income-tax as income of the previous year in which the transfer
took place, it shall, subject to the provisions of sub-section (2), be dealt with in accordance with the following
provisions of this section, that is to say,
(i) if the amount of the capital gain is greater than the cost and expenses incurred in relation to all or any of the
purposes mentioned in clauses (a) to (d) (such cost and expenses being hereafter in this section referred to as
the new asset), the difference between the amount of the capital gain and the cost of the new asset shall be
charged under section 45 as the income of the previous year; and for the purpose of computing in respect of
the new asset any capital gain arising from its transfer within a period of three years of its being purchased,
acquired, constructed or transferred, as the case may be, the cost shall be Nil; or
(ii) if the amount of the capital gain is equal to, or less than, the cost of the new asset, the capital gain shall not be
charged under section 45, and for the purpose of computing in respect of the new asset any capital gain
arising from its transfer within a period of three years of its being purchased, acquired, constructed or
transferred, as the case may be, the cost shall be reduced by the amount of the capital gain.
Explanation.In this sub-section,
(a) "Special Economic Zone" shall have the meaning assigned to it in clause (za) of *[section 2 of] the Special
Economic Zones Act, 2005;
(b) "urban area" means any such area within the limits of a municipal corporation or municipality as the Central
Government may, having regard to the population, concentration of industries, need for proper planning of
the area and other relevant factors, by general or special order, declare to be an urban area for the purposes of
this sub-section.
(2) The amount of capital gain which is not appropriated by the assessee towards the cost and expenses incurred in
relation to all or any of the purposes mentioned in clauses (a) to (d) of sub-section (1) within one year before the
date on which the transfer of the original asset took place, or which is not utilised by him for all or any of the
purposes aforesaid before the date of furnishing the return of income under section 139, shall be deposited by him
before furnishing such return [such deposit being made in any case not later than the due date applicable in the case
of the assessee for furnishing the return of income under sub-section (1) of section 139] in an account in any such
bank or institution as may be specified in, and utilised in accordance with, any scheme which the Central
Government may, by notification, frame in this behalf and such return shall be accompanied by proof of such
deposit; and, for the purposes of sub-section (1), the amount, if any, already utilised by the assessee for all or any of
the aforesaid purposes together with the amount so deposited shall be deemed to be the cost of the new asset :
Provided that if the amount deposited under this sub-section is not utilised wholly or partly for all or any of the
purposes mentioned in clauses (a) to (d) of sub-section (1) within the period specified in that sub-section, then,
(i) the amount not so utilised shall be charged under section 45 as the income of the previous year in which the
period of three years from the date of the transfer of the original asset expires; and
(ii) the assessee shall be entitled to withdraw such amount in accordance with the scheme aforesaid.
Capital gain on transfer of residential property not to be charged in certain cases
54GB. (1) Where,
(i) the capital gain arises from the transfer of a long-term capital asset, being a residential property (a house or a
plot of land), owned by the eligible assessee (herein referred to as the assessee); and
(ii) the assessee, before the due date of furnishing of return of income under sub-section (1) of section 139,
utilises the net consideration for subscription in the equity shares of an eligible company (herein-referred to
as the company); and

Capital Gains

10.103

(iii) the company has, within one year from the date of subscription in equity shares by the assessee, utilised this
amount for purchase of new asset,
then, instead of the capital gain being charged to income-tax as the income of the previous year in which the transfer
takes place, it shall be dealt with in accordance with the following provisions of this section, that is to say,
(a) if the amount of the net consideration is greater than the cost of the new asset, then, so much of the capital
gain as it bears to the whole of the capital gain the same proportion as the cost of the new asset bears to the
net consideration, shall not be charged under section 45 as the income of the previous year; or
(b) if the amount of the net consideration is equal to or less than the cost of the new asset, the capital gain shall
not be charged under section 45 as the income of the previous year.
(2) The amount of the net consideration, which has been received by the company for issue of shares to the assessee,
to the extent it is not utilised by the company for the purchase of the new asset before the due date of furnishing of
the return of income by the assessee under section 139, shall be deposited by the company, before the said due date
in an account in any such bank or institution as may be specified and shall be utilised in accordance with any scheme
which the Central Government may, by notification in the Official Gazette, frame in this behalf and the return
furnished by the assessee shall be accompanied by proof of such deposit having been made.
(3) For the purposes of sub-section (1), the amount, if any, already utilised by the company for the purchase of the
new asset together with the amount deposited under sub-section (2) shall be deemed to be the cost of the new asset:
Provided that if the amount so deposited is not utilised, wholly or partly, for the purchase of the new asset within
the period specified in sub-section (1), then,
(i) the amount by which
(a) the amount of capital gain arising from the transfer of the residential property not charged
under section 45 on the basis of the cost of the new asset as provided in sub-section (1),
exceeds
(b) the amount that would not have been so charged had the amount actually utilised for the purchase of
the new asset within the period specified in sub-section (1) been the cost of the new asset,
shall be charged under section 45 as income of the assessee for the previous year in which the period of one
year from the date of the subscription in equity shares by the assessee expires; and
(ii) the company shall be entitled to withdraw such amount in accordance with the scheme.
(4) If the equity shares of the company or the new asset acquired by the company are sold or otherwise transferred
within a period of five years from the date of their acquisition, the amount of capital gain arising from the transfer of
the residential property not charged under section 45 as provided in sub-section (1) shall be deemed to be the income
of the assessee chargeable under the head "Capital gains" of the previous year in which such equity shares or such
new asset are sold or otherwise transferred, in addition to taxability of gains, arising on account of transfer of shares
or of the new asset, in the hands of the assessee or the company, as the case may be.
(5) The provisions of this section shall not apply to any transfer of residential property made after the 31st day of
March, 2017.
(6) For the purposes of this section,
(a) "eligible assessee" means an individual or a Hindu undivided family;
(b) "eligible company" means a company which fulfils the following conditions, namely:
(i) it is a company incorporated in India during the period from the 1st day of April of the previous year
relevant to the assessment year in which the capital gain arises to the due date of furnishing of return
of income under sub-section (1) of section 139 by the assessee;
(ii) it is engaged in the business of manufacture of an article or a thing;

Income Tax

10.104

(iii) it is a company in which the assessee has more than fifty per cent share capital or more than fifty per
cent voting rights after the subscription in shares by the assessee; and
(iv) it is a company which qualifies to be a small or medium enterprise under the Micro, Small and
Medium Enterprises Act, 2006 (27 of 2006);
(c) "net consideration" shall have the meaning assigned to it in the Explanation to section 54F;
(d) "new asset" means new plant and machinery but does not include
(i) any machinery or plant which, before its installation by the assessee, was used either within or outside
India by any other person;
(ii) any machinery or plant installed in any office premises or any residential accommodation, including
accommodation in the nature of a guest-house;
(iii) any office appliances including computers or computer software;
(iv) any vehicle; or
(v) any machinery or plant, the whole of the actual cost of which is allowed as a deduction (whether by
way of depreciation or otherwise) in computing the income chargeable under the head "Profits and
gains of business or profession" of any previous year.
Extension of time for acquiring new asset or depositing or investing amount of capital gain.
54H. Notwithstanding anything contained in sections 54, 54B, 54D, 54EC and 54F, where the transfer of the
original asset is by way of compulsory acquisition under any law and the amount of compensation awarded for such
acquisition is not received by the assessee on the date of such transfer, the period for acquiring the new asset by the
assessee referred to in those sections or, as the case may be, the period available to the assessee under those sections
for depositing or investing the amount of capital gain in relation to such compensation as is not received on the date
of the transfer, shall be reckoned from the date of receipt of such compensation :
Provided that where the compensation in respect of transfer of the original asset by way of compulsory acquisition
under any law is received before the 1st day of April, 1991, the aforesaid period or periods, if expired, shall extend
up to the 31st day of December, 1991.
Meaning of "adjusted", "cost of improvement" and "cost of acquisition".
55. (1) For the purposes of sections 48 and 49,
(a) [***]
(b) "cost of any improvement",
(1) in relation to a capital asset being goodwill of a business or a right to manufacture, produce or process
any article or thing or right to carry on any business shall be taken to be nil ; and
(2) in relation to any other capital asset,
(i) where the capital asset became the property of the previous owner or the assessee before the
1st day of April, 1981, means all expenditure of a capital nature incurred in making any
additions or alterations to the capital asset on or after the said date by the previous owner or
the assessee, and
(ii) in any other case, means all expenditure of a capital nature incurred in making any additions
or alterations to the capital asset by the assessee after it became his property, and, where the
capital asset became the property of the assessee by any of the modes specified in sub-section
(1) of section 49, by the previous owner,
but does not include any expenditure which is deductible in computing the income chargeable under
the head "Interest on securities", "Income from house property", "Profits and gains of business or

Capital Gains

10.105

profession", or "Income from other sources", and the expression "improvement" shall be construed
accordingly.
(2) For the purposes of sections 48 and 49, "cost of acquisition",
(a) in relation to a capital asset, being goodwill of a business or a trade mark or brand name associated
with a business or a right to manufacture, produce or process any article or thing or right to carry on
any business, tenancy rights, stage carriage permits or loom hours,
(i) in the case of acquisition of such asset by the assessee by purchase from a previous owner,
means the amount of the purchase price ; and
(ii) in any other case [not being a case falling under sub-clauses (i) to (iv) of sub-section (1)
of section 49], shall be taken to benil ;
(aa) in a case where, by virtue of holding a capital asset, being a share or any other security, within the
meaning of clause (h) of section 2 of the Securities Contracts (Regulation) Act, 1956 (42 of 1956)
(hereafter in this clause referred to as the financial asset), the assessee
(A) becomes entitled to subscribe to any additional financial asset ; or
(B) is allotted any additional financial asset without any payment,
then, subject to the provisions of sub-clauses (i) and (ii) of clause (b),
(i) in relation to the original financial asset, on the basis of which the assessee becomes entitled to
any additional financial asset, means the amount actually paid for acquiring the original
financial asset ;
(ii) in relation to any right to renounce the said entitlement to subscribe to the financial asset,
when such right is renounced by the assessee in favour of any person, shall be taken to
be nil in the case of such assessee ;
(iii) in relation to the financial asset, to which the assessee has subscribed on the basis of the said
entitlement, means the amount actually paid by him for acquiring such asset ;
(iiia) in relation to the financial asset allotted to the assessee without any payment and on the basis
of holding of any other financial asset, shall be taken to be nil in the case of such assessee ;
and
(iv) in relation to any financial asset purchased by any person in whose favour the right to
subscribe to such asset has been renounced, means the aggregate of the amount of the
purchase price paid by him to the person renouncing such right and the amount paid by him to
the company or institution, as the case may be, for acquiring such financial asset ;
(ab) in relation to a capital asset, being equity share or shares allotted to a shareholder of a recognised
stock exchange in India under a scheme for demutualisation or corporatisation approved by the
Securities and Exchange Board of India established under section 3 of the Securities and Exchange
Board of India Act, 1992 (15 of 1992), shall be the cost of acquisition of his original membership of
the exchange:
Provided that the cost of a capital asset, being trading or clearing rights of the recognised stock
exchange acquired by a shareholder who has been allotted equity share or shares under such scheme
of demutualisation or corporatisation, shall be deemed to be nil;
(b) in relation to any other capital asset,
(i) where the capital asset became the property of the assessee before the 1st day of April, 1981,
means the cost of acquisition of the asset to the assessee or the fair market value of the asset
on the 1st day of April, 1981, at the option of the assessee ;
(ii) where the capital asset became the property of the assessee by any of the modes specified in
sub-section (1) of section 49, and the capital asset became the property of the previous owner

Income Tax

10.106

before the 1st day of April, 1981, means the cost of the capital asset to the previous owner or
the fair market value of the asset on the 1st day of April, 1981, at the option of the assessee ;
(iii) where the capital asset became the property of the assessee on the distribution of the capital
assets of a company on its liquidation and the assessee has been assessed to income-tax under
the head "Capital gains" in respect of that asset undersection 46, means the fair market value
of the asset on the date of distribution ;
(iv) [***]
(v) where the capital asset, being a share or a stock of a company, became the property of the
assessee on
(a) the consolidation and division of all or any of the share capital of the company into
shares of larger amount than its existing shares,
(b) the conversion of any shares of the company into stock,
(c) the re-conversion of any stock of the company into shares,
(d) the sub-division of any of the shares of the company into shares of smaller amount, or
(e) the conversion of one kind of shares of the company into another kind,
means the cost of acquisition of the asset calculated with reference to the cost of acquisition
of the shares or stock from which such asset is derived.
(3) Where the cost for which the previous owner acquired the property cannot be ascertained, the cost of acquisition
to the previous owner means the fair market value on the date on which the capital asset became the property of the
previous owner.
Reference to Valuation Officer.
55A. With a view to ascertaining the fair market value of a capital asset for the purposes of this Chapter, the
Assessing Officer may refer the valuation of capital asset to a Valuation Officer
(a) in a case where the value of the asset as claimed by the assessee is in accordance with the estimate made by a
registered valuer, if the Assessing Officer is of opinion that the value so claimed is at variance with its fair
market value;
(b) in any other case, if the Assessing Officer is of opinion
(i) that the fair market value of the asset exceeds the value of the asset as claimed by the assessee by
more than such percentage6 of the value of the asset as so claimed or by more than such amount 6 as
may be prescribed in this behalf ; or
(ii) that having regard to the nature of the asset and other relevant circumstances, it is necessary so to do,
7

and where any such reference is made, the provisions of sub-sections (2), (3), (4), (5) and (6) of section 16A,
clauses (ha) and (i) of sub-section (1) and sub-sections (3A) and (4) of section 23, sub-section (5) of section 24,
section 34AA, section 35 and section 37 of the Wealth-tax Act, 1957 (27 of 1957), shall with the necessary
modifications, apply in relation to such reference as they apply in relation to a reference made by the Assessing
Officer under sub-section (1) of section 16A of that Act.
Explanation.In this section, "Valuation Officer" has the same meaning, as in clause (r) of section 2 of the Wealthtax Act, 1957 (27 of 1957).

Capital Gains

10.107

You might also like